Cardiac

Réussis tes devoirs et examens dès maintenant avec Quizwiz!

The nurse is caring for four hypertensive clients. Which drug-laboratory value combination should the nurse report immediately to the health care provider? a. Furosemide (Lasix)/potassium: 2.1 mEq/L b. Hydrochlorothiazide (Hydrodiuril)/potassium: 4.2 mEq/L c. Spironolactone (Aldactone)/potassium: 5.1 mEq/L d. Torsemide (Demadex)/sodium: 142 mEq/L

ANS: A Lasix is a loop diuretic and can cause hypokalemia. A potassium level of 2.1 mEq/L is quite low and should be reported immediately. Spironolactone is a potassium-sparing diuretic that can cause hyperkalemia. A potassium level of 5.1 mEq/L is on the high side, but it is not as critical as the low potassium with furosemide. The other two laboratory values are normal.

A client presents to the emergency department with a severely lacerated artery. What is the priority action for the nurse? a. Administer oxygen via non-rebreather mask. b. Ensure the client has a patent airway. c. Prepare to assist with suturing the artery. d. Start two large-bore IVs with normal saline.

ANS: B Airway always takes priority, followed by breathing and circulation. The nurse ensures the client has a patent airway prior to providing any other care measures.

A client with coronary artery disease (CAD) asks the nurse about taking fish oil supplements. What response by the nurse is best? a. "Fish oil is contraindicated with most drugs for CAD." b. "The best source is fish, but pills have benefits too." c. "There is no evidence to support fish oil use with CAD." d. "You can reverse CAD totally with diet and supplements."

ANS: B Omega-3 fatty acids have shown benefit in reducing lipid levels, in reducing the incidence of sudden cardiac death, and for stabilizing atherosclerotic plaque. The best source is fish three times a week or some fish oil supplements. The other options are not accurate.

A nurse administers prescribed adenosine (Adenocard) to a client. Which response should the nurse assess for as the expected therapeutic response? a. Decreased intraocular pressure b. Increased heart rate c. Short period of asystole d. Hypertensive crisis

ANS: C Clients usually respond to adenosine with a short period of asystole, bradycardia, hypotension, dyspnea, and chest pain. Adenosine has no conclusive impact on intraocular pressure.

When planning the care of a patient with an implanted pacemaker, what assessment should the nurse prioritize? A) Core body temperature B) Heart rate and rhythm C) Blood pressure D) Oxygen saturation level

Ans: Heart rate and rhythm Feedback: For patients with pacemakers, close monitoring of the heart rate and rhythm is a priority, even though each of the other listed vital signs must be assessed.

The nurse is caring for a patient who is admitted to the medical unit for the treatment of a venous ulcer in the area of her lateral malleolus that has been unresponsive to treatment. What is the nurse most likely to find during an assessment of this patient's wound? A) Hemorrhage B) Heavy exudate C) Deep wound bed D) Pale-colored wound bed

Ans: Heavy exudate Feedback: Venous ulcerations in the area of the medial or lateral malleolus (gaiter area) are typically large, superficial, and highly exudative. Venous hypertension causes extravasation of blood, which discolors the area of the wound bed. Bleeding is not normally present.

A 17-year-old boy is being treated in the ICU after going into cardiac arrest during a football practice. Diagnostic testing reveals cardiomyopathy as the cause of the arrest. What type of cardiomyopathy is particularly common among young people who appear otherwise healthy? A) Dilated cardiomyopathy (DCM). B) Arrhythmogenic right ventricular cardiomyopathy (ARVC) C) Hypertrophic cardiomyopathy (HCM) D) Restrictive or constrictive cardiomyopathy (RCM)

Ans: Hypertrophic cardiomyopathy (HCM) Feedback: With HCM, cardiac arrest (i.e., sudden cardiac death) may be the initial manifestation in young people, including athletes. DCM, ARVC, and RCM are not typically present in younger adults who appear otherwise healthy.

A patient has had a myocardial infarction and has been diagnosed as having damage to the layer of the heart responsible for the pumping action. You are aware that the damage occurred where? A) Endocardium B) Pericardium C) Myocardium D) Visceral pericardium

Ans: Myocardium Feedback: The myocardium is the layer of the heart responsible for the pumping action.

A critical care nurse is caring for a patient with a hemodynamic monitoring system in place. For what complications should the nurse assess? Select all that apply. A) Pneumothorax B) Infection C) Atelectasis D) Bronchospasm E) Air embolism

Ans: Pneumothorax, Infection, Air embolism Feedback: Complications from use of hemodynamic monitoring systems are uncommon, but can include pneumothorax, infection, and air embolism. Complications of hemodynamic monitoring systems do not include atelectasis or bronchospasm.

The prevention of VTE is an important part of the nursing care of high-risk patients. When providing patient teaching for these high-risk patients, the nurse should advise lifestyle changes, including which of the following? Select all that apply. A) High-protein diet B) Weight loss C) Regular exercise D) Smoking cessation E) Calcium and vitamin D supplementation

Ans: Weight loss, Regular exercise, Smoking cessation Feedback: Patients at risk for VTE should be advised to make lifestyle changes, as appropriate, which may include weight loss, smoking cessation, and regular exercise. Increased protein intake and supplementation with vitamin D and calcium do not address the main risk factors for VTE.

A client with severe sepsis has a serum lactate level of 6.2 mmol/L. The client weighs 250 pounds. To infuse the amount of fluid this client requires in 24 hours, at what rate does the nurse set the IV pump? (Record your answer using a whole number.) ____ mL/hr

ANS: 142 mL/hr The client weighs 250 pounds = 113.63636 kg. The fluid requirement for this client is 30 mL/kg = 3409 mL. To infuse this amount over 24 hours, set the pump at 142 mL/hr (3409/24 = 142).

A nurse is caring for a client who weighs 220 pounds and is started on enoxaparin (Lovenox). How much enoxaparin does the nurse anticipate administering? (Record your answer using a whole number.) _____ mg

ANS: 90 mg The dose of enoxaparin is 1 mg/kg body weight, not to exceed 90 mg. This client weighs 220 pounds (110 kg), and so will get the maximal dose.

A nurse is caring for a client with a nonhealing arterial lower leg ulcer. What action by the nurse is best? a. Consult with the Wound Ostomy Care Nurse. b. Give pain medication prior to dressing changes. c. Maintain sterile technique for dressing changes. d. Prepare the client for eventual amputation.

ANS: A A nonhealing wound needs the expertise of the Wound Ostomy Care Nurse (or Wound Ostomy Continence Nurse). Premedicating prior to painful procedures and maintaining sterile technique are helpful, but if the wound is not healing, more needs to be done. The client may need an amputation, but other options need to be tried first.

A nurse is interested in providing community education and screening on hypertension. In order to reach a priority population, to what target audience should the nurse provide this service? a. African-American churches b. Asian-American groceries c. High school sports camps d. Women's health clinics

ANS: A African Americans in the United States have one of the highest rates of hypertension in the world. The nurse has the potential to reach this priority population by providing services at African-American churches. Although hypertension education and screening are important for all groups, African Americans are the priority population for this intervention.

A nurse is teaching a larger female client about alcohol intake and how it affects hypertension. The client asks if drinking two beers a night is an acceptable intake. What answer by the nurse is best? a. "No, women should only have one beer a day as a general rule." b. "No, you should not drink any alcohol with hypertension." c. "Yes, since you are larger, you can have more alcohol." d. "Yes, two beers per day is an acceptable amount of alcohol."

ANS: A Alcohol intake should be limited to two drinks a day for men and one drink a day for women. A "drink" is classified as one beer, 1.5 ounces of hard liquor, or 5 ounces of wine. Limited alcohol intake is acceptable with hypertension. The woman's size does not matter.

A nurse is caring for a client who is intubated and has an intra-aortic balloon pump. The client is restless and agitated. What action should the nurse perform first for comfort? a. Allow family members to remain at the bedside. b. Ask the family if the client would like a fan in the room. c. Keep the television tuned to the client's favorite channel. d. Speak loudly to the client in case of hearing problems.

ANS: A Allowing the family to remain at the bedside can help calm the client with familiar voices (and faces if the client wakes up). A fan might be helpful but may also spread germs through air movement. The TV should not be kept on all the time to allow for rest. Speaking loudly may agitate the client more.

A nurse assesses clients on a cardiac unit. Which client should the nurse identify as being at greatest risk for the development of left-sided heart failure? a. A 36-year-old woman with aortic stenosis b. A 42-year-old man with pulmonary hypertension c. A 59-year-old woman who smokes cigarettes daily d. A 70-year-old man who had a cerebral vascular accident

ANS: A Although most people with heart failure will have failure that progresses from left to right, it is possible to have left-sided failure alone for a short period. It is also possible to have heart failure that progresses from right to left. Causes of left ventricular failure include mitral or aortic valve disease, coronary artery disease, and hypertension. Pulmonary hypertension and chronic cigarette smoking are risk factors for right ventricular failure. A cerebral vascular accident does not increase the risk of heart failure.

A nurse is teaching a client with heart failure who has been prescribed enalapril (Vasotec). Which statement should the nurse include in this client's teaching? a. "Avoid using salt substitutes." b. "Take your medication with food." c. "Avoid using aspirin-containing products." d. "Check your pulse daily."

ANS: A Angiotensin-converting enzyme (ACE) inhibitors such as enalapril inhibit the excretion of potassium. Hyperkalemia can be a life-threatening side effect, and clients should be taught to limit potassium intake. Salt substitutes are composed of potassium chloride. ACE inhibitors do not need to be taken with food and have no impact on the client's pulse rate. Aspirin is often prescribed in conjunction with ACE inhibitors and is not contraindicated.

A nurse is assessing a client with peripheral artery disease (PAD). The client states walking five blocks is possible without pain. What question asked next by the nurse will give the best information? a. "Could you walk further than that a few months ago?" b. "Do you walk mostly uphill, downhill, or on flat surfaces?" c. "Have you ever considered swimming instead of walking?" d. "How much pain medication do you take each day?"

ANS: A As PAD progresses, it takes less oxygen demand to cause pain. Needing to cut down on activity to be pain free indicates the client's disease is worsening. The other questions are useful, but not as important.

A nurse admits a client who is experiencing an exacerbation of heart failure. Which action should the nurse take first? a. Assess the client's respiratory status. b. Draw blood to assess the client's serum electrolytes. c. Administer intravenous furosemide (Lasix). d. Ask the client about current medications.

ANS: A Assessment of respiratory and oxygenation status is the priority nursing intervention for the prevention of complications. Monitoring electrolytes, administering diuretics, and asking about current medications are important but do not take priority over assessing respiratory status.

A nurse assesses a client with tachycardia. Which clinical manifestation requires immediate intervention by the nurse? a. Mid-sternal chest pain b. Increased urine output c. Mild orthostatic hypotension d. P wave touching the T wave

ANS: A Chest pain, possibly angina, indicates that tachycardia may be increasing the client's myocardial workload and oxygen demand to such an extent that normal oxygen delivery cannot keep pace. This results in myocardial hypoxia and pain. Increased urinary output and mild orthostatic hypotension are not life-threatening conditions and therefore do not require immediate intervention. The P wave touching the T wave indicates significant tachycardia and should be assessed to determine the underlying rhythm and cause; this is an important assessment but is not as critical as chest pain, which indicates cardiac cell death.

A nursing student is caring for a client who had a myocardial infarction. The student is confused because the client states nothing is wrong and yet listens attentively while the student provides education on lifestyle changes and healthy menu choices. What response by the faculty member is best? a. "Continue to educate the client on possible healthy changes." b. "Emphasize complications that can occur with noncompliance." c. "Tell the client that denial is normal and will soon go away." d. "You need to make sure the client understands this illness."

ANS: A Clients are often in denial after a coronary event. The client who seems to be in denial but is compliant with treatment may be using a healthy form of coping that allows time to process the event and start to use problem-focused coping. The student should not discourage this type of denial and coping, but rather continue providing education in a positive manner. Emphasizing complications may make the client defensive and more anxious. Telling the client that denial is normal is placing too much attention on the process. Forcing the client to verbalize understanding of the illness is also potentially threatening to the client

A client has peripheral arterial disease (PAD). What statement by the client indicates misunderstanding about self-management activities? a. "I can use a heating pad on my legs if it's set on low." b. "I should not cross my legs when sitting or lying down." c. "I will go out and buy some warm, heavy socks to wear." d. "It's going to be really hard but I will stop smoking."

ANS: A Clients with PAD should never use heating pads as skin sensitivity is diminished and burns can result. The other statements show good understanding of self-management.

A client had an inferior wall myocardial infarction (MI). The nurse notes the client's cardiac rhythm as shown below: What action by the nurse is most important? a. Assess the client's blood pressure and level of consciousness. b. Call the health care provider or the Rapid Response Team. c. Obtain a permit for an emergency temporary pacemaker insertion. d. Prepare to administer antidysrhythmic medication.

ANS: A Clients with an inferior wall MI often have bradycardia and blocks that lead to decreased perfusion, as seen in this ECG strip showing sinus bradycardia. The nurse should first assess the client's hemodynamic status, including vital signs and level of consciousness. The client may or may not need the Rapid Response Team, a temporary pacemaker, or medication; there is no indication of this in the question.

A client is in the clinic a month after having a myocardial infarction. The client reports sleeping well since moving into the guest bedroom. What response by the nurse is best? a. "Do you have any concerns about sexuality?" b. "I'm glad to hear you are sleeping well now." c. "Sleep near your spouse in case of emergency." d. "Why would you move into the guest room?"

ANS: A Concerns about resuming sexual activity are common after cardiac events. The nurse should gently inquire if this is the issue. While it is good that the client is sleeping well, the nurse should investigate the reason for the move. The other two responses are likely to cause the client to be defensive.

A nurse cares for a client with right-sided heart failure. The client asks, "Why do I need to weigh myself every day?" How should the nurse respond? a. "Weight is the best indication that you are gaining or losing fluid." b. "Daily weights will help us make sure that you're eating properly." c. "The hospital requires that all inpatients be weighed daily." d. "You need to lose weight to decrease the incidence of heart failure."

ANS: A Daily weights are needed to document fluid retention or fluid loss. One liter of fluid equals 2.2 pounds. The other responses do not address the importance of monitoring fluid retention or loss.

A nurse cares for an older adult client with heart failure. The client states, "I don't know what to do. I don't want to be a burden to my daughter, but I can't do it alone. Maybe I should die." How should the nurse respond? a. "Would you like to talk more about this?" b. "You are lucky to have such a devoted daughter." c. "It is normal to feel as though you are a burden." d. "Would you like to meet with the chaplain?"

ANS: A Depression can occur in clients with heart failure, especially older adults. Having the client talk about his or her feelings will help the nurse focus on the actual problem. Open-ended statements allow the client to respond safely and honestly. The other options minimize the client's concerns and do not allow the nurse to obtain more information to provide client-centered care.

A client is on a dopamine infusion via a peripheral line. What action by the nurse takes priority for safety? a. Assess the IV site hourly. b. Monitor the pedal pulses. c. Monitor the client's vital signs. d. Obtain consent for a central line.

ANS: A Dopamine should be infused through a central line to prevent extravasation and necrosis of tissue. If it needs to be run peripherally, the nurse assesses the site hourly for problems. When the client is getting the central line, ensuring informed consent is on the chart is a priority. But at this point, the client has only a peripheral line, so caution must be taken to preserve the integrity of the client's integumentary system. Monitoring pedal pulses and vital signs give indications as to how well the drug is working.

An emergency room nurse obtains the health history of a client. Which statement by the client should alert the nurse to the occurrence of heart failure? a. "I get short of breath when I climb stairs." b. "I see halos floating around my head." c. "I have trouble remembering things." d. "I have lost weight over the past month."

ANS: A Dyspnea on exertion is an early manifestation of heart failure and is associated with an activity such as stair climbing. The other findings are not specific to early occurrence of heart failure.

A client had a femoropopliteal bypass graft with a synthetic graft. What action by the nurse is most important to prevent wound infection? a. Appropriate hand hygiene before giving care b. Assessing the client's temperature every 4 hours c. Clean technique when changing dressings d. Monitoring the client's daily white blood cell count

ANS: A Hand hygiene is the best way to prevent infections in hospitalized clients. Dressing changes should be done with sterile technique. Assessing vital signs and white blood cell count will not prevent infection.

A client is in shock and the nurse prepares to administer insulin for a blood glucose reading of 208 mg/dL. The spouse asks why the client needs insulin as the client is not a diabetic. What response by the nurse is best? a. "High glucose is common in shock and needs to be treated." b. "Some of the medications we are giving are to raise blood sugar." c. "The IV solution has lots of glucose, which raises blood sugar." d. "The stress of this illness has made your spouse a diabetic."

ANS: A High glucose readings are common in shock, and best outcomes are the result of treating them and maintaining glucose readings in the normal range. Medications and IV solutions may raise blood glucose levels, but this is not the most accurate answer. The stress of the illness has not "made" the client diabetic.

A client is taking warfarin (Coumadin) and asks the nurse if taking St. John's wort is acceptable. What response by the nurse is best? a. "No, it may interfere with the warfarin." b. "There isn't any information about that." c. "Why would you want to take that?" d. "Yes, it is a good supplement for you."

ANS: A Many foods and drugs interfere with warfarin, St. John's wort being one of them. The nurse should advise the client against taking it. The other answers are not accurate.

A client is receiving norepinephrine (Levophed) for shock. What assessment finding best indicates a therapeutic effect from this drug? a. Alert and oriented, answering questions b. Client denial of chest pain or chest pressure c. IV site without redness or swelling d. Urine output of 30 mL/hr for 2 hours

ANS: A Normal cognitive function is a good indicator that the client is receiving the benefits of norepinephrine. The brain is very sensitive to changes in oxygenation and perfusion. Norepinephrine can cause chest pain as an adverse reaction, so the absence of chest pain does not indicate therapeutic effect. The IV site is normal. The urine output is normal, but only minimally so.

A nurse assesses a client who is recovering from a myocardial infarction. The client's pulmonary artery pressure reading is 25/12 mm Hg. Which action should the nurse take first? a. Compare the results with previous pulmonary artery pressure readings. b. Increase the intravenous fluid rate because these readings are low. c. Immediately notify the health care provider of the elevated pressures. d. Document the finding in the client's chart as the only action.

ANS: A Normal pulmonary artery pressures range from 15 to 26 mm Hg for systolic and from 5 to 15 mm Hg for diastolic. Although this client's readings are within normal limits, the nurse needs to assess any trends that may indicate a need for medical treatment to prevent complications. There is no need to increase intravenous fluids or notify the provider.

An older adult is on cardiac monitoring after a myocardial infarction. The client shows frequent dysrhythmias. What action by the nurse is most appropriate? a. Assess for any hemodynamic effects of the rhythm. b. Prepare to administer antidysrhythmic medication. c. Notify the provider or call the Rapid Response Team. d. Turn the alarms off on the cardiac monitor.

ANS: A Older clients may have dysrhythmias due to age-related changes in the cardiac conduction system. They may have no significant hemodynamic effects from these changes. The nurse should first assess for the effects of the dysrhythmia before proceeding further. The alarms on a cardiac monitor should never be shut off. The other two actions may or may not be needed.

A nurse teaches a client who experiences occasional premature atrial contractions (PACs) accompanied by palpitations that resolve spontaneously without treatment. Which statement should the nurse include in this client's teaching? a. "Minimize or abstain from caffeine." b. "Lie on your side until the attack subsides." c. "Use your oxygen when you experience PACs." d. "Take amiodarone (Cordarone) daily to prevent PACs."

ANS: A PACs usually have no hemodynamic consequences. For a client experiencing infrequent PACs, the nurse should explore possible lifestyle causes, such as excessive caffeine intake and stress. Lying on the side will not prevent or resolve PACs. Oxygen is not necessary. Although medications may be needed to control symptomatic dysrhythmias, for infrequent PACs, the client first should try lifestyle changes to control them.

A nurse assesses a client who has aortic regurgitation. In which location in the illustration shown below should the nurse auscultate to best hear a cardiac murmur related to aortic regurgitation? a. Location A b. Location B c. Location C d. Location D

ANS: A The aortic valve is auscultated in the second intercostal space just to the right of the sternum.

A nurse cares for a client with infective endocarditis. Which infection control precautions should the nurse use? a. Standard Precautions b. Bleeding precautions c. Reverse isolation d. Contact isolation

ANS: A The client with infective endocarditis does not pose any specific threat of transmitting the causative organism. Standard Precautions should be used. Bleeding precautions or reverse or contact isolation is not necessary.

A client has been diagnosed with a deep vein thrombosis and is to be discharged on warfarin (Coumadin). The client is adamant about refusing the drug because "it's dangerous." What action by the nurse is best? a. Assess the reason behind the client's fear. b. Remind the client about laboratory monitoring. c. Tell the client drugs are safer today than before. d. Warn the client about consequences of noncompliance.

ANS: A The first step is to assess the reason behind the client's fear, which may be related to the experience of someone the client knows who took warfarin. If the nurse cannot address the specific rationale, teaching will likely be unsuccessful. Laboratory monitoring once every few weeks may not make the client perceive the drug to be safe. General statements like "drugs are safer today" do not address the root cause of the problem. Warning the client about possible consequences of not taking the drug is not therapeutic and is likely to lead to an adversarial relationship.

A nurse prepares to discharge a client with cardiac dysrhythmia who is prescribed home health care services. Which priority information should be communicated to the home health nurse upon discharge? a. Medication reconciliation b. Immunization history c. Religious beliefs d. Nutrition preferences

ANS: A The home health nurse needs to know current medications the client is taking to ensure assessment, evaluation, and further education related to these medications. The other information will not assist the nurse to develop a plan of care for the client.

A client in shock is apprehensive and slightly confused. What action by the nurse is best? a. Offer to remain with the client for awhile. b. Prepare to administer antianxiety medication. c. Raise all four siderails on the client's bed. d. Tell the client everything possible is being done.

ANS: A The nurse's presence will be best to reassure this client. Antianxiety medication is not warranted as this will lower the client's blood pressure. Using all four siderails on a hospital bed is considered a restraint in most facilities, although the nurse should ensure the client's safety. Telling a confused client that everything is being done is not the most helpful response.

A client has been brought to the emergency department after being shot multiple times. What action should the nurse perform first? a. Apply personal protective equipment. b. Notify local law enforcement officials. c. Obtain "universal" donor blood. d. Prepare the client for emergency surgery.

ANS: A The nurse's priority is to care for the client. Since the client has gunshot wounds and is bleeding, the nurse applies personal protective equipment (i.e., gloves) prior to care. This takes priority over calling law enforcement. Requesting blood bank products can be delegated. The nurse may or may not have to prepare the client for emergency surgery.

While assessing a client on a cardiac unit, a nurse identifies the presence of an S3 gallop. Which action should the nurse take next? a. Assess for symptoms of left-sided heart failure. b. Document this as a normal finding. c. Call the health care provider immediately. d. Transfer the client to the intensive care unit.

ANS: A The presence of an S3 gallop is an early diastolic filling sound indicative of increasing left ventricular pressure and left ventricular failure. The other actions are not warranted.

A nurse is working with a client who takes atorvastatin (Lipitor). The client's recent laboratory results include a blood urea nitrogen (BUN) of 33 mg/dL and creatinine of 2.8 mg/dL. What action by the nurse is best? a. Ask if the client eats grapefruit. b. Assess the client for dehydration. c. Facilitate admission to the hospital. d. Obtain a random urinalysis.

ANS: A There is a drug-food interaction between statins and grapefruit that can lead to acute kidney failure. This client has elevated renal laboratory results, indicating some degree of kidney involvement. The nurse should assess if the client eats grapefruit or drinks grapefruit juice. Dehydration can cause the BUN to be elevated, but the elevation in creatinine is more specific for a kidney injury. The client does not necessarily need to be admitted. A urinalysis may or may not be ordered.

A client with a history of heart failure and hypertension is in the clinic for a follow-up visit. The client is on lisinopril (Prinivil) and warfarin (Coumadin). The client reports new-onset cough. What action by the nurse is most appropriate? a. Assess the client's lung sounds and oxygenation. b. Instruct the client on another antihypertensive. c. Obtain a set of vital signs and document them. d. Remind the client that cough is a side effect of Prinivil.

ANS: A This client could be having an exacerbation of heart failure or be experiencing a side effect of lisinopril (and other angiotensin-converting enzyme inhibitors). The nurse should assess the client's lung sounds and other signs of oxygenation first. The client may or may not need to switch antihypertensive medications. Vital signs and documentation are important, but the nurse should assess the respiratory system first. If the cough turns out to be a side effect, reminding the client is appropriate, but then more action needs to be taken.

The nurse gets the hand-off report on four clients. Which client should the nurse assess first? a. Client with a blood pressure change of 128/74 to 110/88 mm Hg b. Client with oxygen saturation unchanged at 94% c. Client with a pulse change of 100 to 88 beats/min d. Client with urine output of 40 mL/hr for the last 2 hours

ANS: A This client has a falling systolic blood pressure, rising diastolic blood pressure, and narrowing pulse pressure, all of which may be indications of the progressive stage of shock. The nurse should assess this client first. The client with the unchanged oxygen saturation is stable at this point. Although the client with a change in pulse has a slower rate, it is not an indicator of shock since the pulse is still within the normal range; it may indicate the client's pain or anxiety has been relieved, or he or she is sleeping or relaxing. A urine output of 40 mL/hr is only slightly above the normal range, which is 30 mL/hr.

The nurse is assessing a client on admission to the hospital. The client's leg appears as shown below: What action by the nurse is best? a. Assess the client's ankle-brachial index. b. Elevate the client's leg above the heart. c. Obtain an ice pack to provide comfort. d. Prepare to teach about heparin sodium.

ANS: A This client has dependent rubor, a classic finding in peripheral arterial disease. The nurse should measure the client's ankle-brachial index. Elevating the leg above the heart will further impede arterial blood flow. Ice will cause vasoconstriction, also impeding circulation and perhaps causing tissue injury. Heparin sodium is not the drug of choice for this condition.

A nurse supervises an unlicensed assistive personnel (UAP) applying electrocardiographic monitoring. Which statement should the nurse provide to the UAP related to this procedure? a. "Clean the skin and clip hairs if needed." b. "Add gel to the electrodes prior to applying them." c. "Place the electrodes on the posterior chest." d. "Turn off oxygen prior to monitoring the client."

ANS: A To ensure the best signal transmission, the skin should be clean and hairs clipped. Electrodes should be placed on the anterior chest, and no additional gel is needed. Oxygen has no impact on electrocardiographic monitoring.

A nurse cares for a client who is on a cardiac monitor. The monitor displayed the rhythm shown below: Which action should the nurse take first? a. Assess airway, breathing, and level of consciousness. b. Administer an amiodarone bolus followed by a drip. c. Cardiovert the client with a biphasic defibrillator. d. Begin cardiopulmonary resuscitation (CPR).

ANS: A Ventricular tachycardia occurs with repetitive firing of an irritable ventricular ectopic focus, usually at a rate of 140 to 180 beats/min or more. Ventricular tachycardia is a lethal dysrhythmia. The nurse should first assess if the client is alert and breathing. Then the nurse should call a Code Blue and begin CPR. If this client is pulseless, the treatment of choice is defibrillation. Amiodarone is the antidysrhythmic of choice, but it is not the first action.

A nurse assesses a client who had a myocardial infarction and is hypotensive. Which additional assessment finding should the nurse expect? a. Heart rate of 120 beats/min b. Cool, clammy skin c. Oxygen saturation of 90% d. Respiratory rate of 8 breaths/min

ANS: A When a client experiences hypotension, baroreceptors in the aortic arch sense a pressure decrease in the vessels. The parasympathetic system responds by lessening the inhibitory effect on the sinoatrial node. This results in an increase in heart rate and respiratory rate. This tachycardia is an early response and is seen even when blood pressure is not critically low. An increased heart rate and respiratory rate will compensate for the low blood pressure and maintain oxygen saturations and perfusion. The client may not be able to compensate for long, and decreased oxygenation and cool, clammy skin will occur later.

A nurse assesses a client who is recovering from a heart transplant. Which assessment findings should alert the nurse to the possibility of heart transplant rejection? (Select all that apply.) a. Shortness of breath b. Abdominal bloating c. New-onset bradycardia d. Increased ejection fraction e. Hypertension

ANS: A, B, C Clinical manifestations of heart transplant rejection include shortness of breath, fatigue, fluid gain, abdominal bloating, new-onset bradycardia, hypotension, atrial fibrillation or flutter, decreased activity tolerance, and decreased ejection fraction.

A nurse is caring for a client with a history of renal insufficiency who is scheduled for a cardiac catheterization. Which actions should the nurse take prior to the catheterization? (Select all that apply.) a. Assess for allergies to iodine. b. Administer intravenous fluids. c. Assess blood urea nitrogen (BUN) and creatinine results. d. Insert a Foley catheter. e. Administer a prophylactic antibiotic. f. Insert a central venous catheter.

ANS: A, B, C If the client has kidney disease (as indicated by BUN and creatinine results), fluids and Mucomyst may be given 12 to 24 hours before the procedure for renal protection. The client should be assessed for allergies to iodine, including shellfish; the contrast medium used during the catheterization contains iodine. A Foley catheter and central venous catheter are not required for the procedure and would only increase the client's risk for infection. Prophylactic antibiotics are not administered prior to a cardiac catheterization.

A nurse is caring for a client with a nonhealing arterial ulcer. The physician has informed the client about possibly needing to amputate the client's leg. The client is crying and upset. What actions by the nurse are best? (Select all that apply.) a. Ask the client to describe his or her current emotions. b. Assess the client for support systems and family. c. Offer to stay with the client if he or she desires. d. Relate how smoking contributed to this situation. e. Tell the client that many people have amputations.

ANS: A, B, C When a client is upset, the nurse should offer self by remaining with the client if desired. Other helpful measures include determining what and whom the client has for support systems and asking the client to describe what he or she is feeling. Telling the client how smoking has led to this situation will only upset the client further and will damage the therapeutic relationship. Telling the client that many people have amputations belittles the client's feelings.

The nurse caring frequently for older adults in the hospital is aware of risk factors that place them at a higher risk for shock. For what factors would the nurse assess? (Select all that apply.) a. Altered mobility/immobility b. Decreased thirst response c. Diminished immune response d. Malnutrition e. Overhydration

ANS: A, B, C, D Immobility, decreased thirst response, diminished immune response, and malnutrition can place the older adult at higher risk of developing shock. Overhydration is not a common risk factor for shock.

A nursing student planning to teach clients about risk factors for coronary artery disease (CAD) would include which topics? (Select all that apply.) a. Advanced age b. Diabetes c. Ethnic background d. Medication use e. Smoking

ANS: A, B, C, E Age, diabetes, ethnic background, and smoking are all risk factors for developing CAD; medication use is not.

A nurse is teaching a client with premature ectopic beats. Which education should the nurse include in this client's teaching? (Select all that apply.) a. Smoking cessation b. Stress reduction and management c. Avoiding vagal stimulation d. Adverse effects of medications e. Foods high in potassium

ANS: A, B, D A client who has premature beats or ectopic rhythms should be taught to stop smoking, manage stress, take medications as prescribed, and report adverse effects of medications. Clients with premature beats are not at risk for vasovagal attacks or potassium imbalances.

A nurse assesses clients on a cardiac unit. Which clients should the nurse identify as at greatest risk for the development of acute pericarditis? (Select all that apply.) a. A 36-year-old woman with systemic lupus erythematosus (SLE) b. A 42-year-old man recovering from coronary artery bypass graft surgery c. A 59-year-old woman recovering from a hysterectomy d. An 80-year-old man with a bacterial infection of the respiratory tract e. An 88-year-old woman with a stage III sacral ulcer

ANS: A, B, D Acute pericarditis is most commonly associated acute exacerbations of systemic connective tissue disease, including SLE; with Dressler's syndrome, or inflammation of the cardiac sac after cardiac surgery or a myocardial infarction; and with infective organisms, including bacterial, viral, and fungal infections. Abdominal and reproductive surgeries and pressure ulcers do not increase clients' risk for acute pericarditis.

A nurse is caring for a client on IV infusion of heparin. What actions does this nurse include in the client's plan of care? (Select all that apply.) a. Assess the client for bleeding. b. Monitor the daily activated partial thromboplastin time (aPTT) results. c. Stop the IV for aPTT above baseline. d. Use an IV pump for the infusion. e. Weigh the client daily on the same scale

ANS: A, B, D Assessing for bleeding, monitoring aPTT, and using an IV pump for the infusion are all important safety measures for heparin to prevent injury from bleeding. The aPTT needs to be 1.5 to 2 times normal in order to demonstrate that the heparin is therapeutic. Weighing the client is not related.

A nurse prepares to discharge a client who has heart failure. Based on the Heart Failure Core Measure Set, which actions should the nurse complete prior to discharging this client? (Select all that apply.) a. Teach the client about dietary restrictions. b. Ensure the client is prescribed an angiotensin-converting enzyme (ACE) inhibitor. c. Encourage the client to take a baby aspirin each day. d. Confirm that an echocardiogram has been completed. e. Consult a social worker for additional resources.

ANS: A, B, D The Heart Failure Core Measure Set includes discharge instructions on diet, activity, medications, weight monitoring and plan for worsening symptoms, evaluation of left ventricular systolic function (usually with an echocardiogram), and prescribing an ACE inhibitor or angiotensin receptor blocker. Aspirin is not part of the Heart Failure Core Measure Set and is usually prescribed for clients who experience a myocardial infarction. Although the nurse may consult the social worker or case manager for additional resources, this is not part of the Core Measures.

A client has been bedridden for several days after major abdominal surgery. What action does the nurse delegate to the unlicensed assistive personnel (UAP) for deep vein thrombosis (DVT) prevention? (Select all that apply.) a. Apply compression stockings. b. Assist with ambulation. c. Encourage coughing and deep breathing. d. Offer fluids frequently. e. Teach leg exercises.

ANS: A, B, D The UAP can apply compression stockings, assist with ambulation, and offer fluids frequently to help prevent DVT. The UAP can also encourage the client to do pulmonary exercises, but these do not decrease the risk of DVT. Teaching is a nursing function.

A nurse prepares to discharge a client who has heart failure. Which questions should the nurse ask to ensure this client's safety prior to discharging home? (Select all that apply.) a. "Are your bedroom and bathroom on the first floor?" b. "What social support do you have at home?" c. "Will you be able to afford your oxygen therapy?" d. "What spiritual beliefs may impact your recovery?" e. "Are you able to accurately weigh yourself at home?"

ANS: A, B, D To ensure safety upon discharge, the nurse should assess for structural barriers to functional ability, such as stairs. The nurse should also assess the client's available social support, which may include family, friends, and home health services. The client's ability to adhere to medication and treatments, including daily weights, should also be reviewed. The other questions do not address the client's safety upon discharge.

A nursing student studying acute coronary syndromes learns that the pain of a myocardial infarction (MI) differs from stable angina in what ways? (Select all that apply.) a. Accompanied by shortness of breath b. Feelings of fear or anxiety c. Lasts less than 15 minutes d. No relief from taking nitroglycerin e. Pain occurs without known cause

ANS: A, B, D, E The pain from an MI is often accompanied by shortness of breath and fear or anxiety. It lasts longer than 15 minutes and is not relieved by nitroglycerin. It occurs without a known cause such as exertion.

A nurse is assessing a client with left-sided heart failure. For which clinical manifestations should the nurse assess? (Select all that apply.) a. Pulmonary crackles b. Confusion, restlessness c. Pulmonary hypertension d. Dependent edema e. Cough that worsens at night

ANS: A, B, E Left-sided heart failure occurs with a decrease in contractility of the heart or an increase in afterload. Most of the signs will be noted in the respiratory system. Right-sided heart failure occurs with problems from the pulmonary vasculature onward including pulmonary hypertension. Signs will be noted before the right atrium or ventricle including dependent edema.

A nurse teaches a client with a new permanent pacemaker. Which instructions should the nurse include in this client's teaching? (Select all that apply.) a. "Until your incision is healed, do not submerge your pacemaker. Only take showers." b. "Report any pulse rates lower than your pacemaker settings." c. "If you feel weak, apply pressure over your generator." d. "Have your pacemaker turned off before having magnetic resonance imaging (MRI)." e. "Do not lift your left arm above the level of your shoulder for 8 weeks."

ANS: A, B, E The client should not submerge in water until the site has healed; after the incision is healed, the client may take showers or baths without concern for the pacemaker. The client should be instructed to report changes in heart rate or rhythm, such as rates lower than the pacemaker setting or greater than 100 beats/min. The client should be advised of restrictions on physical activity for 8 weeks to allow the pacemaker to settle in place. The client should never apply pressure over the generator and should avoid tight clothing. The client should never have MRI because, whether turned on or off, the pacemaker contains metal. The client should be advised to inform all health care providers that he or she has a pacemaker.

The nurse is caring for a client with suspected severe sepsis. What does the nurse prepare to do within 3 hours of the client being identified as being at risk? (Select all that apply.) a. Administer antibiotics. b. Draw serum lactate levels. c. Infuse vasopressors. d. Measure central venous pressure. e. Obtain blood cultures.

ANS: A, B, E Within the first 3 hours of suspecting severe sepsis, the nurse should draw (or facilitate) serum lactate levels, obtain blood cultures (or other cultures), and administer antibiotics (after the cultures have been obtained). Infusing vasopressors and measuring central venous pressure are actions that should occur within the first 6 hours.

A nurse evaluates laboratory results for a client with heart failure. Which results should the nurse expect? (Select all that apply.) a. Hematocrit: 32.8% b. Serum sodium: 130 mEq/L c. Serum potassium: 4.0 mEq/L d. Serum creatinine: 1.0 mg/dL e. Proteinuria f. Microalbuminuria

ANS: A, B, E, F A hematocrit of 32.8% is low (should be 42.6%), indicating a dilutional ratio of red blood cells to fluid. A serum sodium of 130 mEq/L is low because of hemodilution. Microalbuminuria and proteinuria are present, indicating a decrease in renal filtration. These are early warning signs of decreased compliance of the heart. The potassium level is on the high side of normal and the serum creatinine level is normal.

The student nurse studying shock understands that the common manifestations of this condition are directly related to which problems? (Select all that apply.) a. Anaerobic metabolism b. Hyperglycemia c. Hypotension d. Impaired renal perfusion e. Increased perfusion

ANS: A, C The common manifestations of shock, no matter the cause, are directly related to the effects of anaerobic metabolism and hypotension. Hyperglycemia, impaired renal function, and increased perfusion are not manifestations of shock.

A nurse collaborates with an unlicensed assistive personnel (UAP) to provide care for a client with congestive heart failure. Which instructions should the nurse provide to the UAP when delegating care for this client? (Select all that apply.) a. "Reposition the client every 2 hours." b. "Teach the client to perform deep-breathing exercises." c. "Accurately record intake and output." d. "Use the same scale to weigh the client each morning." e. "Place the client on oxygen if the client becomes short of breath."

ANS: A, C, D The UAP should reposition the client every 2 hours to improve oxygenation and prevent atelectasis. The UAP can also accurately record intake and output, and use the same scale to weigh the client each morning before breakfast. UAPs are not qualified to teach clients or assess the need for and provide oxygen therapy.

The nurse caring for hospitalized clients includes which actions on their care plans to reduce the possibility of the clients developing shock? (Select all that apply.) a. Assessing and identifying clients at risk b. Monitoring the daily white blood cell count c. Performing proper hand hygiene d. Removing invasive lines as soon as possible e. Using aseptic technique during procedures

ANS: A, C, D, E Assessing and identifying clients at risk for shock is probably the most critical action the nurse can take to prevent shock from occurring. Proper hand hygiene, using aseptic technique, and removing IV lines and catheters are also important actions to prevent shock. Monitoring laboratory values does not prevent shock but can indicate a change.

A client is being discharged on warfarin (Coumadin) therapy. What discharge instructions is the nurse required to provide? (Select all that apply.) a. Dietary restrictions b. Driving restrictions c. Follow-up laboratory monitoring d. Possible drug-drug interactions e. Reason to take medication

ANS: A, C, D, E The Joint Commission's Core Measures state that clients being discharged on warfarin need instruction on follow-up monitoring, dietary restrictions, drug-drug interactions, and reason for compliance. Driving is typically not restricted.

A nurse reviews a client's laboratory results. Which findings should alert the nurse to the possibility of atherosclerosis? (Select all that apply.) a. Total cholesterol: 280 mg/dL b. High-density lipoprotein cholesterol: 50 mg/dL c. Triglycerides: 200 mg/dL d. Serum albumin: 4 g/dL e. Low-density lipoprotein cholesterol: 160 mg/dL

ANS: A, C, E A lipid panel is often used to screen for cardiovascular risk. Total cholesterol, triglycerides, and low-density lipoprotein cholesterol levels are all high, indicating higher risk for cardiovascular disease. High-density lipoprotein cholesterol is within the normal range for both males and females. Serum albumin is not assessed for atherosclerosis.

A nurse cares for a client who is recovering from a right-sided heart catheterization. For which complications of this procedure should the nurse assess? (Select all that apply.) a. Thrombophlebitis b. Stroke c. Pulmonary embolism d. Myocardial infarction e. Cardiac tamponade

ANS: A, C, E Complications from a right-sided heart catheterization include thrombophlebitis, pulmonary embolism, and vagal response. Cardiac tamponade is a risk of both right- and left-sided heart catheterizations. Stroke and myocardial infarction are complications of left-sided heart catheterizations.

A nurse is caring for a client who had coronary artery bypass grafting yesterday. What actions does the nurse delegate to the unlicensed assistive personnel (UAP)? (Select all that apply.) a. Assist the client to the chair for meals and to the bathroom. b. Encourage the client to use the spirometer every 4 hours. c. Ensure the client wears TED hose or sequential compression devices. d. Have the client rate pain on a 0-to-10 scale and report to the nurse. e. Take and record a full set of vital signs per hospital protocol.

ANS: A, C, E The nurse can delegate assisting the client to get up in the chair or ambulate to the bathroom, applying TEDs or sequential compression devices, and taking/recording vital signs. The spirometer should be used every hour the day after surgery. Assessing pain using a 0-to-10 scale is a nursing assessment, although if the client reports pain, the UAP should inform the nurse so a more detailed assessment is done.

The nurse working in the emergency department knows that which factors are commonly related to aneurysm formation? (Select all that apply.) a. Atherosclerosis b. Down syndrome c. Frequent heartburn d. History of hypertension e. History of smoking

ANS: A, D, E Atherosclerosis, hypertension, hyperlipidemia, and smoking are the most common related factors. Down syndrome and heartburn have no relation to aneurysm formation.

A nurse cares for a client with congestive heart failure who has a regular cardiac rhythm of 128 beats/min. For which physiologic alterations should the nurse assess? (Select all that apply.) a. Decrease in cardiac output b. Increase in cardiac output c. Decrease in blood pressure d. Increase in blood pressure e. Decrease in urine output f. Increase in urine output

ANS: A, D, E Elevated heart rates in a healthy client initially cause blood pressure and cardiac output to increase. However, in a client who has congestive heart failure or a client with long-term tachycardia, ventricular filling time, cardiac output, and blood pressure eventually decrease. As cardiac output and blood pressure decrease, urine output will fall

After teaching a client with congestive heart failure (CHF), the nurse assesses the client's understanding. Which client statements indicate a correct understanding of the teaching related to nutritional intake? (Select all that apply.) a. "I'll read the nutritional labels on food items for salt content." b. "I will drink at least 3 liters of water each day." c. "Using salt in moderation will reduce the workload of my heart." d. "I will eat oatmeal for breakfast instead of ham and eggs." e. "Substituting fresh vegetables for canned ones will lower my salt intake."

ANS: A, D, E Nutritional therapy for a client with CHF is focused on decreasing sodium and water retention to decrease the workload of the heart. The client should be taught to read nutritional labels on all food items, omit table salt and foods high in sodium (e.g., ham and canned foods), and limit water intake to a normal 2 L/day.

A client is in the early stages of shock and is restless. What comfort measures does the nurse delegate to the nursing student? (Select all that apply.) a. Bringing the client warm blankets b. Giving the client hot tea to drink c. Massaging the client's painful legs d. Reorienting the client as needed e. Sitting with the client for reassurance

ANS: A, D, E The student can bring the client warm blankets, reorient the client as needed to decrease anxiety, and sit with the client for reassurance. The client should be NPO at this point, so hot tea is prohibited. Massaging the legs is not recommended as this can dislodge any clots present, which may lead to pulmonary embolism.

A nurse teaches a client with heart failure about energy conservation. Which statement should the nurse include in this client's teaching? a. "Walk until you become short of breath, and then walk back home." b. "Gather everything you need for a chore before you begin." c. "Pull rather than push or carry items heavier than 5 pounds." d. "Take a walk after dinner every day to build up your strength."

ANS: B A client who has heart failure should be taught to conserve energy. Gathering all supplies needed for a chore at one time decreases the amount of energy needed. The client should not walk until becoming short of breath because he or she may not make it back home. Pushing a cart takes less energy than pulling or lifting. Although walking after dinner may help the client, the nurse should teach the client to complete activities when he or she has the most energy. This is usually in the morning.

A nurse is caring for a client with a deep vein thrombosis (DVT). What nursing assessment indicates a priority outcome has been met? a. Ambulates with assistance b. Oxygen saturation of 98% c. Pain of 2/10 after medication d. Verbalizing risk factors

ANS: B A critical complication of DVT is pulmonary embolism. A normal oxygen saturation indicates that this has not occurred. The other assessments are also positive, but not the priority.

A nurse is caring for several clients at risk for shock. Which laboratory value requires the nurse to communicate with the health care provider? a. Creatinine: 0.9 mg/dL b. Lactate: 6 mmol/L c. Sodium: 150 mEq/L d. White blood cell count: 11,000/mm3

ANS: B A lactate level of 6 mmol/L is high and is indicative of possible shock. A creatinine level of 0.9 mg/dL is normal. A sodium level of 150 mEq/L is high, but that is not related directly to shock. A white blood cell count of 11,000/mm3 is slightly high but is not as critical as the lactate level.

A client has intra-arterial blood pressure monitoring after a myocardial infarction. The nurse notes the client's heart rate has increased from 88 to 110 beats/min, and the blood pressure dropped from 120/82 to 100/60 mm Hg. What action by the nurse is most appropriate? a. Allow the client to rest quietly. b. Assess the client for bleeding. c. Document the findings in the chart. d. Medicate the client for pain.

ANS: B A major complication related to intra-arterial blood pressure monitoring is hemorrhage from the insertion site. Since these vital signs are out of the normal range, are a change, and are consistent with blood loss, the nurse should assess the client for any bleeding associated with the arterial line. The nurse should document the findings after a full assessment. The client may or may not need pain medication and rest; the nurse first needs to rule out any emergent bleeding.

After administering newly prescribed captopril (Capoten) to a client with heart failure, the nurse implements interventions to decrease complications. Which priority intervention should the nurse implement for this client? a. Provide food to decrease nausea and aid in absorption. b. Instruct the client to ask for assistance when rising from bed. c. Collaborate with unlicensed assistive personnel to bathe the client. d. Monitor potassium levels and check for symptoms of hypokalemia.

ANS: B Administration of the first dose of angiotensin-converting enzyme (ACE) inhibitors is often associated with hypotension, usually termed first-dose effect. The nurse should instruct the client to seek assistance before arising from bed to prevent injury from postural hypotension. ACE inhibitors do not need to be taken with food. Collaboration with unlicensed assistive personnel to provide hygiene is not a priority. The client should be encouraged to complete activities of daily living as independently as possible. The nurse should monitor for hyperkalemia, not hypokalemia, especially if the client has renal insufficiency secondary to heart failure.

A client received tissue plasminogen activator (t-PA) after a myocardial infarction and now is on an intravenous infusion of heparin. The client's spouse asks why the client needs this medication. What response by the nurse is best? a. "The t-PA didn't dissolve the entire coronary clot." b. "The heparin keeps that artery from getting blocked again." c. "Heparin keeps the blood as thin as possible for a longer time." d. "The heparin prevents a stroke from occurring as the t-PA wears off."

ANS: B After the original intracoronary clot has dissolved, large amounts of thrombin are released into the bloodstream, increasing the chance of the vessel reoccluding. The other statements are not accurate. Heparin is not a "blood thinner," although laypeople may refer to it as such.

A client arrives in the emergency department after being in a car crash with fatalities. The client has a nearly amputated leg that is bleeding profusely. What action by the nurse takes priority? a. Apply direct pressure to the bleeding. b. Ensure the client has a patent airway. c. Obtain consent for emergency surgery. d. Start two large-bore IV catheters.

ANS: B Airway is the priority, followed by breathing and circulation (IVs and direct pressure). Obtaining consent is done by the physician.

A client has hypertension and high risk factors for cardiovascular disease. The client is overwhelmed with the recommended lifestyle changes. What action by the nurse is best? a. Assess the client's support system. b. Assist in finding one change the client can control. c. Determine what stressors the client faces in daily life. d. Inquire about delegating some of the client's obligations.

ANS: B All options are appropriate when assessing stress and responses to stress. However, this client feels overwhelmed by the suggested lifestyle changes. Instead of looking at all the needed changes, the nurse should assist the client in choosing one the client feels optimistic about controlling. Once the client has mastered that change, he or she can move forward with another change. Determining support systems, daily stressors, and delegation opportunities does not directly impact the client's feelings of control.

A client is being discharged home after a large myocardial infarction and subsequent coronary artery bypass grafting surgery. The client's sternal wound has not yet healed. What statement by the client most indicates a higher risk of developing sepsis after discharge? a. "All my friends and neighbors are planning a party for me." b. "I hope I can get my water turned back on when I get home." c. "I am going to have my daughter scoop the cat litter box." d. "My grandkids are so excited to have me coming home!"

ANS: B All these statements indicate a potential for leading to infection once the client gets back home. A large party might include individuals who are themselves ill and contagious. Having litter boxes in the home can expose the client to microbes that can lead to infection. Small children often have upper respiratory infections and poor hand hygiene that spread germs. However, the most worrisome statement is the lack of running water for handwashing and general hygiene and cleaning purposes.

A nurse wants to provide community service that helps meet the goals of Healthy People 2020 (HP2020) related to cardiovascular disease and stroke. What activity would best meet this goal? a. Teach high school students heart-healthy living. b. Participate in blood pressure screenings at the mall. c. Provide pamphlets on heart disease at the grocery store. d. Set up an "Ask the nurse" booth at the pet store.

ANS: B An important goal of HP2020 is to increase the proportion of adults who have had their blood pressure measured within the preceding 2 years and can state whether their blood pressure was normal or high. Participating in blood pressure screening in a public spot will best help meet that goal. The other options are all appropriate but do not specifically help meet a goal.

A nurse is caring for four clients. Which one should the nurse see first? a. Client who needs a beta blocker, and has a blood pressure of 92/58 mm Hg b. Client who had a first dose of captopril (Capoten) and needs to use the bathroom c. Hypertensive client with a blood pressure of 188/92 mm Hg d. Client who needs pain medication prior to a dressing change of a surgical wound

ANS: B Angiotensin-converting enzyme inhibitors such as captopril can cause hypotension, especially after the first dose. The nurse should see this client first to prevent falling if the client decides to get up without assistance. The two blood pressure readings are abnormal but not critical. The nurse should check on the client with higher blood pressure next to assess for problems related to the reading. The nurse can administer the beta blocker as standards state to hold it if the systolic blood pressure is below 90 mm Hg. The client who needs pain medication prior to the dressing change is not a priority over client safety and assisting the other client to the bathroom.

A client had a percutaneous transluminal coronary angioplasty for peripheral arterial disease. What assessment finding by the nurse indicates a priority outcome for this client has been met? a. Pain rated as 2/10 after medication b. Distal pulse on affected extremity 2+/4+ c. Remains on bedrest as directed d. Verbalizes understanding of procedure

ANS: B Assessing circulation distal to the puncture site is a critical nursing action. A pulse of 2+/4+ indicates good perfusion. Pain control, remaining on bedrest as directed after the procedure, and understanding are all important, but do not take priority over perfusion.

A nurse assesses a client who has mitral valve regurgitation. For which cardiac dysrhythmia should the nurse assess? a. Preventricular contractions b. Atrial fibrillation c. Symptomatic bradycardia d. Sinus tachycardia

ANS: B Atrial fibrillation is a clinical manifestation of mitral valve regurgitation and stenosis. Preventricular contractions and bradycardia are not associated with valvular problems. These are usually identified in clients with electrolyte imbalances, myocardial infarction, and sinus node problems. Sinus tachycardia is a manifestation of aortic regurgitation due to a decrease in cardiac output.

A nurse is assessing clients on a medical-surgical unit. Which client should the nurse identify as being at greatest risk for atrial fibrillation? a. A 45-year-old who takes an aspirin daily b. A 50-year-old who is post coronary artery bypass graft surgery c. A 78-year-old who had a carotid endarterectomy d. An 80-year-old with chronic obstructive pulmonary disease

ANS: B Atrial fibrillation occurs commonly in clients with cardiac disease and is a common occurrence after coronary artery bypass graft surgery. The other conditions do not place these clients at higher risk for atrial fibrillation.

A nurse evaluates prescriptions for a client with chronic atrial fibrillation. Which medication should the nurse expect to find on this client's medication administration record to prevent a common complication of this condition? a. Sotalol (Betapace) b. Warfarin (Coumadin) c. Atropine (Sal-Tropine) d. Lidocaine (Xylocaine)

ANS: B Atrial fibrillation puts clients at risk for developing emboli. Clients at risk for emboli are treated with anticoagulants, such as heparin, enoxaparin, or warfarin. Sotalol, atropine, and lidocaine are not appropriate for this complication.

A nurse cares for a client who has a heart rate averaging 56 beats/min with no adverse symptoms. Which activity modification should the nurse suggest to avoid further slowing of the heart rate? a. "Make certain that your bath water is warm." b. "Avoid straining while having a bowel movement." c. "Limit your intake of caffeinated drinks to one a day." d. "Avoid strenuous exercise such as running."

ANS: B Bearing down strenuously during a bowel movement is one type of Valsalva maneuver, which stimulates the vagus nerve and results in slowing of the heart rate. Such a response is not desirable in a person who has bradycardia. The other instructions are not appropriate for this condition.

A nurse teaches a client with diabetes mellitus and a body mass index of 42 who is at high risk for coronary artery disease. Which statement related to nutrition should the nurse include in this client's teaching? a. "The best way to lose weight is a high-protein, low-carbohydrate diet." b. "You should balance weight loss with consuming necessary nutrients." c. "A nutritionist will provide you with information about your new diet." d. "If you exercise more frequently, you won't need to change your diet."

ANS: B Clients at risk for cardiovascular diseases should follow the American Heart Association guidelines to combat obesity and improve cardiac health. The nurse should encourage the client to eat vegetables, fruits, unrefined whole-grain products, and fat-free dairy products while losing weight. High-protein food items are often high in fat and calories. Although the nutritionist can assist with client education, the nurse should include nutrition education and assist the client to make healthy decisions. Exercising and eating nutrient-rich foods are both important components in reducing cardiovascular risk.

A client is receiving an infusion of alteplase (Activase) for an intra-arterial clot. The client begins to mumble and is disoriented. What action by the nurse takes priority? a. Assess the client's neurologic status. b. Notify the Rapid Response Team. c. Prepare to administer vitamin K. d. Turn down the infusion rate.

ANS: B Clients on fibrinolytic therapy are at high risk of bleeding. The sudden onset of neurologic signs may indicate the client is having a hemorrhagic stroke. The nurse does need to complete a thorough neurological examination, but should first call the Rapid Response Team based on the client's manifestations. The nurse notifies the Rapid Response Team first. Vitamin K is not the antidote for this drug. Turning down the infusion rate will not be helpful if the client is still receiving any of the drug.

After teaching a client who is being discharged home after mitral valve replacement surgery, the nurse assesses the client's understanding. Which client statement indicates a need for additional teaching? a. "I'll be able to carry heavy loads after 6 months of rest." b. "I will have my teeth cleaned by my dentist in 2 weeks." c. "I must avoid eating foods high in vitamin K, like spinach." d. "I must use an electric razor instead of a straight razor to shave."

ANS: B Clients who have defective or repaired valves are at high risk for endocarditis. The client who has had valve surgery should avoid dental procedures for 6 months because of the risk for endocarditis. When undergoing a mitral valve replacement surgery, the client needs to be placed on anticoagulant therapy to prevent vegetation forming on the new valve. Clients on anticoagulant therapy should be instructed on bleeding precautions, including using an electric razor. If the client is prescribed warfarin, the client should avoid foods high in vitamin K. Clients recovering from open heart valve replacements should not carry anything heavy for 6 months while the chest incision and muscle heal.

A nurse assesses a client who has a history of heart failure. Which question should the nurse ask to assess the extent of the client's heart failure? a. "Do you have trouble breathing or chest pain?" b. "Are you able to walk upstairs without fatigue?" c. "Do you awake with breathlessness during the night?" d. "Do you have new-onset heaviness in your legs?"

ANS: B Clients with a history of heart failure generally have negative findings, such as shortness of breath. The nurse needs to determine whether the client's activity is the same or worse, or whether the client identifies a decrease in activity level. Trouble breathing, chest pain, breathlessness at night, and peripheral edema are symptoms of heart failure, but do not provide data that can determine the extent of the client's heart failure.

A nurse assesses a client with atrial fibrillation. Which manifestation should alert the nurse to the possibility of a serious complication from this condition? a. Sinus tachycardia b. Speech alterations c. Fatigue d. Dyspnea with activity

ANS: B Clients with atrial fibrillation are at risk for embolic stroke. Evidence of embolic events includes changes in mentation, speech, sensory function, and motor function. Clients with atrial fibrillation often have a rapid ventricular response as a result. Fatigue is a nonspecific complaint. Clients with atrial fibrillation often have dyspnea as a result of the decreased cardiac output caused by the rhythm disturbance.

A nurse assesses a client with mitral valve stenosis. What clinical manifestation should alert the nurse to the possibility that the client's stenosis has progressed? a. Oxygen saturation of 92% b. Dyspnea on exertion c. Muted systolic murmur d. Upper extremity weakness

ANS: B Dyspnea on exertion develops as the mitral valvular orifice narrows and pressure in the lungs increases. The other manifestations do not relate to the progression of mitral valve stenosis.

A home health care nurse is visiting an older client who lives alone after being discharged from the hospital after a coronary artery bypass graft. What finding in the home most causes the nurse to consider additional referrals? a. Dirty carpets in need of vacuuming b. Expired food in the refrigerator c. Old medications in the kitchen d. Several cats present in the home

ANS: B Expired food in the refrigerator demonstrates a safety concern for the client and a possible lack of money to buy food. The nurse can consider a referral to Meals on Wheels or another home-based food program. Dirty carpets may indicate the client has no household help and is waiting for clearance to vacuum. Old medications can be managed by the home health care nurse and the client working collaboratively. Having pets is not a cause for concern.

A nurse assists with the cardioversion of a client experiencing acute atrial fibrillation. Which action should the nurse take prior to the initiation of cardioversion? a. Administer intravenous adenosine. b. Turn off oxygen therapy. c. Ensure a tongue blade is available. d. Position the client on the left side.

ANS: B For safety during cardioversion, the nurse should turn off any oxygen therapy to prevent fire. The other interventions are not appropriate for a cardioversion. The client should be placed in a supine position.

A nurse is in charge of the coronary intensive care unit. Which client should the nurse see first? a. Client on a nitroglycerin infusion at 5 mcg/min, not titrated in the last 4 hours b. Client who is 1 day post coronary artery bypass graft, blood pressure 180/100 mm Hg c. Client who is 1 day post percutaneous coronary intervention, going home this morning d. Client who is 2 days post coronary artery bypass graft, became dizzy this a.m. while walking

ANS: B Hypertension after coronary artery bypass graft surgery can be dangerous because it puts too much pressure on the suture lines and can cause bleeding. The charge nurse should see this client first. The client who became dizzy earlier should be seen next. The client on the nitroglycerin drip is stable. The client going home can wait until the other clients are cared for.

After assessing a client who is receiving an amiodarone intravenous infusion for unstable ventricular tachycardia, the nurse documents the findings and compares these with the previous assessment findings: Vital Signs Nursing Assessment Time: 0800 Temperature: 98° F Heart rate: 68 beats/min Blood pressure: 135/60 mm Hg Respiratory rate: 14 breaths/min Oxygen saturation: 96% Oxygen therapy: 2 L nasal cannula Time: 1000 Temperature: 98.2° F Heart rate: 50 beats/min Blood pressure: 132/57 mm Hg Respiratory rate: 16 breaths/min Oxygen saturation: 95% Oxygen therapy: 2 L nasal cannula Time: 0800 Client alert and oriented. Cardiac rhythm: normal sinus rhythm. Skin: warm, dry, and appropriate for race. Respirations equal and unlabored. Client denies shortness of breath and chest pain. Time: 1000 Client alert and oriented. Cardiac rhythm: sinus bradycardia. Skin: warm, dry, and appropriate for race. Respirations equal and unlabored. Client denies shortness of breath and chest pain. Client voids 420 mL of clear yellow urine. Based on the assessments, which action should the nurse take? a. Stop the infusion and flush the IV. b. Slow the amiodarone infusion rate. c. Administer IV normal saline. d. Ask the client to cough and deep breathe.

ANS: B IV administration of amiodarone may cause bradycardia and atrioventricular (AV) block. The correct action for the nurse to take at this time is to slow the infusion, because the client is asymptomatic and no evidence reveals AV block that might require pacing. Abruptly ceasing the medication could allow fatal dysrhythmias to occur. The administration of IV fluids and encouragement of coughing and deep breathing exercises are not indicated, and will not increase the client's heart rate.

A client has hemodynamic monitoring after a myocardial infarction. What safety precaution does the nurse implement for this client? a. Document pulmonary artery wedge pressure (PAWP) readings and assess their trends. b. Ensure the balloon does not remain wedged. c. Keep the client on strict NPO status. d. Maintain the client in a semi-Fowler's position.

ANS: B If the balloon remains inflated, it can cause pulmonary infarction or rupture. The nurse should ensure the balloon remains deflated between PAWP readings. Documenting PAWP readings and assessing trends is an important nursing action related to hemodynamic monitoring, but is not specifically related to safety. The client does not have to be NPO while undergoing hemodynamic monitoring. Positioning may or may not affect readings.

The nurse is caring for a client with a chest tube after a coronary artery bypass graft. The drainage slows significantly. What action by the nurse is most important? a. Increase the setting on the suction. b. Notify the provider immediately. c. Re-position the chest tube. d. Take the tubing apart to assess for clots.

ANS: B If the drainage in the chest tube decreases significantly and dramatically, the tube may be blocked by a clot. This could lead to cardiac tamponade. The nurse should notify the provider immediately. The nurse should not independently increase the suction, re-position the chest tube, or take the tubing apart.

A nurse cares for a client with an intravenous temporary pacemaker for bradycardia. The nurse observes the presence of a pacing spike but no QRS complex on the client's electrocardiogram. Which action should the nurse take next? a. Administer intravenous diltiazem (Cardizem). b. Assess vital signs and level of consciousness. c. Administer sublingual nitroglycerin. d. Assess capillary refill and temperature.

ANS: B In temporary pacing, the wires are threaded onto the epicardial surface of the heart and exit through the chest wall. The pacemaker spike should be followed immediately by a QRS complex. Pacing spikes seen without subsequent QRS complexes imply loss of capture. If there is no capture, then there is no ventricular depolarization and contraction. The nurse should assess for cardiac output via vital signs and level of consciousness. The other interventions would not determine if the client is tolerating the loss of capture.

A student is caring for a client who suffered massive blood loss after trauma. How does the student correlate the blood loss with the client's mean arterial pressure (MAP)? a. It causes vasoconstriction and increased MAP. b. Lower blood volume lowers MAP. c. There is no direct correlation to MAP. d. It raises cardiac output and MAP.

ANS: B Lower blood volume will decrease MAP. The other answers are not accurate.

A client has been diagnosed with hypertension but does not take the antihypertensive medications because of a lack of symptoms. What response by the nurse is best? a. "Do you have trouble affording your medications?" b. "Most people with hypertension do not have symptoms." c. "You are lucky; most people get severe morning headaches." d. "You need to take your medicine or you will get kidney failure.

ANS: B Most people with hypertension are asymptomatic, although a small percentage do have symptoms such as headache. The nurse should explain this to the client. Asking about paying for medications is not related because the client has already admitted nonadherence. Threatening the client with possible complications will not increase compliance.

A client is 4 hours postoperative after a femoropopliteal bypass. The client reports throbbing leg pain on the affected side, rated as 7/10. What action by the nurse takes priority? a. Administer pain medication as ordered. b. Assess distal pulses and skin color. c. Document the findings in the client's chart. d. Notify the surgeon immediately.

ANS: B Once perfusion has been restored or improved to an extremity, clients can often feel a throbbing pain due to the increased blood flow. However, it is important to differentiate this pain from ischemia. The nurse should assess for other signs of perfusion, such as distal pulses and skin color/temperature. Administering pain medication is done once the nurse determines the client's perfusion status is normal. Documentation needs to be thorough. Notifying the surgeon is not necessary.

A nurse works at a community center for older adults. What self-management measure can the nurse teach the clients to prevent shock? a. Do not get dehydrated in warm weather. b. Drink fluids on a regular schedule. c. Seek attention for any lacerations. d. Take medications as prescribed.

ANS: B Preventing dehydration in older adults is important because the age-related decrease in the thirst mechanism makes them prone to dehydration. Having older adults drink fluids on a regular schedule will help keep them hydrated without the influence of thirst (or lack of thirst). Telling clients not to get dehydrated is important, but not the best answer because it doesn't give them the tools to prevent it from occurring. Older adults should seek attention for lacerations, but this is not as important an issue as staying hydrated. Taking medications as prescribed may or may not be related to hydration.

A nurse assesses a client admitted to the cardiac unit. Which statement by the client alerts the nurse to the possibility of right-sided heart failure? a. "I sleep with four pillows at night." b. "My shoes fit really tight lately." c. "I wake up coughing every night." d. "I have trouble catching my breath."

ANS: B Signs of systemic congestion occur with right-sided heart failure. Fluid is retained, pressure builds in the venous system, and peripheral edema develops. Left-sided heart failure symptoms include respiratory symptoms. Orthopnea, coughing, and difficulty breathing all could be results of left-sided heart failure.

A nurse is caring for a client after surgery. The client's respiratory rate has increased from 12 to 18 breaths/min and the pulse rate increased from 86 to 98 beats/min since they were last assessed 4 hours ago. What action by the nurse is best? a. Ask if the client needs pain medication. b. Assess the client's tissue perfusion further. c. Document the findings in the client's chart. d. Increase the rate of the client's IV infusion.

ANS: B Signs of the earliest stage of shock are subtle and may manifest in slight increases in heart rate, respiratory rate, or blood pressure. Even though these readings are not out of the normal range, the nurse should conduct a thorough assessment of the client, focusing on indicators of perfusion. The client may need pain medication, but this is not the priority at this time. Documentation should be done thoroughly but is not the priority either. The nurse should not increase the rate of the IV infusion without an order.

A nurse cares for a client recovering from prosthetic valve replacement surgery. The client asks, "Why will I need to take anticoagulants for the rest of my life?" How should the nurse respond? a. "The prosthetic valve places you at greater risk for a heart attack." b. "Blood clots form more easily in artificial replacement valves." c. "The vein taken from your leg reduces circulation in the leg." d. "The surgery left a lot of small clots in your heart and lungs."

ANS: B Synthetic valve prostheses and scar tissue provide surfaces on which platelets can aggregate easily and initiate the formation of blood clots. The other responses are inaccurate.

A client has presented to the emergency department with an acute myocardial infarction (MI). What action by the nurse is best to meet The Joint Commission's Core Measures outcomes? a. Obtain an electrocardiogram (ECG) now and in the morning. b. Give the client an aspirin. c. Notify the Rapid Response Team. d. Prepare to administer thrombolytics.

ANS: B The Joint Commission's Core Measures set for acute MI require that aspirin is administered when a client with MI presents to the emergency department or when an MI occurs in the hospital. A rapid ECG is vital, but getting another one in the morning is not part of the Core Measures set. The Rapid Response Team is not needed if an emergency department provider is available. Thrombolytics may or may not be needed.

A nurse cares for a client who has an 80% blockage of the right coronary artery (RCA) and is scheduled for bypass surgery. Which intervention should the nurse be prepared to implement while this client waits for surgery? a. Administration of IV furosemide (Lasix) b. Initiation of an external pacemaker c. Assistance with endotracheal intubation d. Placement of central venous access

ANS: B The RCA supplies the right atrium, the right ventricle, the inferior portion of the left ventricle, and the atrioventricular (AV) node. It also supplies the sinoatrial node in 50% of people. If the client totally occludes the RCA, the AV node would not function and the client would go into heart block, so emergency pacing should be available for the client. Furosemide, intubation, and central venous access will not address the primary complication of RCA occlusion, which is AV node malfunction.

After teaching a client who has an implantable cardioverter-defibrillator (ICD), a nurse assesses the client's understanding. Which statement by the client indicates a correct understanding of the teaching? a. "I should wear a snug-fitting shirt over the ICD." b. "I will avoid sources of strong electromagnetic fields." c. "I should participate in a strenuous exercise program." d. "Now I can discontinue my antidysrhythmic medication."

ANS: B The client being discharged with an ICD is instructed to avoid strong sources of electromagnetic fields. Clients should avoid tight clothing, which could cause irritation over the ICD generator. The client should be encouraged to exercise but should not engage in strenuous activities that cause the heart rate to meet or exceed the ICD cutoff point because the ICD can discharge inappropriately. The client should continue all prescribed medications.

A nurse assesses a client with pericarditis. Which assessment finding should the nurse expect to find? a. Heart rate that speeds up and slows down b. Friction rub at the left lower sternal border c. Presence of a regular gallop rhythm d. Coarse crackles in bilateral lung bases

ANS: B The client with pericarditis may present with a pericardial friction rub at the left lower sternal border. This sound is the result of friction from inflamed pericardial layers when they rub together. The other assessments are not related.

The nurse is caring for a client on the medical-surgical unit who suddenly becomes unresponsive and has no pulse. The cardiac monitor shows the rhythm below: After calling for assistance and a defibrillator, which action should the nurse take next? a. Perform a pericardial thump. b. Initiate cardiopulmonary resuscitation (CPR). c. Start an 18-gauge intravenous line. d. Ask the client's family about code status.

ANS: B The client's rhythm is ventricular fibrillation. This is a lethal rhythm that is best treated with immediate defibrillation. While the nurse is waiting for the defibrillator to arrive, the nurse should start CPR. A pericardial thump is not a treatment for ventricular fibrillation. If the client does not already have an IV, other members of the team can insert one after defibrillation. The client's code status should already be known by the nurse prior to this event.

The nurse is evaluating a 3-day diet history with a client who has an elevated lipid panel. What meal selection indicates the client is managing this condition well with diet? a. A 4-ounce steak, French fries, iceberg lettuce b. Baked chicken breast, broccoli, tomatoes c. Fried catfish, cornbread, peas d. Spaghetti with meat sauce, garlic bread

ANS: B The diet recommended for this client would be low in saturated fats and red meat, high in vegetables and whole grains (fiber), low in salt, and low in trans fat. The best choice is the chicken with broccoli and tomatoes. The French fries have too much fat and the iceberg lettuce has little fiber. The catfish is fried. The spaghetti dinner has too much red meat and no vegetables.

A nurse cares for a client who is prescribed magnetic resonance imaging (MRI) of the heart. The client's health history includes a previous myocardial infarction and pacemaker implantation. Which action should the nurse take? a. Schedule an electrocardiogram just before the MRI. b. Notify the health care provider before scheduling the MRI. c. Call the physician and request a laboratory draw for cardiac enzymes. d. Instruct the client to increase fluid intake the day before the MRI.

ANS: B The magnetic fields of the MRI can deactivate the pacemaker. The nurse should call the health care provider and report that the client has a pacemaker so the provider can order other diagnostic tests. The client does not need an electrocardiogram, cardiac enzymes, or increased fluids.

A nurse is caring for four clients. Which client should the nurse assess first? a. Client with an acute myocardial infarction, pulse 102 beats/min b. Client who is 1 hour post angioplasty, has tongue swelling and anxiety c. Client who is post coronary artery bypass, chest tube drained 100 mL/hr d. Client who is post coronary artery bypass, potassium 4.2 mEq/L

ANS: B The post-angioplasty client with tongue swelling and anxiety is exhibiting manifestations of an allergic reaction that could progress to anaphylaxis. The nurse should assess this client first. The client with a heart rate of 102 beats/min may have increased oxygen demands but is just over the normal limit for heart rate. The two post coronary artery bypass clients are stable.

A nurse auscultated heart tones on an older adult client. Which action should the nurse take based on heart tones heard? (Click the media button to hear the audio clip.) a. Administer a diuretic. b. Document the finding. c. Decrease the IV flow rate. d. Evaluate the client's medications.

ANS: B The sound heard is an atrial gallop S4. An atrial gallop may be heard in older clients because of a stiffened ventricle. The nurse should document the finding, but no other intervention is needed at this time.

A nurse teaches a client recovering from a heart transplant who is prescribed cyclosporine (Sandimmune). Which statement should the nurse include in this client's discharge teaching? a. "Use a soft-bristled toothbrush and avoid flossing." b. "Avoid large crowds and people who are sick." c. "Change positions slowly to avoid hypotension." d. "Check your heart rate before taking the medication."

ANS: B These agents cause immune suppression, leaving the client more vulnerable to infection. The medication does not place the client at risk for bleeding, orthostatic hypotension, or a change in heart rate.

A client is in the hospital after suffering a myocardial infarction and has bathroom privileges. The nurse assists the client to the bathroom and notes the client's O2 saturation to be 95%, pulse 88 beats/min, and respiratory rate 16 breaths/min after returning to bed. What action by the nurse is best? a. Administer oxygen at 2 L/min. b. Allow continued bathroom privileges. c. Obtain a bedside commode. d. Suggest the client use a bedpan.

ANS: B This client's physiologic parameters did not exceed normal during and after activity, so it is safe for the client to continue using the bathroom. There is no indication that the client needs oxygen, a commode, or a bedpan.

A nurse is caring for a client after surgery who is restless and apprehensive. The unlicensed assistive personnel (UAP) reports the vital signs and the nurse sees they are only slightly different from previous readings. What action does the nurse delegate next to the UAP? a. Assess the client for pain or discomfort. b. Measure urine output from the catheter. c. Reposition the client to the unaffected side. d. Stay with the client and reassure him or her.

ANS: B Urine output changes are a sensitive early indicator of shock. The nurse should delegate emptying the urinary catheter and measuring output to the UAP as a baseline for hourly urine output measurements. The UAP cannot assess for pain. Repositioning may or may not be effective for decreasing restlessness, but does not take priority over physical assessments. Reassurance is a therapeutic nursing action, but the nurse needs to do more in this situation.

A client has a deep vein thrombosis (DVT). What comfort measure does the nurse delegate to the unlicensed assistive personnel (UAP)? a. Ambulate the client. b. Apply a warm moist pack. c. Massage the client's leg. d. Provide an ice pack.

ANS: B Warm moist packs will help with the pain of a DVT. Ambulation is not a comfort measure. Massaging the client's legs is contraindicated to prevent complications such as pulmonary embolism. Ice packs are not recommended for DVT.

A nurse obtains the health history of a client who is newly admitted to the medical unit. Which statement by the client should alert the nurse to the presence of edema? a. "I wake up to go to the bathroom at night." b. "My shoes fit tighter by the end of the day." c. "I seem to be feeling more anxious lately." d. "I drink at least eight glasses of water a day."

ANS: B Weight gain can result from fluid accumulation in the interstitial spaces. This is known as edema. The nurse should note whether the client feels that his or her shoes or rings are tight, and should observe, when present, an indentation around the leg where the socks end. The other answers do not describe edema.

What nonpharmacologic comfort measures should the nurse include in the plan of care for a client with severe varicose veins? (Select all that apply.) a. Administering mild analgesics for pain b. Applying elastic compression stockings c. Elevating the legs when sitting or lying d. Reminding the client to do leg exercises e. Teaching the client about surgical options

ANS: B, C, D The three E's of care for varicose veins include elastic compression hose, exercise, and elevation. Mild analgesics are not a nonpharmacologic measure. Teaching about surgical options is not a comfort measure.

A nursing student learns about modifiable risk factors for coronary artery disease. Which factors does this include? (Select all that apply.) a. Age b. Hypertension c. Obesity d. Smoking e. Stress

ANS: B, C, D, E Hypertension, obesity, smoking, and excessive stress are all modifiable risk factors for coronary artery disease. Age is a nonmodifiable risk factor.

A nurse assesses a client who is diagnosed with infective endocarditis. Which assessment findings should the nurse expect? (Select all that apply.) a. Weight gain b. Night sweats c. Cardiac murmur d. Abdominal bloating e. Osler's nodes

ANS: B, C, E Clinical manifestations of infective endocarditis include fever with chills, night sweats, malaise and fatigue, anorexia and weight loss, cardiac murmur, and Osler's nodes on palms of the hands and soles of the feet. Abdominal bloating is a manifestation of heart transplantation rejection.

An emergency room nurse assesses a female client. Which assessment findings should alert the nurse to request a prescription for an electrocardiogram? (Select all that apply.) a. Hypertension b. Fatigue despite adequate rest c. Indigestion d. Abdominal pain e. Shortness of breath

ANS: B, C, E Women may not have chest pain with myocardial infarction, but may feel discomfort or indigestion. They often present with a triad of symptoms—indigestion or feeling of abdominal fullness, feeling of chronic fatigue despite adequate rest, and feeling unable to catch their breath. Frequently, women are not diagnosed and therefore are not treated adequately. Hypertension and abdominal pain are not associated with acute coronary syndrome.

A nurse prepares a client for a pharmacologic stress echocardiogram. Which actions should the nurse take when preparing this client for the procedure? (Select all that apply.) a. Assist the provider to place a central venous access device. b. Prepare for continuous blood pressure and pulse monitoring. c. Administer the client's prescribed beta blocker. d. Give the client nothing by mouth 3 to 6 hours before the procedure. e. Explain to the client that dobutamine will simulate exercise for this examination.

ANS: B, D, E Clients receiving a pharmacologic stress echocardiogram will need peripheral venous access and continuous blood pressure and pulse monitoring. The client must be NPO 3 to 6 hours prior to the procedure. Education about dobutamine, which will be administered during the procedure, should be performed. Beta blockers are often held prior to the procedure.

A nurse assesses a client who is recovering after a coronary catheterization. Which assessment findings in the first few hours after the procedure require immediate action by the nurse? (Select all that apply.) a. Blood pressure of 140/88 mm Hg b. Serum potassium of 2.9 mEq/L c. Warmth and redness at the site d. Expanding groin hematoma e. Rhythm changes on the cardiac monitor

ANS: B, D, E In the first few hours postprocedure, the nurse monitors for complications such as bleeding from the insertion site, hypotension, acute closure of the vessel, dye reaction, hypokalemia, and dysrhythmias. The client's blood pressure is slightly elevated but does not need immediate action. Warmth and redness at the site would indicate an infection, but this would not be present in the first few hours.

A client is 1 day postoperative after a coronary artery bypass graft. What nonpharmacologic comfort measures does the nurse include when caring for this client? (Select all that apply.) a. Administer pain medication before ambulating. b. Assist the client into a position of comfort in bed. c. Encourage high-protein diet selections. d. Provide complementary therapies such as music. e. Remind the client to splint the incision when coughing.

ANS: B, D, E Nonpharmacologic comfort measures can include positioning, complementary therapies, and splinting the chest incision. Medications are not nonpharmacologic. Food choices are not comfort measures.

A client with a known abdominal aortic aneurysm reports dizziness and severe abdominal pain. The nurse assesses the client's blood pressure at 82/40 mm Hg. What actions by the nurse are most important? (Select all that apply.) a. Administer pain medication. b. Assess distal pulses every 10 minutes. c. Have the client sign a surgical consent. d. Notify the Rapid Response Team. e. Take vital signs every 10 minutes.

ANS: B, D, E This client may have a ruptured/rupturing aneurysm. The nurse should notify the Rapid Response team and perform frequent client assessments. Giving pain medication will lower the client's blood pressure even further. The nurse cannot have the client sign a consent until the physician has explained the procedure.

A client is receiving an infusion of tissue plasminogen activator (t-PA). The nurse assesses the client to be disoriented to person, place, and time. What action by the nurse is best? a. Assess the client's pupillary responses. b. Request a neurologic consultation. c. Stop the infusion and call the provider. d. Take and document a full set of vital signs.

ANS: C A change in neurologic status in a client receiving t-PA could indicate intracranial hemorrhage. The nurse should stop the infusion and notify the provider immediately. A full assessment, including pupillary responses and vital signs, occurs next. The nurse may or may not need to call a neurologist.

A telemetry nurse assesses a client with third-degree heart block who has wide QRS complexes and a heart rate of 35 beats/min on the cardiac monitor. Which assessment should the nurse complete next? a. Pulmonary auscultation b. Pulse strength and amplitude c. Level of consciousness d. Mobility and gait stability

ANS: C A heart rate of 40 beats/min or less with widened QRS complexes could have hemodynamic consequences. The client is at risk for inadequate cerebral perfusion. The nurse should assess for level of consciousness, light-headedness, confusion, syncope, and seizure activity. Although the other assessments should be completed, the client's level of consciousness is the priority.

A nurse assesses a client who is recovering after a left-sided cardiac catheterization. Which assessment finding requires immediate intervention? a. Urinary output less than intake b. Bruising at the insertion site c. Slurred speech and confusion d. Discomfort in the left leg

ANS: C A left-sided cardiac catheterization specifically increases the risk for a cerebral vascular accident. A change in neurologic status needs to be acted on immediately. Discomfort and bruising are expected at the site. If intake decreases, a client can become dehydrated because of dye excretion. The second intervention would be to increase the client's fluid status. Neurologic changes would take priority.

The provider requests the nurse start an infusion of an inotropic agent on a client. How does the nurse explain the action of these drugs to the client and spouse? a. "It constricts vessels, improving blood flow." b. "It dilates vessels, which lessens the work of the heart." c. "It increases the force of the heart's contractions." d. "It slows the heart rate down for better filling."

ANS: C A positive inotrope is a medication that increases the strength of the heart's contractions. The other options are not correct.

A client in the cardiac stepdown unit reports severe, crushing chest pain accompanied by nausea and vomiting. What action by the nurse takes priority? a. Administer an aspirin. b. Call for an electrocardiogram (ECG). c. Maintain airway patency. d. Notify the provider.

ANS: C Airway always is the priority. The other actions are important in this situation as well, but the nurse should stay with the client and ensure the airway remains patent (especially if vomiting occurs) while another person calls the provider (or Rapid Response Team) and facilitates getting an ECG done. Aspirin will probably be administered, depending on the provider's prescription and the client's current medications.

After teaching a client who is recovering from a heart transplant to change positions slowly, the client asks, "Why is this important?" How should the nurse respond? a. "Rapid position changes can create shear and friction forces, which can tear out your internal vascular sutures." b. "Your new vascular connections are more sensitive to position changes, leading to increased intravascular pressure and dizziness." c. "Your new heart is not connected to the nervous system and is unable to respond to decreases in blood pressure caused by position changes." d. "While your heart is recovering, blood flow is diverted away from the brain, increasing the risk for stroke when you stand up."

ANS: C Because the new heart is denervated, the baroreceptor and other mechanisms that compensate for blood pressure drops caused by position changes do not function. This allows orthostatic hypotension to persist in the postoperative period. The other options are false statements and do not correctly address the client's question.

A client in shock has been started on dopamine. What assessment finding requires the nurse to communicate with the provider immediately? a. Blood pressure of 98/68 mm Hg b. Pedal pulses 1+/4+ bilaterally c. Report of chest heaviness d. Urine output of 32 mL/hr

ANS: C Chest heaviness or pain indicates myocardial ischemia, a possible adverse effect of dopamine. While taking dopamine, the oxygen requirements of the heart are increased due to increased myocardial workload, and may cause ischemia. Without knowing the client's previous blood pressure or pedal pulses, there is not enough information to determine if these are an improvement or not. A urine output of 32 mL/hr is acceptable.

A nurse cares for a client who is recovering from a myocardial infarction. The client states, "I will need to stop eating so much chili to keep that indigestion pain from returning." How should the nurse respond? a. "Chili is high in fat and calories; it would be a good idea to stop eating it." b. "The provider has prescribed an antacid for you to take every morning." c. "What do you understand about what happened to you?" d. "When did you start experiencing this indigestion?"

ANS: C Clients who experience myocardial infarction often respond with denial, which is a defense mechanism. The nurse should ask the client what he or she thinks happened, or what the illness means to him or her. The other responses do not address the client's misconception about recent pain and the cause of that pain.

A nurse cares for a client with atrial fibrillation who reports fatigue when completing activities of daily living. What interventions should the nurse implement to address this client's concerns? a. Administer oxygen therapy at 2 liters per nasal cannula. b. Provide the client with a sleeping pill to stimulate rest. c. Schedule periods of exercise and rest during the day. d. Ask unlicensed assistive personnel to help bathe the client.

ANS: C Clients who have atrial fibrillation are at risk for decreased cardiac output and fatigue when completing activities of daily living. The nurse should schedule periods of exercise and rest during the day to decrease fatigue. The other interventions will not assist the client with self-care activities.

The nurse asks a client who has experienced ventricular dysrhythmias about substance abuse. The client asks, "Why do you want to know if I use cocaine?" How should the nurse respond? a. "Substance abuse puts clients at risk for many health issues." b. "The hospital requires that I ask you about cocaine use." c. "Clients who use cocaine are at risk for fatal dysrhythmias." d. "We can provide services for cessation of substance abuse."

ANS: C Clients who use cocaine or illicit inhalants are particularly at risk for potentially fatal dysrhythmias. The other responses do not adequately address the client's question.

A nurse assesses a client in an outpatient clinic. Which statement alerts the nurse to the possibility of left-sided heart failure? a. "I have been drinking more water than usual." b. "I am awakened by the need to urinate at night." c. "I must stop halfway up the stairs to catch my breath." d. "I have experienced blurred vision on several occasions."

ANS: C Clients with left-sided heart failure report weakness or fatigue while performing normal activities of daily living, as well as difficulty breathing, or "catching their breath." This occurs as fluid moves into the alveoli. Nocturia is often seen with right-sided heart failure. Thirst and blurred vision are not related to heart failure.

A nurse is assessing an obese client in the clinic for follow-up after an episode of deep vein thrombosis. The client has lost 20 pounds since the last visit. What action by the nurse is best? a. Ask if the weight loss was intended. b. Encourage a high-protein, high-fiber diet. c. Measure for new compression stockings. d. Review a 3-day food recall diary.

ANS: C Compression stockings must fit correctly in order to work. After losing a significant amount of weight, the client should be re-measured and new stockings ordered if needed. The other options are appropriate, but not the most important.

A student nurse asks what "essential hypertension" is. What response by the registered nurse is best? a. "It means it is caused by another disease." b. "It means it is 'essential' that it be treated." c. "It is hypertension with no specific cause." d. "It refers to severe and life-threatening hypertension."

ANS: C Essential hypertension is the most common type of hypertension and has no specific cause such as an underlying disease process. Hypertension that is due to another disease process is called secondary hypertension. A severe, life-threatening form of hypertension is malignant hypertension.

A client has an intra-arterial blood pressure monitoring line. The nurse notes bright red blood on the client's sheets. What action should the nurse perform first? a. Assess the insertion site. b. Change the client's sheets. c. Put on a pair of gloves. d. Assess blood pressure.

ANS: C For the nurse's safety, he or she should put on a pair of gloves to prevent blood exposure. The other actions are appropriate as well, but first the nurse must don a pair of gloves.

A nurse assesses an older adult client who is experiencing a myocardial infarction. Which clinical manifestation should the nurse expect? a. Excruciating pain on inspiration b. Left lateral chest wall pain c. Disorientation and confusion d. Numbness and tingling of the arm

ANS: C In older adults, disorientation or confusion may be the major manifestation of myocardial infarction caused by poor cardiac output. Pain manifestations and numbness and tingling of the arm could also be related to the myocardial infarction. However, the nurse should be more concerned about the new onset of disorientation or confusion caused by decreased perfusion.

A nurse assesses a client 2 hours after a cardiac angiography via the left femoral artery. The nurse notes that the left pedal pulse is weak. Which action should the nurse take? a. Elevate the leg and apply a sandbag to the entrance site. b. Increase the flow rate of intravenous fluids. c. Assess the color and temperature of the left leg. d. Document the finding as "left pedal pulse of +1/4."

ANS: C Loss of a pulse distal to an angiography entry site is serious, indicating a possible arterial obstruction. The pulse may be faint because of edema. The left pulse should be compared with the right, and pulses should be compared with previous assessments, especially before the procedure. Assessing color (pale, cyanosis) and temperature (cool, cold) will identify a decrease in circulation. Once all peripheral and vascular assessment data are acquired, the primary health care provider should be notified. Simply documenting the findings is inappropriate. The leg should be positioned below the level of the heart or dangling to increase blood flow to the distal portion of the leg. Increasing intravenous fluids will not address the client's problem.

A student nurse is caring for a client who will be receiving sodium nitroprusside (Nipride) via IV infusion. What action by the student causes the registered nurse to intervene? a. Assessing the IV site before giving the drug b. Obtaining a programmable ("smart") IV pump c. Removing the IV bag from the brown plastic cover d. Taking and recording a baseline set of vital signs

ANS: C Nitroprusside degrades in the presence of light, so it must be protected by leaving it in the original brown plastic bag when infusing. The other actions are correct, although a "smart" pump is not necessarily required if the facility does not have them available. The drug must be administered via an IV pump, although the programmable pump is preferred for safety.

A nurse assesses an older adult client who has multiple chronic diseases. The client's heart rate is 48 beats/min. Which action should the nurse take first? a. Document the finding in the chart. b. Initiate external pacing. c. Assess the client's medications. d. Administer 1 mg of atropine.

ANS: C Pacemaker cells in the conduction system decrease in number as a person ages, resulting in bradycardia. The nurse should check the medication reconciliation for medications that might cause such a drop in heart rate, then should inform the health care provider. Documentation is important, but it is not the priority action. The heart rate is not low enough for atropine or an external pacemaker to be needed.

A client had an acute myocardial infarction. What assessment finding indicates to the nurse that a significant complication has occurred? a. Blood pressure that is 20 mm Hg below baseline b. Oxygen saturation of 94% on room air c. Poor peripheral pulses and cool skin d. Urine output of 1.2 mL/kg/hr for 4 hours

ANS: C Poor peripheral pulses and cool skin may be signs of impending cardiogenic shock and should be reported immediately. A blood pressure drop of 20 mm Hg is not worrisome. An oxygen saturation of 94% is just slightly below normal. A urine output of 1.2 mL/kg/hr for 4 hours is normal.

A client presents to the emergency department with an acute myocardial infarction (MI) at 1500 (3:00 PM). The facility has 24-hour catheterization laboratory abilities. To meet The Joint Commission's Core Measures set, by what time should the client have a percutaneous coronary intervention performed? a. 1530 (3:30 PM) b. 1600 (4:00 PM) c. 1630 (4:30 PM) d. 1700 (5:00 PM)

ANS: C The Joint Commission's Core Measures set for MI includes percutaneous coronary intervention within 90 minutes of diagnosis of myocardial infarction. Therefore, the client should have a percutaneous coronary intervention performed no later than 1630 (4:30 PM).

A nurse assesses clients on a medical-surgical unit. Which client should the nurse identify as having the greatest risk for cardiovascular disease? a. An 86-year-old man with a history of asthma b. A 32-year-old Asian-American man with colorectal cancer c. A 45-year-old American Indian woman with diabetes mellitus d. A 53-year-old postmenopausal woman who is on hormone therapy

ANS: C The incidence of coronary artery disease and hypertension is higher in American Indians than in whites or Asian Americans. Diabetes mellitus increases the risk for hypertension and coronary artery disease in people of any race or ethnicity. Asthma, colorectal cancer, and hormone therapy do not increase risk for cardiovascular disease.

A nurse prepares a client for coronary artery bypass graft surgery. The client states, "I am afraid I might die." How should the nurse respond? a. "This is a routine surgery and the risk of death is very low." b. "Would you like to speak with a chaplain prior to surgery?" c. "Tell me more about your concerns about the surgery." d. "What support systems do you have to assist you?"

ANS: C The nurse should discuss the client's feelings and concerns related to the surgery. The nurse should not provide false hope or push the client's concerns off on the chaplain. The nurse should address support systems after addressing the client's current issue.

A nurse caring for a client notes the following assessments: white blood cell count 3800/mm3, blood glucose level 198 mg/dL, and temperature 96.2° F (35.6° C). What action by the nurse takes priority? a. Document the findings in the client's chart. b. Give the client warmed blankets for comfort. c. Notify the health care provider immediately. d. Prepare to administer insulin per sliding scale.

ANS: C This client has several indicators of sepsis with systemic inflammatory response. The nurse should notify the health care provider immediately. Documentation needs to be thorough but does not take priority. The client may appreciate warm blankets, but comfort measures do not take priority. The client may or may not need insulin.

A nurse performs an admission assessment on a 75-year-old client with multiple chronic diseases. The client's blood pressure is 135/75 mm Hg and oxygen saturation is 94% on 2 liters per nasal cannula. The nurse assesses the client's rhythm on the cardiac monitor and observes the reading shown below: Which action should the nurse take first? a. Begin external temporary pacing. b. Assess peripheral pulse strength. c. Ask the client what medications he or she takes. d. Administer 1 mg of atropine.

ANS: C This client is stable and therefore does not require any intervention except to determine the cause of the bradycardia. Bradycardia is often caused by medications. Clients who have multiple chronic diseases are often on multiple medications that can interact with each other. The nurse should assess the client's current medications first.

Which statements by the client indicate good understanding of foot care in peripheral vascular disease? (Select all that apply.) a. "A good abrasive pumice stone will keep my feet soft." b. "I'll always wear shoes if I can buy cheap flip-flops." c. "I will keep my feet dry, especially between the toes." d. "Lotion is important to keep my feet smooth and soft." e. "Washing my feet in room-temperature water is best."

ANS: C, D, E Good foot care includes appropriate hygiene and injury prevention. Keeping the feet dry; wearing good, comfortable shoes; using lotion; washing the feet in room-temperature water; and cutting the nails straight across are all important measures. Abrasive material such as pumice stones should not be used. Cheap flip-flops may not fit well and won't offer much protection against injury.

A nursing student is caring for a client with an abdominal aneurysm. What action by the student requires the registered nurse to intervene? a. Assesses the client for back pain b. Auscultates over abdominal bruit c. Measures the abdominal girth d. Palpates the abdomen in four quadrants

ANS: D Abdominal aneurysms should never be palpated as this increases the risk of rupture. The registered nurse should intervene when the student attempts to do this. The other actions are appropriate.

An emergency department nurse triages clients who present with chest discomfort. Which client should the nurse plan to assess first? a. A 42-year-old female who describes her pain as a dull ache with numbness in her fingers b. A 49-year-old male who reports moderate pain that is worse on inspiration c. A 53-year-old female who reports substernal pain that radiates to her abdomen d. A 58-year-old male who describes his pain as intense stabbing that spreads across his chest

ANS: D All clients who have chest pain should be assessed more thoroughly. To determine which client should be seen first, the nurse must understand common differences in pain descriptions. Intense stabbing, vise-like substernal pain that spreads through the client's chest, arms, jaw, back, or neck is indicative of a myocardial infarction. The nurse should plan to see this client first to prevent cardiac cell death. A dull ache with numbness in the fingers is consistent with anxiety. Pain that gets worse with inspiration is usually related to a pleuropulmonary problem. Pain that spreads to the abdomen is often associated with an esophageal-gastric problem, especially when this pain is experienced by a male client. Female clients may experience abdominal discomfort with a myocardial event. Although clients with anxiety, pleuropulmonary, and esophageal-gastric problems should be seen, they are not a higher priority than myocardial infarction.

A nurse assesses a client who is scheduled for a cardiac catheterization. Which assessment should the nurse complete prior to this procedure? a. Client's level of anxiety b. Ability to turn self in bed c. Cardiac rhythm and heart rate d. Allergies to iodine-based agents

ANS: D Before the procedure, the nurse should ascertain whether the client has an allergy to iodine-containing preparations, such as seafood or local anesthetics. The contrast medium used during the procedure is iodine based. This allergy can cause a life-threatening reaction, so it is a high priority. Second, it is important for the nurse to assess anxiety, mobility, and baseline cardiac status.

A nurse assesses a client after administering a prescribed beta blocker. Which assessment should the nurse expect to find? a. Blood pressure increased from 98/42 mm Hg to 132/60 mm Hg b. Respiratory rate decreased from 25 breaths/min to 14 breaths/min c. Oxygen saturation increased from 88% to 96% d. Pulse decreased from 100 beats/min to 80 beats/min

ANS: D Beta blockers block the stimulation of beta1-adrenergic receptors. They block the sympathetic (fight-or-flight) response and decrease the heart rate (HR). The beta blocker will decrease HR and blood pressure, increasing ventricular filling time. It usually does not have effects on beta2-adrenergic receptor sites. Cardiac output will drop because of decreased HR.

An older client with peripheral vascular disease (PVD) is explaining the daily foot care regimen to the family practice clinic nurse. What statement by the client may indicate a barrier to proper foot care? a. "I nearly always wear comfy sweatpants and house shoes." b. "I'm glad I get energy assistance so my house isn't so cold." c. "My daughter makes sure I have plenty of lotion for my feet." d. "My hands shake when I try to do things requiring coordination."

ANS: D Clients with PVD need to pay special attention to their feet. Toenails need to be kept short and cut straight across. The client whose hands shake may cause injury when trimming toenails. The nurse should refer this client to a podiatrist. Comfy sweatpants and house shoes are generally loose and not restrictive, which is important for clients with PVD. Keeping the house at a comfortable temperature makes it less likely the client will use alternative heat sources, such as heating pads, to stay warm. The client should keep the feet moist and soft with lotion.

A nurse teaches a client who has a history of heart failure. Which statement should the nurse include in this client's discharge teaching? a. "Avoid drinking more than 3 quarts of liquids each day." b. "Eat six small meals daily instead of three larger meals." c. "When you feel short of breath, take an additional diuretic." d. "Weigh yourself daily while wearing the same amount of clothing."

ANS: D Clients with heart failure are instructed to weigh themselves daily to detect worsening heart failure early, and thus avoid complications. Other signs of worsening heart failure include increasing dyspnea, exercise intolerance, cold symptoms, and nocturia. Fluid overload increases symptoms of heart failure. The client should be taught to eat a heart-healthy diet, balance intake and output to prevent dehydration and overload, and take medications as prescribed. The most important discharge teaching is daily weights as this provides the best data related to fluid retention.

A nurse teaches a client who is prescribed digoxin (Lanoxin) therapy. Which statement should the nurse include in this client's teaching? a. "Avoid taking aspirin or aspirin-containing products." b. "Increase your intake of foods that are high in potassium." c. "Hold this medication if your pulse rate is below 80 beats/min." d. "Do not take this medication within 1 hour of taking an antacid."

ANS: D Gastrointestinal absorption of digoxin is erratic. Many medications, especially antacids, interfere with its absorption. Clients are taught to hold their digoxin for bradycardia; a heart rate of 80 beats/min is too high for this cutoff. Potassium and aspirin have no impact on digoxin absorption, nor do these statements decrease complications of digoxin therapy.

A nurse assesses a client's electrocardiograph tracing and observes that not all QRS complexes are preceded by a P wave. How should the nurse interpret this observation? a. The client has hyperkalemia causing irregular QRS complexes. b. Ventricular tachycardia is overriding the normal atrial rhythm. c. The client's chest leads are not making sufficient contact with the skin. d. Ventricular and atrial depolarizations are initiated from different sites.

ANS: D Normal rhythm shows one P wave preceding each QRS complex, indicating that all depolarization is initiated at the sinoatrial node. QRS complexes without a P wave indicate a different source of initiation of depolarization. This finding on an electrocardiograph tracing is not an indication of hyperkalemia, ventricular tachycardia, or disconnection of leads.

A client undergoing hemodynamic monitoring after a myocardial infarction has a right atrial pressure of 0.5 mm Hg. What action by the nurse is most appropriate? a. Level the transducer at the phlebostatic axis. b. Lay the client in the supine position. c. Prepare to administer diuretics. d. Prepare to administer a fluid bolus.

ANS: D Normal right atrial pressures are from 1 to 8 mm Hg. Lower pressures usually indicate hypovolemia, so the nurse should prepare to administer a fluid bolus. The transducer should remain leveled at the phlebostatic axis. Positioning may or may not influence readings. Diuretics would be contraindicated.

A nurse is caring for a client with acute pericarditis who reports substernal precordial pain that radiates to the left side of the neck. Which nonpharmacologic comfort measure should the nurse implement? a. Apply an ice pack to the client's chest. b. Provide a neck rub, especially on the left side. c. Allow the client to lie in bed with the lights down. d. Sit the client up with a pillow to lean forward on.

ANS: D Pain from acute pericarditis may worsen when the client lays supine. The nurse should position the client in a comfortable position, which usually is upright and leaning slightly forward. Pain is decreased by using gravity to take pressure off the heart muscle. An ice pack and neck rub will not relieve this pain.

A client is in the preoperative holding area prior to an emergency coronary artery bypass graft (CABG). The client is yelling at family members and tells the doctor to "just get this over with" when asked to sign the consent form. What action by the nurse is best? a. Ask the family members to wait in the waiting area. b. Inform the client that this behavior is unacceptable. c. Stay out of the room to decrease the client's stress levels. d. Tell the client that anxiety is common and that you can help.

ANS: D Preoperative fear and anxiety are common prior to cardiac surgery, especially in emergent situations. The client is exhibiting anxiety, and the nurse should reassure the client that fear is common and offer to help. The other actions will not reduce the client's anxiety.

A nurse assesses a client's electrocardiogram (ECG) and observes the reading shown below: How should the nurse document this client's ECG strip? a. Ventricular tachycardia b. Ventricular fibrillation c. Sinus rhythm with premature atrial contractions (PACs) d. Sinus rhythm with premature ventricular contractions (PVCs)

ANS: D Sinus rhythm with PVCs has an underlying regular sinus rhythm with ventricular depolarization that sometimes precede atrial depolarization. Ventricular tachycardia and ventricular fibrillation rhythms would not have sinus beats present. Premature atrial contractions are atrial contractions initiated from another region of the atria before the sinus node initiates atrial depolarization.

A nurse cares for a client who has advanced cardiac disease and states, "I am having trouble sleeping at night." How should the nurse respond? a. "I will consult the provider to prescribe a sleep study to determine the problem." b. "You become hypoxic while sleeping; oxygen therapy via nasal cannula will help." c. "A continuous positive airway pressure, or CPAP, breathing mask will help you breathe at night." d. "Use pillows to elevate your head and chest while you are sleeping."

ANS: D The client is experiencing orthopnea (shortness of breath while lying flat). The nurse should teach the client to elevate the head and chest with pillows or sleep in a recliner. A sleep study is not necessary to diagnose this client. Oxygen and CPAP will not help a client with orthopnea.

A nurse cares for a client with end-stage heart failure who is awaiting a transplant. The client appears depressed and states, "I know a transplant is my last chance, but I don't want to become a vegetable." How should the nurse respond? a. "Would you like to speak with a priest or chaplain?" b. "I will arrange for a psychiatrist to speak with you." c. "Do you want to come off the transplant list?" d. "Would you like information about advance directives?"

ANS: D The client is verbalizing a real concern or fear about negative outcomes of the surgery. This anxiety itself can have a negative effect on the outcome of the surgery because of sympathetic stimulation. The best action is to allow the client to verbalize the concern and work toward a positive outcome without making the client feel as though he or she is crazy. The client needs to feel that he or she has some control over the future. The nurse personally provides care to address the client's concerns instead of pushing the client's issues off on a chaplain or psychiatrist. The nurse should not jump to conclusions and suggest taking the client off the transplant list, which is the best treatment option.

A nurse is assessing a client who had a myocardial infarction. Upon auscultating heart sounds, the nurse hears the following sound. What action by the nurse is most appropriate? (Click the media button to hear the audio clip.) a. Assess for further chest pain. b. Call the Rapid Response Team. c. Have the client sit upright. d. Listen to the client's lung sounds.

ANS: D The sound the nurse hears is an S3 heart sound, an abnormal sound that may indicate heart failure. The nurse should next assess the client's lung sounds. Assessing for chest pain is not directly related. There is no indication that the Rapid Response Team is needed. Having the client sit up will not change the heart sound.

A student nurse is assessing the peripheral vascular system of an older adult. What action by the student would cause the faculty member to intervene? a. Assessing blood pressure in both upper extremities b. Auscultating the carotid arteries for any bruits c. Classifying capillary refill of 4 seconds as normal d. Palpating both carotid arteries at the same time

ANS: D The student should not compress both carotid arteries at the same time to avoid brain ischemia. Blood pressure should be taken and compared in both arms. Prolonged capillary refill is considered to be greater than 5 seconds in an older adult, so classifying refill of 4 seconds as normal would not require intervention. Bruits should be auscultated.

A nurse assesses a client after administering isosorbide mononitrate (Imdur). The client reports a headache. Which action should the nurse take? a. Initiate oxygen therapy. b. Hold the next dose of Imdur. c. Instruct the client to drink water. d. Administer PRN acetaminophen.

ANS: D The vasodilating effects of isosorbide mononitrate frequently cause clients to have headaches during the initial period of therapy. Clients should be told about this side effect and encouraged to take the medication with food. Some clients obtain relief with mild analgesics, such as acetaminophen. The client's headache is not related to hypoxia or dehydration; therefore, these interventions would not help. The client needs to take the medication as prescribed to prevent angina; the medication should not be held.

A nurse prepares to defibrillate a client who is in ventricular fibrillation. Which priority intervention should the nurse perform prior to defibrillating this client? a. Make sure the defibrillator is set to the synchronous mode. b. Administer 1 mg of intravenous epinephrine. c. Test the equipment by delivering a smaller shock at 100 joules. d. Ensure that everyone is clear of contact with the client and the bed.

ANS: D To avoid injury, the rescuer commands that all personnel clear contact with the client or the bed and ensures their compliance before delivery of the shock. A precordial thump can be delivered when no defibrillator is available. Defibrillation is done in asynchronous mode. Equipment should not be tested before a client is defibrillated because this is an emergency procedure; equipment should be checked on a routine basis. Epinephrine should be administered after defibrillation.

The nurse is preparing to change a client's sternal dressing. What action by the nurse is most important? a. Assess vital signs. b. Don a mask and gown. c. Gather needed supplies. d. Perform hand hygiene.

ANS: D To prevent a sternal wound infection, the nurse washes hands or performs hand hygiene as a priority. Vital signs do not necessarily need to be assessed beforehand. A mask and gown are not needed. The nurse should gather needed supplies, but this is not the priority.

The nurse is reviewing the lipid panel of a male client who has atherosclerosis. Which finding is most concerning? a. Cholesterol: 126 mg/dL b. High-density lipoprotein cholesterol (HDL-C): 48 mg/dL c. Low-density lipoprotein cholesterol (LDL-C): 122 mg/dL d. Triglycerides: 198 mg/dL

ANS: D Triglycerides in men should be below 160 mg/dL. The other values are appropriate for adult males.

A nurse is preparing a client for a femoropopliteal bypass operation. What actions does the nurse delegate to the unlicensed assistive personnel (UAP)? (Select all that apply.) a. Administering preoperative medication b. Ensuring the consent is signed c. Marking pulses with a pen d. Raising the siderails on the bed e. Recording baseline vital signs

ANS: D, E The UAP can raise the siderails of the bed for client safety and take and record the vital signs. Administering medications, ensuring a consent is on the chart, and marking the pulses for later comparison should be done by the registered nurse. This is also often done by the postanesthesia care nurse and is part of the hand-off report.

The clinic nurse is caring for a 57-year-old client who reports experiencing leg pain whenever she walks several blocks. The patient has type 1 diabetes and has smoked a pack of cigarettes every day for the past 40 years. The physician diagnoses intermittent claudication. The nurse should provide what instruction about long-term care to the client? A) "Be sure to practice meticulous foot care." B) "Consider cutting down on your smoking." C) "Reduce your activity level to accommodate your limitations." D) "Try to make sure you eat enough protein."

Ans: "Be sure to practice meticulous foot care." Feedback: The patient with peripheral vascular disease or diabetes should receive education or reinforcement about skin and foot care. Intermittent claudication and other chronic peripheral vascular diseases reduce oxygenation to the feet, making them susceptible to injury and poor healing; therefore, meticulous foot care is essential. The patient should stop smoking—not just cut down—because nicotine is a vasoconstrictor. Daily walking benefits the patient with intermittent claudication. Increased protein intake will not alleviate the patient's symptoms.

New nurses on the telemetry unit have been paired with preceptors. One new nurse asks her preceptor to explain depolarization. What would be the best answer by the preceptor? A) "Depolarization is the mechanical contraction of the heart muscles." B) "Depolarization is the electrical stimulation of the heart muscles." C) "Depolarization is the electrical relaxation of the heart muscles." D) "Depolarization is the mechanical relaxation of the heart muscles."

Ans: "Depolarization is the electrical stimulation of the heart muscles." Feedback: The electrical stimulation of the heart is called depolarization, and the mechanical contraction is called systole. Electrical relaxation is called repolarization, and mechanical relaxation is called diastole.

A patient has been diagnosed with a valvular disorder. The patient tells the nurse that he has read about numerous treatment options, including valvuloplasty. What should the nurse teach the patient about valvuloplasty? A) "For some patients, valvuloplasty can be done in a cardiac catheterization laboratory." B) "Valvuloplasty is a dangerous procedure, but it has excellent potential if it goes well." C) "Valvuloplasty is open heart surgery, but this is very safe these days and normally requires only an overnight hospital stay." D) "It's prudent to get a second opinion before deciding to have valvuloplasty."

Ans: "For some patients, valvuloplasty can be done in a cardiac catheterization laboratory." Feedback: Some valvuloplasty procedures do not require general anesthesia or cardiopulmonary bypass and can be performed in a cardiac catheterization laboratory or hybrid room. Open heart surgery is not required and the procedure does not carry exceptional risks that would designate it as being dangerous. Normally there is no need for the nurse to advocate for a second opinion.

In preparation for cardiac surgery, a patient was taught about measures to prevent venous thromboembolism. What statement indicates that the patient clearly understood this education? A) "I'll try to stay in bed for the first few days to allow myself to heal." B) "I'll make sure that I don't cross my legs when I'm resting in bed." C) "I'll keep pillows under my knees to help my blood circulate better." D) "I'll put on those compression stockings if I get pain in my calves."

Ans: "I'll make sure that I don't cross my legs when I'm resting in bed." Feedback: To prevent venous thromboembolism, patients should avoid crossing the legs. Activity is generally begun as soon as possible and pillows should not be placed under the popliteal space. Compression stockings are often used to prevent venous thromboembolism, but they would not be applied when symptoms emerge.

A patient is a candidate for percutaneous balloon valvuloplasty, but is concerned about how this procedure will affect her busy work schedule. What guidance should the nurse provide to the patient? A) "Patients generally stay in the hospital for 6 to 8 days." B) "Patients are kept in the hospital until they are independent with all aspects of their care." C) "Patients need to stay in the hospital until they regain normal heart function for their age." D) "Patients usually remain at the hospital for 24 to 48 hours."

Ans: "Patients usually remain at the hospital for 24 to 48 hours." Feedback: After undergoing percutaneous balloon valvuloplasty, the patient usually remains in the hospital for 24 to 48 hours. Prediagnosis levels of heart function are not always attainable and the patient does not need to be wholly independent prior to discharge.

During a CPR class, a participant asks about the difference between cardioversion and defibrillation. What would be the instructor's best response? A) "Cardioversion is done on a beating heart; defibrillation is not." B) "The difference is the timing of the delivery of the electric current." C) "Defibrillation is synchronized with the electrical activity of the heart, but cardioversion is not." D) "Cardioversion is always attempted before defibrillation because it has fewer risks."

Ans: "The difference is the timing of the delivery of the electric current." Feedback: One major difference between cardioversion and defibrillation is the timing of the delivery of electrical current. In cardioversion, the delivery of the electrical current is synchronized with the patient's electrical events; in defibrillation, the delivery of the current is immediate and unsynchronized. Both can be done on beating heart (i.e., in a dysrhythmia). Cardioversion is not necessarily attempted first.

The nurse is providing care to a patient who has just undergone an electrophysiologic (EP) study. The patient states that she is nervous about "things going wrong" during the procedure. What is the nurse's best response? A) "This is basically a risk-free procedure." B) "Thousands of patients undergo EP every year." C) "Remember that this is a step that will bring you closer to enjoying good health." D) "The whole team will be monitoring you very closely for the entire procedure."

Ans: "The whole team will be monitoring you very closely for the entire procedure." Feedback: Patients who are to undergo an EP study may be anxious about the procedure and its outcome. A detailed discussion involving the patient, the family, and the electrophysiologist usually occurs to ensure that the patient can give informed consent and to reduce the patient's anxiety about the procedure. It is inaccurate to state that EP is "risk-free" and stating that it is common does not necessarily relieve the patient's anxiety. Characterizing EP as a step toward good health does not directly address the patient's anxiety.

A patient who is a candidate for an implantable cardioverter defibrillator (ICD) asks the nurse about the purpose of this device. What would be the nurse's best response? A) "To detect and treat dysrhythmias such as ventricular fibrillation and ventricular tachycardia" B) "To detect and treat bradycardia, which is an excessively slow heart rate" C) "To detect and treat atrial fibrillation, in which your heart beats too quickly and inefficiently" D) "To shock your heart if you have a heart attack at home"

Ans: "To detect and treat dysrhythmias such as ventricular fibrillation and ventricular tachycardia" Feedback: The ICD is a device that detects and terminates life-threatening episodes of ventricular tachycardia and ventricular fibrillation. It does not treat atrial fibrillation, MI, or bradycardia.

A nurse in the rehabilitation unit is caring for an older adult patient who is in cardiac rehabilitation following an MI. The nurse's plan of care calls for the patient to walk for 10 minutes 3 times a day. The patient questions the relationship between walking and heart function. How should the nurse best reply? A) "The arteries in your legs constrict when you walk and allow the blood to move faster and with more pressure on the tissue." B) Walking increases your heart rate and blood pressure. Therefore your heart is under less stress." C) "Walking helps your heart adjust to your new arteries and helps build your self-esteem." D) "When you walk, the muscles in your legs contract and pump the blood in your veins back toward your heart, which allows more blood to return to your heart."

Ans: "When you walk, the muscles in your legs contract and pump the blood in your veins back toward your heart, which allows more blood to return to your heart." Feedback: Veins, unlike arteries, are equipped with valves that allow blood to move against the force of gravity. The legs have one-way bicuspid valves that prevent blood from seeping backward as it moves forward by the muscles in our legs pressing on the veins as we walk and increasing venous return. Leg arteries do constrict when walking, which allows the blood to move faster and with more pressure on the tissue, but the greater concern is increasing the flow of venous blood to the heart. Walking increases, not decreases, the heart' pumping ability, which increases heart rate and blood pressure and the hearts ability to manage stress. Walking does help the heart adjust to new arteries and may enhance self-esteem, but the patient had an MI—there are no "new arteries."

A patient is undergoing diagnostic testing for mitral stenosis. What statement by the patient during the nurse's interview is most suggestive of this valvular disorder? A) "I get chest pain from time to time, but it usually resolves when I rest." B) "Sometimes when I'm resting, I can feel my heart skip a beat." C) "Whenever I do any form of exercise I get terribly short of breath." D) "My feet and ankles have gotten terribly puffy the last few weeks."

Ans: "Whenever I do any form of exercise I get terribly short of breath." Feedback: The first symptom of mitral stenosis is often breathing difficulty (dyspnea) on exertion as a result of pulmonary venous hypertension. Patients with mitral stenosis are likely to show progressive fatigue as a result of low cardiac output. Palpitations occur in some patients, but dyspnea is a characteristic early symptom. Peripheral edema and chest pain are atypical.

The nurse is calculating a cardiac patient's pulse pressure. If the patient's blood pressure is 122/76 mm Hg, what is the patient's pulse pressure? A) 46 mm Hg B) 99 mm Hg C) 198 mm Hg D) 76 mm Hg

Ans: 46 mm Hg Feedback: Pulse pressure is the difference between the systolic and diastolic pressure. In this case, this value is 46 mm Hg.

The patient has a homocysteine level ordered. What aspects of this test should inform the nurse's care? Select all that apply. A) A 12-hour fast is necessary before drawing the blood sample. B) Recent inactivity can depress homocysteine levels. C) Genetic factors can elevate homocysteine levels. D) A diet low in folic acid elevates homocysteine levels. E) An ECG should be performed immediately before drawing a sample.

Ans: A 12-hour fast is necessary before drawing the blood sample, Genetic factors can elevate homocysteine levels, A diet low in folic acid elevates homocysteine levels Feedback: Genetic factors and a diet low in folic acid, vitamin B6, and vitamin B12 are associated with elevated homocysteine levels. A 12-hour fast is necessary before drawing a blood sample for an accurate serum measurement. An ECG is unnecessary and recent inactivity does not influence the results of the test.

While auscultating a patient's heart sounds, the nurse hears an extra heart sound immediately after the second heart sound (S2). An audible S3 would be considered an expected finding in what patient? A) An older adult B) A 20-year-old patient C) A patient who has undergone valve replacement D) A patient who takes a beta-adrenergic blocker

Ans: A 20-year-old patient Feedback: S3 represents a normal finding in children and adults up to 35 or 40 years of age. In these cases, it is called a physiologic S3. It is an abnormal finding in a patient with an artificial valve, an older adult, or a patient who takes a beta blocker.

The nurse is caring for a patient who has a history of heart disease. What factor should the nurse identify as possibly contributing to a decrease in cardiac output? A) A change in position from standing to sitting B) A heart rate of 54 bpm C) A pulse oximetry reading of 94% D) An increase in preload related to ambulation

Ans: A heart rate of 54 bpm Feedback: Cardiac output is computed by multiplying the stroke volume by the heart rate. Cardiac output can be affected by changes in either stroke volume or heart rate, such as a rate of 54 bpm. An increase in preload will lead to an increase in stroke volume. A pulse oximetry reading of 94% does not indicate hypoxemia, as hypoxia can decrease contractility. Transitioning from standing to sitting would more likely increase rather than decrease cardiac output.

An older adult patient has been treated for a venous ulcer and a plan is in place to prevent the occurrence of future ulcers. What should the nurse include in this plan? A) Use of supplementary oxygen to aid tissue oxygenation B) Daily use of normal saline compresses on the lower limbs C) Daily administration of prophylactic antibiotics D) A high-protein diet that is rich in vitamins

Ans: A high-protein diet that is rich in vitamins Feedback: A diet that is high in protein, vitamins C and A, iron, and zinc is encouraged to promote healing and prevent future ulcers. Prophylactic antibiotics and saline compresses are not used to prevent ulcers. Oxygen supplementation does not prevent ulcer formation.

A medical nurse has admitted four patients over the course of a 12-hour shift. For which patient would assessment of ankle-brachial index (ABI) be most clearly warranted? A) A patient who has peripheral edema secondary to chronic heart failure B) An older adult patient who has a diagnosis of unstable angina C) A patient with poorly controlled type 1 diabetes who is a smoker D) A patient who has community-acquired pneumonia and a history of COPD

Ans: A patient with poorly controlled type 1 diabetes who is a smoker Feedback: Nurses should perform a baseline ABI on any patient with decreased pulses or any patient 50 years of age or older with a history of diabetes or smoking. The other answers do not apply.

A nurse is working with a patient who has been scheduled for a percutaneous coronary intervention (PCI) later in the week. What anticipatory guidance should the nurse provide to the patient? A) He will remain on bed rest for 48 to 72 hours after the procedure. B) He will be given vitamin K infusions to prevent bleeding following PCI. C) A sheath will be placed over the insertion site after the procedure is finished. D) The procedure will likely be repeated in 6 to 8 weeks to ensure success.

Ans: A sheath will be placed over the insertion site after the procedure is finished. Feedback: A sheath is placed over the PCI access site and kept in place until adequate coagulation is achieved. Patients resume activity a few hours after PCI and repeated treatments may or may not be necessary. Anticoagulants, not vitamin K, are administered during PCI.

When hemodynamic monitoring is ordered for a patient, a catheter is inserted into the appropriate blood vessel or heart chamber. When assessing a patient who has such a device in place, the nurse should check which of the following components? Select all that apply. A) A transducer B) A flush system C) A leveler D) A pressure bag E) An oscillator

Ans: A transducer, A flush system, A pressure bag Feedback: To perform hemodynamic monitoring, a CVP, pulmonary artery, or arterial catheter is introduced into the appropriate blood vessel or heart chamber. It is connected to a pressure monitoring system that has several components. Included among these are a transducer, a flush system, and a pressure bag. A pressure monitoring system does not have a leveler or an oscillator.

The nurse providing care for a patient post PTCA knows to monitor the patient closely. For what complications should the nurse monitor the patient? Select all that apply. A) Abrupt closure of the coronary artery B) Venous insufficiency C) Bleeding at the insertion site D) Retroperitoneal bleeding E) Arterial occlusion

Ans: Abrupt closure of the coronary artery, Bleeding at the insertion site, Retroperitoneal bleeding, Arterial occlusion Feedback: Complications after the procedure may include abrupt closure of the coronary artery and vascular complications, such as bleeding at the insertion site, retroperitoneal bleeding, hematoma, and arterial occlusion, as well as acute renal failure. Venous insufficiency is not a postprocedure complication of a PTCA.

The nurse is caring for a 72-year-old patient who is in cardiac rehabilitation following heart surgery. The patient has been walking on a regular basis for about a week and walks for 15 minutes 3 times a day. The patient states that he is having a cramp-like pain in the legs every time he walks and that the pain gets "better when I rest." The patient's care plan should address what problem? A) Decreased mobility related to VTE B) Acute pain related to intermittent claudication C) Decreased mobility related to venous insufficiency D) Acute pain related to vasculitis

Ans: Acute pain related to intermittent claudication Feedback: Intermittent claudication presents as a muscular, cramp-type pain in the extremities consistently reproduced with the same degree of exercise or activity and relieved by rest. Patients with peripheral arterial insufficiency often complain of intermittent claudication due to a lack of oxygen to muscle tissue. Venous insufficiency presents as a disorder of venous blood reflux and does not present with cramp-type pain with exercise. Vasculitis is an inflammation of the blood vessels and presents with weakness, fever, and fatigue, but does not present with cramp-type pain with exercise. The pain associated with VTE does not have this clinical presentation.

A patient with pericarditis has just been admitted to the CCU. The nurse planning the patient's care should prioritize what nursing diagnosis? A) Anxiety related to pericarditis B) Acute pain related to pericarditis C) Ineffective tissue perfusion related to pericarditis D) Ineffective breathing pattern related to pericarditis

Ans: Acute pain related to pericarditis Feedback: The most characteristic symptom of pericarditis is chest pain, although pain also may be located beneath the clavicle, in the neck, or in the left trapezius (scapula) region. The pain or discomfort usually remains fairly constant, but it may worsen with deep inspiration and when lying down or turning. Anxiety is highly plausible and should be addressed, but chest pain is a nearly certain accompaniment to the disease. Breathing and tissue perfusion are likely to be at risk, but pain is certain, especially in the early stages of treatment.

A patient is brought to the ED and determined to be experiencing symptomatic sinus bradycardia. The nurse caring for this patient is aware the medication of choice for treatment of this dysrhythmia is the administration of atropine. What guidelines will the nurse follow when administering atropine? A) Administer atropine 0.5 mg as an IV bolus every 3 to 5 minutes to a maximum of 3.0 mg. B) Administer atropine as a continuous infusion until symptoms resolve. C) Administer atropine as a continuous infusion to a maximum of 30 mg in 24 hours. D) Administer atropine 1.0 mg sublingually.

Ans: Administer atropine 0.5 mg as an IV bolus every 3 to 5 minutes to a maximum of 3.0 mg. Feedback: Atropine 0.5 mg given rapidly as an intravenous (IV) bolus every 3 to 5 minutes to a maximum total dose of 3.0 mg is the medication of choice in treating symptomatic sinus bradycardia. By this guideline, the other listed options are inappropriate.

The nurse is creating a plan of care for a patient with acute coronary syndrome. What nursing action should be included in the patient's care plan? A) Facilitate daily arterial blood gas (ABG) sampling. B) Administer supplementary oxygen, as needed. C) Have patient maintain supine positioning when in bed. D) Perform chest physiotherapy, as indicated.

Ans: Administer supplementary oxygen, as needed. Feedback: Oxygen should be administered along with medication therapy to assist with symptom relief. Administration of oxygen raises the circulating level of oxygen to reduce pain associated with low levels of myocardial oxygen. Physical rest in bed with the head of the bed elevated or in a supportive chair helps decrease chest discomfort and dyspnea. ABGs are diagnostic, not therapeutic, and they are rarely needed on a daily basis. Chest physiotherapy is not used in the treatment of ACS.

A lipid profile has been ordered for a patient who has been experiencing cardiac symptoms. When should a lipid profile be drawn in order to maximize the accuracy of results? A) As close to the end of the day as possible B) After a meal high in fat C) After a 12-hour fast D) Thirty minutes after a normal meal

Ans: After a 12-hour fast Feedback: Although cholesterol levels remain relatively constant over 24 hours, the blood specimen for the lipid profile should be obtained after a 12-hour fast.

A patient with a history rheumatic heart disease knows that she is at risk for bacterial endocarditis when undergoing invasive procedures. Prior to a scheduled cystoscopy, the nurse should ensure that the patient knows the importance of taking which of the following drugs? A) Enoxaparin (Lovenox) B) Metoprolol (Lopressor) C) Azathioprine (Imuran) D) Amoxicillin (Amoxil)

Ans: Amoxicillin (Amoxil) Feedback: Although rare, bacterial endocarditis may be life-threatening. A key strategy is primary prevention in high-risk patients (i.e., those with rheumatic heart disease, mitral valve prolapse, or prosthetic heart valves). Antibiotic prophylaxis is recommended for high-risk patients immediately before and sometimes after certain procedures. Amoxicillin is the drug of choice. None of the other listed drugs is an antibiotic.

A nurse on a medical unit is caring for a patient who has been diagnosed with lymphangitis. When reviewing this patient's medication administration record, the nurse should anticipate which of the following? A) Coumadin (warfarin) B) Lasix (furosemide) C) An antibiotic D) An antiplatelet aggregator

Ans: An antibiotic Feedback: Lymphangitis is an acute inflammation of the lymphatic channels caused by an infectious process. Antibiotics are always a component of treatment. Diuretics are of nominal use. Anticoagulants and antiplatelet aggregators are not indicated in this form of infection.

The nurse is doing discharge teaching with a patient who has coronary artery disease. The patient asks why he has to take an aspirin every day if he doesn't have any pain. What would be the nurse's best response? A) "Taking an aspirin every day is an easy way to help restore the normal function of your heart." B) "An aspirin a day can help prevent some of the blockages that can cause chest pain or heart attacks." C) "Taking an aspirin every day is a simple way to make your blood penetrate your heart more freely." D) "An aspirin a day eventually helps your blood carry more oxygen that it would otherwise."

Ans: An aspirin a day can help prevent some of the blockages that can cause chest pain or heart attacks. Feedback: An aspirin a day is a common nonprescription medication that improves outcomes in patients with CAD due to its antiplatelet action. It does not affect oxygen carrying capacity or perfusion. Aspirin does not restore cardiac function.

The triage nurse in the ED is assessing a patient who has presented with complaint of pain and swelling in her right lower leg. The patient's pain became much worse last night and appeared along with fever, chills, and sweating. The patient states, "I hit my leg on the car door 4 or 5 days ago and it has been sore ever since." The patient has a history of chronic venous insufficiency. What intervention should the nurse anticipate for this patient? A) Platelet transfusion to treat thrombocytopenia B) Warfarin to treat arterial insufficiency C) Antibiotics to treat cellulitis D) Heparin IV to treat VTE

Ans: Antibiotics to treat cellulitis Feedback: Cellulitis is the most common infectious cause of limb swelling. The signs and symptoms include acute onset of swelling, localized redness, and pain; it is frequently associated with systemic signs of fever, chills, and sweating. The patient may be able to identify a trauma that accounts for the source of infection. Thrombocytopenia is a loss or decrease in platelets and increases a patient's risk of bleeding; this problem would not cause these symptoms. Arterial insufficiency would present with ongoing pain related to activity. This patient does not have signs and symptoms of VTE.

You are writing a care plan for a patient who has been diagnosed with angina pectoris. The patient describes herself as being "distressed" and "shocked" by her new diagnosis. What nursing diagnosis is most clearly suggested by the woman's statement? A) Spiritual distress related to change in health status B) Acute confusion related to prognosis for recovery C) Anxiety related to cardiac symptoms D) Deficient knowledge related to treatment of angina pectoris

Ans: Anxiety related to cardiac symptoms Feedback: Although further assessment is warranted, it is not unlikely that the patient is experiencing anxiety. In patients with CAD, this often relates to the threat of sudden death. There is no evidence of confusion (i.e., delirium or dementia) and there may or may not be a spiritual element to her concerns. Similarly, it is not clear that a lack of knowledge or information is the root of her anxiety.

Most individuals who have mitral valve prolapse never have any symptoms, although this is not the case for every patient. What symptoms might a patient have with mitral valve prolapse? Select all that apply. A) Anxiety B) Fatigue C) Shoulder pain D) Tachypnea E) Palpitations

Ans: Anxiety, Fatigue, Palpitations Feedback: Most people who have mitral valve prolapse never have symptoms. A few have symptoms of fatigue, shortness of breath, lightheadedness, dizziness, syncope, palpitations, chest pain, and anxiety. Hyperpnea and shoulder pain are not characteristic symptoms of mitral valve prolapse.

A 79-year-old man is admitted to the medical unit with digital gangrene. The man states that his problems first began when he stubbed his toe going to the bathroom in the dark. In addition to this trauma, the nurse should suspect that the patient has a history of what health problem? A) Raynaud's phenomenon B) CAD C) Arterial insufficiency D) Varicose veins

Ans: Arterial insufficiency Feedback: Arterial insufficiency may result in gangrene of the toe (digital gangrene), which usually is caused by trauma. The toe is stubbed and then turns black. Raynaud's, CAD and varicose veins are not the usual causes of digital gangrene in the elderly.

The nurse working on a cardiac care unit is caring for a patient whose stroke volume has increased. The nurse is aware that afterload influences a patient's stroke volume. The nurse recognizes that afterload is increased when there is what? A) Arterial vasoconstriction B) Venous vasoconstriction C) Arterial vasodilation D) Venous vasodilation

Ans: Arterial vasoconstriction Feedback: Arterial vasoconstriction increases the systemic vascular resistance, which increases the afterload. Venous vasoconstriction decreases preload thereby decreasing stroke volume. Venous vasodilation increases preload.

A patient who has undergone a femoral to popliteal bypass graft surgery returns to the surgical unit. Which assessments should the nurse perform during the first postoperative day? A) Assess pulse of affected extremity every 15 minutes at first. B) Palpate the affected leg for pain during every assessment. C) Assess the patient for signs and symptoms of compartment syndrome every 2 hours. D) Perform Doppler evaluation once daily.

Ans: Assess pulse of affected extremity every 15 minutes at first. Feedback: The primary objective in the postoperative period is to maintain adequate circulation through the arterial repair. Pulses, Doppler assessment, color and temperature, capillary refill, and sensory and motor function of the affected extremity are checked and compared with those of the other extremity; these values are recorded initially every 15 minutes and then at progressively longer intervals if the patient's status remains stable. Doppler evaluations should be performed every 2 hours. Pain is regularly assessed, but palpation is not the preferred method of performing this assessment. Compartment syndrome results from the placement of a cast, not from vascular surgery.

The nurse is caring for a patient who has central venous pressure (CVP) monitoring in place. The nurse's most recent assessment reveals that CVP is 7 mm Hg. What is the nurse's most appropriate action? A) Arrange for continuous cardiac monitoring and reposition the patient. B) Remove the CVP catheter and apply an occlusive dressing. C) Assess the patient for fluid overload and inform the physician. D) Raise the head of the patient's bed and have the patient perform deep breathing exercise, if possible.

Ans: Assess the patient for fluid overload and inform the physician. Feedback: The normal CVP is 2 to 6 mm Hg. Many problems can cause an elevated CVP, but the most common is due to hypervolemia. Assessing the patient and informing the physician are the most prudent actions. Repositioning the patient is ineffective and removing the device is inappropriate.

During a patient's care conference, the team is discussing whether the patient is a candidate for cardiac conduction surgery. What would be the most important criterion for a patient to have this surgery? A) Angina pectoris not responsive to other treatments B) Decreased activity tolerance related to decreased cardiac output C) Atrial and ventricular tachycardias not responsive to other treatments D) Ventricular fibrillation not responsive to other treatments

Ans: Atrial and ventricular tachycardias not responsive to other treatments Feedback: Cardiac conduction surgery is considered in patients who do not respond to medications and antitachycardia pacing. Angina, reduced activity tolerance, and ventricular fibrillation are not criteria.

A patient with mitral stenosis exhibits new symptoms of a dysrhythmia. Based on the pathophysiology of this disease process, the nurse would expect the patient to exhibit what heart rhythm? A) Ventricular fibrillation (VF) B) Ventricular tachycardia (VT) C) Atrial fibrillation D) Sinus bradycardia

Ans: Atrial fibrillation Feedback: In patients with mitral valve stenosis, the pulse is weak and often irregular because of atrial fibrillation. Bradycardia, VF, and VT are not characteristic of this valvular disorder.

A patient with mitral valve stenosis is receiving health education at an outpatient clinic. To minimize the patient's symptoms, the nurse should teach the patient to do which of the following? A) Eat a high-protein, low-carbohydrate diet. B) Avoid activities that cause an increased heart rate. C) Avoid large crowds and public events. D) Perform deep breathing and coughing exercises.

Ans: Avoid activities that cause an increased heart rate. Feedback: Patients with mitral stenosis are advised to avoid strenuous activities, competitive sports, and pregnancy, all of which increase heart rate. Infection prevention is important, but avoiding crowds is not usually necessary. Deep breathing and coughing are not likely to prevent exacerbations of symptoms and increased protein intake is not necessary.

The nurse is planning discharge teaching for a patient with a newly inserted permanent pacemaker. What is the priority teaching point for this patient? A) Start lifting the arm above the shoulder right away to prevent chest wall adhesion. B) Avoid cooking with a microwave oven. C) Avoid exposure to high-voltage electrical generators. D) Avoid walking through store and library antitheft devices.

Ans: Avoid exposure to high-voltage electrical generators. Feedback: High-output electrical generators can reprogram pacemakers and should be avoided. Recent pacemaker technology allows patients to safely use most household electronic appliances and devices (e.g., microwave ovens). The affected arm should not be raised above the shoulder for 1 week following placement of the pacemaker. Antitheft alarms may be triggered so patients should be taught to walk through them quickly and avoid standing in or near these devices. These alarms generally do not interfere with pacemaker function.

A nurse is creating an education plan for a patient with venous insufficiency. What measure should the nurse include in the plan? A) Avoiding tight-fitting socks. B) Limit activity whenever possible. C) Sleep with legs in a dependent position. D) Avoid the use of pressure stockings.

Ans: Avoiding tight-fitting socks. Feedback: Measures taken to prevent complications include avoiding tight-fitting socks and panty girdles; maintaining activities, such as walking, sleeping with legs elevated, and using pressure stockings. Not included in the teaching plan for venous insufficiency would be reducing activity, sleeping with legs dependent, and avoiding pressure stockings. Each of these actions exacerbates venous insufficiency.

The critical care nurse is caring for a patient who is receiving cyclosporine postoperative heart transplant. The patient asks the nurse to remind him what this medication is for. How should the nurse best respond? A) Azathioprine decreases the risk of thrombus formation. B) Azathioprine ensures adequate cardiac output. C) Azathioprine increases the number of white blood cells. D) Azathioprine minimizes rejection of the transplant.

Ans: Azathioprine minimizes rejection of the transplant. Feedback: After heart transplant, patients are constantly balancing the risk of rejection with the risk of infection. Most commonly, patients receive cyclosporine or tacrolimus (FK506, Prograf), azathioprine (Imuran), or mycophenolate mofetil (CellCept), and corticosteroids (prednisone) to minimize rejection. Cyclosporine does not prevent thrombus formation, enhance cardiac output, or increase white cell counts.

The nurse is participating in the care conference for a patient with ACS. What goal should guide the care team's selection of assessments, interventions, and treatments? A) Maximizing cardiac output while minimizing heart rate B) Decreasing energy expenditure of the myocardium C) Balancing myocardial oxygen supply with demand D) Increasing the size of the myocardial muscle

Ans: Balancing myocardial oxygen supply with demand Feedback: Balancing myocardial oxygen supply with demand (e.g., as evidenced by the relief of chest pain) is the top priority in the care of the patient with ACS. Treatment is not aimed directly at minimizing heart rate because some patients experience bradycardia. Increasing the size of the myocardium is never a goal. Reducing the myocardium's energy expenditure is often beneficial, but this must be balanced with productivity.

A 48-year-old man presents to the ED complaining of severe substernal chest pain radiating down his left arm. He is admitted to the coronary care unit (CCU) with a diagnosis of myocardial infarction (MI). What nursing assessment activity is a priority on admission to the CCU? A) Begin ECG monitoring. B) Obtain information about family history of heart disease. C) Auscultate lung fields. D) Determine if the patient smokes.

Ans: Begin ECG monitoring. Feedback: The 12-lead ECG provides information that assists in ruling out or diagnosing an acute MI. It should be obtained within 10 minutes from the time a patient reports pain or arrives in the ED. By monitoring serial ECG changes over time, the location, evolution, and resolution of an MI can be identified and monitored; life-threatening arrhythmias are the leading cause of death in the first hours after an MI. Obtaining information about family history of heart disease and whether the patient smokes are not immediate priorities in the acute phase of MI. Data may be obtained from family members later. Lung fields are auscultated after oxygenation and pain control needs are met.

A patient has returned to the cardiac care unit after having a permanent pacemaker implantation. For which potential complication should the nurse most closely assess this patient? A) Chest pain B) Bleeding at the implantation site C) Malignant hyperthermia D) Bradycardia

Ans: Bleeding at the implantation site Feedback: Bleeding, hematomas, local infections, perforation of the myocardium, and tachycardia are complications of pacemaker implantations. The nurse should monitor for chest pain and bradycardia, but bleeding is a more common immediate complication. Malignant hyperthermia is unlikely because it is a response to anesthesia administration.

A patient has been admitted to the medical unit with signs and symptoms suggestive of endocarditis. The physician's choice of antibiotics would be primarily based on what diagnostic test? A) Echocardiography B) Blood cultures C) Cardiac aspiration D) Complete blood count

Ans: Blood cultures Feedback: To help determine the causative organisms and the most effective antibiotic treatment for the patient, blood cultures are taken. A CBC can help establish the degree and stage of infection, but not the causative microorganism. Echocardiography cannot indicate the microorganisms causing the infection. "Cardiac aspiration" is not a diagnostic test.

An older adult patient has been diagnosed with aortic regurgitation. What change in blood flow should the nurse expect to see on this patient's echocardiogram? A) Blood to flow back from the aorta to the left ventricle B) Obstruction of blood flow from the left ventricle C) Blood to flow back from the left atrium to the left ventricle D) Obstruction of blood from the left atrium to left ventricle

Ans: Blood to flow back from the aorta to the left ventricle Feedback: Aortic regurgitation occurs when the aortic valve does not completely close, and blood flows back to the left ventricle from the aorta during diastole. Aortic regurgitation does not cause obstruction of blood flow from the left ventricle, blood to flow back from the left atrium to the left ventricle, or obstruction of blood from the left atrium to left ventricle.

A patient in the cardiac step-down unit has begun bleeding from the percutaneous coronary intervention (PCI) access site in her femoral region. What is the nurse's most appropriate action? A) Call for assistance and initiate cardiopulmonary resuscitation. B) Reposition the patient's leg in a nondependent position. C) Promptly remove the femoral sheath. D) Call for help and apply pressure to the access site.

Ans: Call for help and apply pressure to the access site. Feedback: The femoral sheath produces pressure on the access site. Pressure will temporarily reduce bleeding and allow for subsequent interventions. Removing the sheath would exacerbate bleeding and repositioning would not halt it. CPR is not indicated unless there is evidence of respiratory or cardiac arrest.

The nurse is caring for a patient admitted with angina who is scheduled for cardiac catheterization. The patient is anxious and asks the reason for this test. What is the best response? A) "Cardiac catheterization is usually done to assess how blocked or open a patients coronary arteries are." B) "Cardiac catheterization is most commonly done to detect how efficiently a patient's heart muscle contracts." C) "Cardiac catheterization is usually done to evaluate cardiovascular response to stress." D) "Cardiac catheterization is most commonly done to evaluate cardiac electrical activity."

Ans: Cardiac catheterization is usually done to assess how blocked or open a patients coronary arteries are. Feedback: Cardiac catheterization is usually used to assess coronary artery patency to determine if revascularization procedures are necessary. A thallium stress test shows myocardial ischemia after stress. An ECG shows the electrical activity of the heart.

The nurse is caring for a patient who has just undergone catheter ablation therapy. The nurse in the step-down unit should prioritize what assessment? A) Cardiac monitoring B) Monitoring the implanted device signal C) Pain assessment D) Monitoring the patient's level of consciousness (LOC)

Ans: Cardiac monitoring Feedback: Following catheter ablation therapy, the patient is closely monitored to ensure the dysrhythmia does not reemerge. This is a priority over monitoring of LOC and pain, although these are valid and important assessments. Ablation does not involve the implantation of a device.

The OR nurse is explaining to a patient that cardiac surgery requires the absence of blood from the surgical field. At the same time, it is imperative to maintain perfusion of body organs and tissues. What technique for achieving these simultaneous goals should the nurse describe? A) Coronary artery bypass graft (CABG) B) Percutaneous transluminal coronary angioplasty (PTCA) C) Atherectomy D) Cardiopulmonary bypass

Ans: Cardiopulmonary bypass Feedback: Cardiopulmonary bypass is often used to circulate and oxygenate blood mechanically while bypassing the heart and lungs. PTCA, atherectomy, and CABG are all surgical procedures, none of which achieves the two goals listed.

The nurse is caring for an acutely ill patient who has central venous pressure monitoring in place. What intervention should be included in the care plan of a patient with CVP in place? A) Apply antibiotic ointment to the insertion site twice daily. B) Change the site dressing whenever it becomes visibly soiled. C) Perform passive range-of-motion exercises to prevent venous stasis. D) Aspirate blood from the device once daily to test pH.

Ans: Change the site dressing whenever it becomes visibly soiled. Feedback: Gauze dressings should be changed every 2 days or transparent dressings at least every 7 days and whenever dressings become damp, loosened, or visibly soiled. Passive ROM exercise is not indicated and it is unnecessary and inappropriate to aspirate blood to test it for pH. Antibiotic ointments are contraindicated.

The nurse is caring for a patient who has had a dysrhythmic event. The nurse is aware of the need to assess for signs of diminished cardiac output (CO). What change in status may signal to the nurse a decrease in cardiac output? A) Increased blood pressure B) Bounding peripheral pulses C) Changes in level of consciousness D) Skin flushing

Ans: Changes in level of consciousness Feedback: The nurse conducts a physical assessment to confirm the data obtained from the history and to observe for signs of diminished cardiac output (CO) during the dysrhythmic event, especially changes in level of consciousness. Blood pressure tends to decrease with lowered CO and bounding peripheral pulses are inconsistent with this problem. Pallor, not skin flushing, is expected.

A nurse is admitting a 45-year-old man to the medical unit who has a history of PAD. While providing his health history, the patient reveals that he smokes about two packs of cigarettes a day, has a history of alcohol abuse, and does not exercise. What would be the priority health education for this patient? A) The lack of exercise, which is the main cause of PAD. B) The likelihood that heavy alcohol intake is a significant risk factor for PAD. C) Cigarettes contain nicotine, which is a powerful vasoconstrictor and may cause or aggravate PAD. D) Alcohol suppresses the immune system, creates high glucose levels, and may cause PAD.

Ans: Cigarettes contain nicotine, which is a powerful vasoconstrictor and may cause or aggravate PAD. Feedback: Tobacco is powerful vasoconstrictor; its use with PAD is highly detrimental, and patients are strongly advised to stop using tobacco. Sedentary lifestyle is also a risk factor, but smoking is likely a more significant risk factor that the nurse should address. Alcohol use is less likely to cause PAD, although it carries numerous health risks.

A nurse has taken on the care of a patient who had a coronary artery stent placed yesterday. When reviewing the patient's daily medication administration record, the nurse should anticipate administering what drug? A) Ibuprofen B) Clopidogrel C) Dipyridamole D) Acetaminophen

Ans: Clopidogrel Feedback: Because of the risk of thrombus formation within the stent, the patient receives antiplatelet medications, usually aspirin and clopidogrel. Ibuprofen and acetaminophen are not antiplatelet drugs. Dipyridamole is not the drug of choice following stent placement.

A patient who is postoperative day 1 following a CABG has produced 20 mL of urine in the past 3 hours and the nurse has confirmed the patency of the urinary catheter. What is the nurse's most appropriate action? A) Document the patient's low urine output and monitor closely for the next several hours. B) Contact the dietitian and suggest the need for increased oral fluid intake. C) Contact the patient's physician and suggest assessment of fluid balance and renal function. D) Increase the infusion rate of the patient's IV fluid to prompt an increase in renal function.

Ans: Contact the patient's physician and suggest assessment of fluid balance and renal function. Feedback: Nursing management includes accurate measurement of urine output. An output of less than 1 mL/kg/h may indicate hypovolemia or renal insufficiency. Prompt referral is necessary. IV fluid replacement may be indicated, but is beyond the independent scope of the dietitian or nurse.

A patient converts from normal sinus rhythm at 80 bpm to atrial fibrillation with a ventricular response at 166 bpm. Blood pressure is 162/74 mm Hg. Respiratory rate is 20 breaths per minute with normal chest expansion and clear lungs bilaterally. IV heparin and Cardizem are given. The nurse caring for the patient understands that the main goal of treatment is what? treatment is what? A) Decrease SA node conduction B) Control ventricular heart rate C) Improve oxygenation D) Maintain anticoagulation

Ans: Control ventricular heart rate Feedback: Treatment for atrial fibrillation is to terminate the rhythm or to control ventricular rate. This is a priority because it directly affects cardiac output. A rapid ventricular response reduces the time for ventricular filling, resulting in a smaller stroke volume. Control of rhythm is the initial treatment of choice, followed by anticoagulation with heparin and then Coumadin.

A patient presents to the walk-in clinic complaining of intermittent chest pain on exertion, which is eventually attributed to angina. The nurse should inform the patient that angina is most often attributable to what cause? A) Decreased cardiac output B) Decreased cardiac contractility C) Infarction of the myocardium D) Coronary arteriosclerosis

Ans: Coronary arteriosclerosis Feedback: In most cases, angina pectoris is due to arteriosclerosis. The disease is not a result of impaired cardiac output or contractility. Infarction may result from untreated angina, but it is not a cause of the disease.

The cardiac nurse is caring for a patient who has been diagnosed with dilated cardiomyopathy (DCM). Echocardiography is likely to reveal what pathophysiological finding? A) Decreased ejection fraction B) Decreased heart rate C) Ventricular hypertrophy D) Mitral valve regurgitation

Ans: Decreased ejection fraction Feedback: DCM is distinguished by significant dilation of the ventricles without simultaneous hypertrophy. The ventricles have elevated systolic and diastolic volumes, but a decreased ejection fraction. Bradycardia and mitral valve regurgitation do not typically occur in patients with DCM.

A nurse in a long-term care facility is caring for an 83-year-old woman who has a history of HF and peripheral arterial disease (PAD). At present the patient is unable to stand or ambulate. The nurse should implement measures to prevent what complication? A) Aoritis B) Deep vein thrombosis C) Thoracic aortic aneurysm D) Raynaud's disease

Ans: Deep vein thrombosis Feedback: Although the exact cause of venous thrombosis remains unclear, three factors, known as Virchow's triad, are believed to play a significant role in its development: stasis of blood (venous stasis), vessel wall injury, and altered blood coagulation. In this woman's case, she has venous stasis from immobility, vessel wall injury from PAD, and altered blood coagulation from HF. The cause of aoritis is unknown, but it has no direct connection to HF, PAD, or mobility issues. The greatest risk factors for thoracic aortic aneurysm are atherosclerosis and hypertension; there is no direct connection to HF, PAD, or mobility issues. Raynaud's disease is a disorder that involves spasms of blood vessels and, again, no direct connection to HF, PAD, or mobility issues.

A nurse is caring for a patient who is exhibiting ventricular tachycardia (VT). Because the patient is pulseless, the nurse should prepare for what intervention? A) Defibrillation B) ECG monitoring C) Implantation of a cardioverter defibrillator D) Angioplasty

Ans: Defibrillation Feedback: Any type of VT in a patient who is unconscious and without a pulse is treated in the same manner as ventricular fibrillation: Immediate defibrillation is the action of choice. ECG monitoring is appropriate, but this is an assessment, not an intervention, and will not resolve the problem. An ICD and angioplasty do not address the dysrhythmia.

A patient who has undergone a valve replacement with a mechanical valve prosthesis is due to be discharged home. During discharge teaching, the nurse should discuss the importance of antibiotic prophylaxis prior to which of the following? A) Exposure to immunocompromised individuals B) Future hospital admissions C) Dental procedures D) Live vaccinations

Ans: Dental procedures Feedback: Following mechanical valve replacement, antibiotic prophylaxis is necessary before dental procedures involving manipulation of gingival tissue, the periapical area of the teeth or perforation of the oral mucosa (not including routine anesthetic injections, placement of orthodontic brackets, or loss of deciduous teeth). There are no current recommendations around antibiotic prophylaxis prior to vaccination, future hospital admissions, or exposure to people who are immunosuppressed.

A patient is scheduled for catheter ablation therapy. When describing this procedure to the patient's family, the nurse should address what aspect of the treatment? A) Resetting of the heart's contractility B) Destruction of specific cardiac cells C) Correction of structural cardiac abnormalities D) Clearance of partially occluded coronary arteries

Ans: Destruction of specific cardiac cells Feedback: Catheter ablation destroys specific cells that are the cause or central conduction route of a tachydysrhythmia. It does not "reset" the heart's contractility and it does not address structural or vascular abnormalities.

A patient is admitted to the cardiac care unit for an electrophysiology (EP) study. What goal should guide the planning and execution of the patient's care? A) Ablate the area causing the dysrhythmia. B) Freeze hypersensitive cells. C) Diagnose the dysrhythmia. D) Determine the nursing plan of care.

Ans: Diagnose the dysrhythmia. Feedback: A patient may undergo an EP study in which electrodes are placed inside the heart to obtain an intracardiac ECG. This is used not only to diagnose the dysrhythmia but also to determine the most effective treatment plan. However, because an EP study is invasive, it is performed in the hospital and may require that the patient be admitted.

The public health nurse is participating in a health fair and interviews a patient with a history of hypertension, who is currently smoking one pack of cigarettes per day. She denies any of the most common manifestations of CAD. Based on these data, the nurse would expect the focuses of CAD treatment most likely to be which of the following? A) Drug therapy and smoking cessation B) Diet and drug therapy C) Diet therapy only D) Diet therapy and smoking cessation

Ans: Diet therapy and smoking cessation Feedback: Due to the absence of symptoms, dietary therapy would likely be selected as the first-line treatment for possible CAD. Drug therapy would be determined based on a number of considerations and diagnostics findings, but would not be directly indicated. Smoking cessation is always indicated, regardless of the presence or absence of symptoms.

The nurse is reviewing the echocardiography results of a patient who has just been diagnosed with dilated cardiomyopathy (DCM). What changes in heart structure characterize DCM? A) Dilated ventricles with atrophy of the ventricles B) Dilated ventricles without hypertrophy of the ventricles C) Dilation and hypertrophy of all four heart chambers D) Dilation of the atria and hypertrophy of the ventricles

Ans: Dilated ventricles without hypertrophy of the ventricles Feedback: DCM is characterized by significant dilation of the ventricles without significant concomitant hypertrophy and systolic dysfunction. The ventricles do not atrophy in patients with DCM.

The nurse working on the coronary care unit is caring for a patient with ACS. How can the nurse best meet the patient's psychosocial needs? A) Reinforce the fact that treatment will be successful. B) Facilitate a referral to a chaplain or spiritual leader. C) Increase the patient's participation in rehabilitation activities. D) Directly address the patient's anxieties and fears.

Ans: Directly address the patient's anxieties and fears. Feedback: Alleviating anxiety and decreasing fear are important nursing functions that reduce the sympathetic stress response. Referrals to spiritual care may or may not be appropriate, and this does not relieve the nurse of responsibility for addressing the patient's psychosocial needs. Treatment is not always successful, and false hope should never be fostered. Participation in rehabilitation may alleviate anxiety for some patients, but it may exacerbate it for others.

Following cardiac resuscitation, a patient has been placed in a state of mild hypothermia before being transferred to the cardiac intensive care unit. The nurse's assessment reveals that the patient is experiencing neuromuscular paralysis. How should the nurse best respond? A) Administer hypertonic IV solution. B) Administer a bolus of warned normal saline. C) Reassess the patient in 15 minutes. D) Document this as an expected assessment finding

Ans: Document this as an expected assessment finding. Feedback: The nurse caring for a patient with hypothermia (passive or induced) needs to monitor for appropriate level of cooling, sedation, and neuromuscular paralysis to prevent seizures; myoclonus; and shivering. Neuromuscular paralysis is an expected finding and does not necessitate further interventions.

When assessing venous disease in a patient's lower extremities, the nurse knows that what test will most likely be ordered? A) Duplex ultrasonography B) Echocardiography C) Positron emission tomography (PET) D) Radiography

Ans: Duplex ultrasonography Feedback: Duplex ultrasound may be used to determine the level and extent of venous disease as well as its chronicity. Radiographs (x-rays), PET scanning, and echocardiography are never used for this purpose as they do not allow visualization of blood flow.

The nurse is assessing a patient with acute coronary syndrome (ACS). The nurse includes a careful history in the assessment, especially with regard to signs and symptoms. What signs and symptoms are suggestive of ACS? Select all that apply. A) Dyspnea B) Unusual fatigue C) Hypotension D) Syncope E) Peripheral cyanosis

Ans: Dyspnea, Unusual fatigue, Syncope Feedback: Systematic assessment includes a careful history, particularly as it relates to symptoms: chest pain or discomfort, difficulty breathing (dyspnea), palpitations, unusual fatigue, faintness (syncope), or sweating (diaphoresis). Each symptom must be evaluated with regard to time, duration, and the factors that precipitate the symptom and relieve it, and in comparison with previous symptoms. Hypotension and peripheral cyanosis are not typically associated with ACS.

The nurse is caring for a patient who is in the recovery room following the implantation of an ICD. The patient has developed ventricular tachycardia (VT). What should the nurse assess and document? A) ECG to compare time of onset of VT and onset of device's shock B) ECG so physician can see what type of dysrhythmia the patient has C) Patient's level of consciousness (LOC) at the time of the dysrhythmia D) Patient's activity at time of dysrhythmia

Ans: ECG to compare time of onset of VT and onset of device's shock Feedback: If the patient has an ICD implanted and develops VT or ventricular fibrillation, the ECG should be recorded to note the time between the onset of the dysrhythmia and the onset of the device's shock or antitachycardia pacing. This is a priority over LOC or activity at the time of onset.

The nurse is admitting a 32-year-old woman to the presurgical unit. The nurse learns during the admission assessment that the patient takes oral contraceptives. Consequently, the nurse's postoperative plan of care should include what intervention? A) Early ambulation and leg exercises B) Cessation of the oral contraceptives until 3 weeks postoperative C) Doppler ultrasound of peripheral circulation twice daily D) Dependent positioning of the patient's extremities when at rest

Ans: Early ambulation and leg exercises Feedback: Oral contraceptive use increases blood coagulability; with bed rest, the patient may be at increased risk of developing deep vein thrombosis. Leg exercises and early ambulation are among the interventions that address this risk. Assessment of peripheral circulation is important, but Doppler ultrasound may not be necessary to obtain these data. Dependent positioning increases the risk of venous thromboembolism (VTE). Contraceptives are not normally discontinued to address the risk of VTE in the short term.

The nurse is evaluating a patient's diagnosis of arterial insufficiency with reference to the adequacy of the patient's blood flow. On what physiological variables does adequate blood flow depend? Select all that apply. A) Efficiency of heart as a pump B) Adequacy of circulating blood volume C) Ratio of platelets to red blood cells D) Size of red blood cells E) Patency and responsiveness of the blood vessels

Ans: Efficiency of heart as a pump, Adequacy of circulating blood volume, Patency and responsiveness of the blood vessels Feedback: Adequate blood flow depends on the efficiency of the heart as a pump, the patency and responsiveness of the blood vessels, and the adequacy of circulating blood volume. Adequacy of blood flow does not primarily depend on the size of red cells or their ratio to the number of platelets.

How should the nurse best position a patient who has leg ulcers that are venous in origin? A) Keep the patient's legs flat and straight. B) Keep the patient's knees bent to 45-degree angle and supported with pillows. C) Elevate the patient's lower extremities. D) Dangle the patient's legs over the side of the bed.

Ans: Elevate the patient's lower extremities. Feedback: Positioning of the legs depends on whether the ulcer is of arterial or venous origin. With venous insufficiency, dependent edema can be avoided by elevating the lower extremities. Dangling the patient's legs and applying pillows may further compromise venous return.

The patient has just returned to the floor after balloon valvuloplasty of the aortic valve and the nurse is planning appropriate assessments. The nurse should know that complications following this procedure include what? Select all that apply. A) Emboli B) Mitral valve damage C) Ventricular dysrhythmia D) Atrial-septal defect E) Plaque formation

Ans: Emboli, Mitral valve damage, Ventricular dysrhythmia Feedback: Possible complications include aortic regurgitation, emboli, ventricular perforation, rupture of the aortic valve annulus, ventricular dysrhythmia, mitral valve damage, and bleeding from the catheter insertion sites. Atrial-septal defect and plaque formation are not complications of a balloon valvuloplasty.

A nurse has written a plan of care for a man diagnosed with peripheral arterial insufficiency. One of the nursing diagnoses in the care plan is altered peripheral tissue perfusion related to compromised circulation. What is the most appropriate intervention for this diagnosis? A) Elevate his legs and arms above his heart when resting. B) Encourage the patient to engage in a moderate amount of exercise. C) Encourage extended periods of sitting or standing. D) Discourage walking in order to limit pain.

Ans: Encourage the patient to engage in a moderate amount of exercise. Feedback: The nursing diagnosis of altered peripheral tissue perfusion related to compromised circulation requires interventions that focus on improving circulation. Encouraging the patient to engage in a moderate amount of exercise serves to improve circulation. Elevating his legs and arms above his heart when resting would be passive and fails to promote circulation. Encouraging long periods of sitting or standing would further compromise circulation. The nurse should encourage, not discourage, walking to increase circulation and decrease pain.

The ED nurse is caring for a patient who has gone into cardiac arrest. During external defibrillation, what action should the nurse perform? A) Place gel pads over the apex and posterior chest for better conduction. B) Ensure no one is touching the patient at the time shock is delivered. C) Continue to ventilate the patient via endotracheal tube during the procedure. D) Allow at least 3 minutes between shocks.

Ans: Ensure no one is touching the patient at the time shock is delivered. Feedback: In external defibrillation, both paddles may be placed on the front of the chest, which is the standard paddle placement. Whether using pads, or paddles, the nurse must observe two safety measures. First, maintain good contact between the pads or paddles and the patient's skin to prevent leaking. Second, ensure that no one is in contact with the patient or with anything that is touching the patient when the defibrillator is discharged, to minimize the chance that electrical current will be conducted to anyone other than the patient. Ventilation should be stopped during defibrillation.

A patient with advanced venous insufficiency is confined following orthopedic surgery. How can the nurse best prevent skin breakdown in the patient's lower extremities? A) Ensure that the patient's heels are protected and supported. B) Closely monitor the patient's serum albumin and prealbumin levels. C) Perform gentle massage of the patient's lower legs, as tolerated. D) Perform passive range-of-motion exercises once per shift.

Ans: Ensure that the patient's heels are protected and supported. Feedback: If the patient is on bed rest, it is important to relieve pressure on the heels to prevent pressure ulcerations, since the heels are among the most vulnerable body regions. Monitoring blood work does not directly prevent skin breakdown, even though albumin is related to wound healing. Massage is not normally indicated and may exacerbate skin breakdown. Passive range- of-motion exercises do not directly reduce the risk of skin breakdown.

The nurse and the other members of the team are caring for a patient who converted to ventricular fibrillation (VF). The patient was defibrillated unsuccessfully and the patient remains in VF. According to national standards, the nurse should anticipate the administration of what medication? treatment is what? A) Decrease SA node conduction B) Control ventricular heart rate C) Improve oxygenation D) Maintain anticoagulation

Ans: Epinephrine 1 mg IV push Feedback: Epinephrine should be administered as soon as possible after the first unsuccessful defibrillation and then every 3 to 5 minutes. Antiarrhythmic medications such as amiodarone and licocaine are given if ventricular dysrhythmia persists.

A patient has been scheduled for cardiovascular computed tomography (CT) with contrast. To prepare the patient for this test, what action should the nurse perform? A) Keep the patient NPO for at least 6 hours prior to the test. B) Establish peripheral IV access. C) Limit the patient's activity for 2 hours before the test. D) Teach the patient to perform incentive spirometry.

Ans: Establish peripheral IV access. Feedback: An IV is necessary if contrast is to be used to enhance the images of the CT. The patient does not need to fast or limit his or her activity. Incentive spirometry is not relevant to this diagnostic test.

The nurse is relating the deficits in a patient's synchronization of the atrial and ventricular events to his diagnosis. What are the physiologic characteristics of the nodal and Purkinje cells that provide this synchronization? Select all that apply. A) Loop connectivity B) Excitability C) Automaticity D) Conductivity E) Independence

Ans: Excitability, Automaticity, Conductivity Feedback: Three physiologic characteristics of two types of specialized electrical cells, the nodal cells and the Purkinje cells, provide this synchronization: automaticity, or the ability to initiate an electrical impulse; excitability, or the ability to respond to an electrical impulse; and conductivity, the ability to transmit an electrical impulse from one cell to another. Loop connectivity is a distracter for this question. Independence of the cells has nothing to do with the synchronization described in the scenario.

When discussing angina pectoris secondary to atherosclerotic disease with a patient, the patient asks why he tends to experience chest pain when he exerts himself. The nurse should describe which of the following phenomena? A) Exercise increases the heart's oxygen demands. B) Exercise causes vasoconstriction of the coronary arteries. C) Exercise shunts blood flow from the heart to the mesenteric area. D) Exercise increases the metabolism of cardiac medications.

Ans: Exercise increases the heart's oxygen demands. Feedback: Physical exertion increases the myocardial oxygen demand. If the patient has arteriosclerosis of the coronary arteries, then blood supply is diminished to the myocardium. Exercise does not cause vasoconstriction or interfere with drug metabolism. Exercise does not shunt blood flow away from the heart.

The nurse has just admitted a 66-year-old patient for cardiac surgery. The patient tearfully admits to the nurse that she is afraid of dying while undergoing the surgery. What is the nurse's best response? A) Explore the factors underlying the patient's anxiety. B) Teach the patient guided imagery techniques. C) Obtain an order for a PRN benzodiazepine. D) Describe the procedure in greater detail.

Ans: Explore the factors underlying the patient's anxiety. Feedback: An assessment of anxiety levels is required in the patient to assist the patient in identifying fears and developing coping mechanisms for those fears. The nurse must further assess and explore the patient's anxiety before providing interventions such as education or medications.

An adult patient with third-degree AV block is admitted to the cardiac care unit and placed on continuous cardiac monitoring. What rhythm characteristic will the ECG most likely show? A) PP interval and RR interval are irregular. B) PP interval is equal to RR interval. C) Fewer QRS complexes than P waves D) PR interval is constant.

Ans: Fewer QRS complexes than P waves Feedback: In third-degree AV block, no atrial impulse is conducted through the AV node into the ventricles. As a result, there are impulses stimulating the atria and impulses stimulating the ventricles. Therefore, there are more P waves than QRS complexes due to the difference in the natural pacemaker (nodes) rates of the heart. The other listed ECG changes are not consistent with this diagnosis.

A patient who has recently recovered from a systemic viral infection is undergoing diagnostic testing for myocarditis. Which of the nurse's assessment findings is most consistent with myocarditis? A) Sudden changes in level of consciousness (LOC) B) Peripheral edema and pulmonary edema C) Pleuritic chest pain D) Flulike symptoms

Ans: Flulike symptoms Feedback: The most common symptoms of myocarditis are flulike. Chest pain, edema, and changes in LOC are not characteristic of myocarditis.

The nurse is auscultating the breath sounds of a patient with pericarditis. What finding is most consistent with this diagnosis? A) Wheezes B) Friction rub C) Fine crackles D) Coarse crackles

Ans: Friction rub Feedback: A pericardial friction rub is diagnostic of pericarditis. Crackles are associated with pulmonary edema and fluid accumulation, whereas wheezes signal airway constriction; neither of these occurs with pericarditis.

An ECG has been ordered for a newly admitted patient. What should the nurse do prior to electrode placement? A) Clean the skin with providone-iodine solution. B) Ensure that the area for electrode placement is dry. C) Apply tincture of benzoin to the electrode sites and wait for it to become "tacky." D) Gently abrade the skin by rubbing the electrode sites with dry gauze or cloth.

Ans: Gently abrade the skin by rubbing the electrode sites with dry gauze or cloth. Feedback: An ECG is obtained by slightly abrading the skin with a clean dry gauze pad and placing electrodes on the body at specific areas. The abrading of skin will enhance signal transmission. Disinfecting the skin is unnecessary and conduction gel is used.

A patient is brought into the ED by family members who tell the nurse the patient grabbed his chest and complained of substernal chest pain. The care team recognizes the need to monitor the patient's cardiac function closely while interventions are performed. What form of monitoring should the nurse anticipate? A) Left-sided heart catheterization B) Cardiac telemetry C) Transesophageal echocardiography D) Hardwire continuous ECG monitoring

Ans: Hardwire continuous ECG monitoring Feedback: Two types of continuous ECG monitoring techniques are used in health care settings: hardwire cardiac monitoring, found in EDs, critical care units, and progressive care units; and telemetry, found in general nursing care units or outpatient cardiac rehabilitation programs. Cardiac catheterization and transesophageal echocardiography would not be used in emergent situations to monitor cardiac function.

A brain (B-type) natriuretic peptide (BNP) sample has been drawn from an older adult patient who has been experienced vital fatigue and shortness of breath. This test will allow the care team to investigate the possibility of what diagnosis? A) Pleurisy B) Heart failure C) Valve dysfunction D) Cardiomyopathy

Ans: Heart failure Feedback: The level of BNP in the blood increases as the ventricular walls expand from increased pressure, making it a helpful diagnostic, monitoring, and prognostic tool in the setting of HF. It is not specific to cardiomyopathy, pleurisy, or valve dysfunction.

A patient has been living with dilated cardiomyopathy for several years but has experienced worsening symptoms despite aggressive medical management. The nurse should anticipate what potential treatment? A) Heart transplantation B) Balloon valvuloplasty C) Cardiac catheterization D) Stent placement

Ans: Heart transplantation Feedback: When heart failure progresses and medical treatment is no longer effective, surgical intervention, including heart transplantation, is considered. Valvuloplasty, stent placement, and cardiac catheterization will not address the pathophysiology of cardiomyopathy.

The nurse on the hospital's infection control committee is looking into two cases of hospital-acquired infective endocarditis among a specific classification of patients. What classification of patients would be at greatest risk for hospital-acquired endocarditis? A) Hemodialysis patients B) Patients on immunoglobulins C) Patients who undergo intermittent urinary catheterization D) Children under the age of 12

Ans: Hemodialysis patients Feedback: Hospital-acquired infective endocarditis occurs most often in patients with debilitating disease or indwelling catheters and in patients who are receiving hemodialysis or prolonged IV fluid or antibiotic therapy. Patients taking immunosuppressive medications or corticosteroids are more susceptible to fungal endocarditis. Patients on immunoglobulins, those who need in and out catheterization, and children are not at increased risk for nosocomial infective endocarditis.

The critical care nurse is caring for a patient with a central venous pressure (CVP) monitoring system. The nurse notes that the patient's CVP is increasing. Of what may this indicate? A) Psychosocial stress B) Hypervolemia C) Dislodgment of the catheter D) Hypomagnesemia

Ans: Hypervolemia Feedback: CVP is a useful hemodynamic parameter to observe when managing an unstable patient's fluid volume status. An increasing pressure may be caused by hypervolemia or by a condition, such as heart failure, that results in decreased myocardial contractility. Stress, dislodgement of the catheter, and low magnesium levels would not typically result in increased CVP.

The nurse caring for a patient with a leg ulcer has finished assessing the patient and is developing a problem list prior to writing a plan of care. What major nursing diagnosis might the care plan include? A) Risk for disuse syndrome B) Ineffective health maintenance C) Sedentary lifestyle D) Imbalanced nutrition: less than body requirements

Ans: Imbalanced nutrition: less than body requirements Feedback: Major nursing diagnoses for the patient with leg ulcers may include imbalanced nutrition: less than body requirements, related to increased need for nutrients that promote wound healing. Risk for disuse syndrome is a state in which an individual is at risk for deterioration of body systems owing to prescribed or unavoidable musculoskeletal inactivity. A leg ulcer will affect activity, but rarely to this degree. Leg ulcers are not necessarily a consequence of ineffective health maintenance or sedentary lifestyle.

While assessing a patient the nurse notes that the patient's ankle-brachial index (ABI) of the right leg is 0.40. How should the nurse best respond to this assessment finding? A) Assess the patient's use of over-the-counter dietary supplements. B) Implement interventions relevant to arterial narrowing. C) Encourage the patient to increase intake of foods high in vitamin K. D) Adjust the patient's activity level to accommodate decreased coronary output.

Ans: Implement interventions relevant to arterial narrowing. Feedback: ABI is used to assess the degree of stenosis of peripheral arteries. An ABI of less than 1.0 indicates possible claudication of the peripheral arteries. It does not indicate inadequate coronary output. There is no direct indication for changes in vitamin K intake and OTC medications are not likely causative.

The nurse is creating a plan of care for a patient with a cardiomyopathy. What priority goal should underlie most of the assessments and interventions that are selected for this patient? A) Absence of complications B) Adherence to the self-care program C) Improved cardiac output D) Increased activity tolerance

Ans: Improved cardiac output Feedback: The priority nursing diagnosis of a patient with cardiomyopathy would include improved or maintained cardiac output. Regardless of the category and cause, cardiomyopathy may lead to severe heart failure, lethal dysrhythmias, and death. The pathophysiology of all cardiomyopathies is a series of progressive events that culminate in impaired cardiac output. Absence of complications, adherence to the self-care program, and increased activity tolerance should be included in the care plan, but they do not have the priority of improved cardiac output.

The nurse is caring for a patient who has undergone percutaneous transluminal coronary angioplasty (PTCA). What is the major indicator of success for this procedure? A) Increase in the size of the artery's lumen B) Decrease in arterial blood flow in relation to venous flow C) Increase in the patient's resting heart rate D) Increase in the patient's level of consciousness (LOC)

Ans: Increase in the size of the artery's lumen Feedback: PTCA is used to open blocked coronary vessels and resolve ischemia. The procedure may result in beneficial changes to the patient's LOC or heart rate, but these are not the overarching goals of PTCA. Increased arterial flow is the focus of the procedures.

The nurse is conducting patient teaching about cholesterol levels. When discussing the patient's elevated LDL and lowered HDL levels, the patient shows an understanding of the significance of these levels by stating what? A) "Increased LDL and decreased HDL increase my risk of coronary artery disease." B) "Increased LDL has the potential to decrease my risk of heart disease." C) "The decreased HDL level will increase the amount of cholesterol moved away from the artery walls." D) "The increased LDL will decrease the amount of cholesterol deposited on the artery walls."

Ans: Increased LDL and decreased HDL increase my risk of coronary artery disease. Feedback: Elevated LDL levels and decreased HDL levels are associated with a greater incidence of coronary artery disease.

A cardiac care nurse is aware of factors that result in positive chronotropy. These factors would affect a patient's cardiac function in what way? A) Exacerbating an existing dysrhythmia B) Initiating a new dysrhythmia C) Resolving ventricular tachycardia D) Increasing the heart rate

Ans: Increasing the heart rate Feedback: Stimulation of the sympathetic system increases heart rate. This phenomenon is known as positive chronotropy. It does not influence dysrhythmias.

The nurse is caring for an adult patient who had symptoms of unstable angina upon admission to the hospital. What nursing diagnosis underlies the discomfort associated with angina? A) Ineffective breathing pattern related to decreased cardiac output B) Anxiety related to fear of death C) Ineffective cardiopulmonary tissue perfusion related to coronary artery disease (CAD) D) Impaired skin integrity related to CAD

Ans: Ineffective cardiopulmonary tissue perfusion related to coronary artery disease (CAD) Feedback: Ineffective cardiopulmonary tissue perfusion directly results in the symptoms of discomfort associated with angina. Anxiety and ineffective breathing may result from angina chest pain, but they are not the causes. Skin integrity is not impaired by the effects of angina.

The nurse is assessing a patient who had a pacemaker implanted 4 weeks ago. During the patient's most recent follow-up appointment, the nurse identifies data that suggest the patient may be socially isolated and depressed. What nursing diagnosis is suggested by these data? A) Decisional conflict related to pacemaker implantation B) Deficient knowledge related to pacemaker implantation C) Spiritual distress related to pacemaker implantation D) Ineffective coping related to pacemaker implantation

Ans: Ineffective coping related to pacemaker implantation Feedback: Depression and isolation may be symptoms of ineffective coping with the implantation. These psychosocial symptoms are not necessarily indicative of issues related to knowledge or decisions. Further data would be needed to determine a spiritual component to the patient's challenges.

You are caring for a patient who is diagnosed with Raynaud's phenomenon. The nurse should plan interventions to address what nursing diagnosis? A) Chronic pain B) Ineffective tissue perfusion C) Impaired skin integrity D) Risk for injury

Ans: Ineffective tissue perfusion Feedback: Raynaud's phenomenon is a form of intermittent arteriolar vasoconstriction resulting in inadequate tissue perfusion. This results in coldness, pain, and pallor of the fingertips or toes. Pain is typically intermittent and acute, not chronic, and skin integrity is rarely at risk. In most cases, the patient is not at a high risk for injury.

The physician has ordered a high-sensitivity C-reactive protein (hs-CRP) drawn on a patient. The results of this test will allow the nurse to evaluate the role of what process that is implicated in the development of atherosclerosis? A) Immunosuppression B) Inflammation C) Infection D) Hemostasis

Ans: Inflammation Feedback: High-sensitivity CRP is a protein produced by the liver in response to systemic inflammation. Inflammation is thought to play a role in the development and progression of atherosclerosis.

A patient with hypertrophic cardiomyopathy (HCM) has been admitted to the medical unit. During the nurse's admission interview, the patient states that she takes over-the-counter "water pills" on a regular basis. How should the nurse best respond to the fact that the patient has been taking diuretics? A) Encourage the patient to drink at least 2 liters of fluid daily. B) Increase the patient's oral sodium intake. C) Inform the care provider because diuretics are contraindicated. D) Ensure that the patient's fluid balance is monitored vigilantly.

Ans: Inform the care provider because diuretics are contraindicated. Feedback: Diuretics are contraindicated in patients with HCM, so the primary care provider should be made aware. Adjusting the patient's sodium or fluid intake or fluid monitoring does not address this important contraindication.

A nurse is preparing a patient for scheduled transesophageal echocardiography. What action should the nurse perform? A) Instruct the patient to drink 1 liter of water before the test. B) Administer IV benzodiazepines and opioids. C) Inform the patient that she will remain on bed rest following the procedure. D) Inform the patient that an access line will be initiated in her femoral artery.

Ans: Inform the patient that she will remain on bed rest following the procedure. Feedback: During the recovery period, the patient must maintain bed rest with the head of the bed elevated to 45 degrees. The patient must be NPO 6 hours preprocedure. The patient is sedated to make him or her comfortable, but will not be heavily sedated, and opioids are not necessary. Also, the patient will have a peripheral IV line initiated preprocedure.

.The nurse is assessing a patient who was admitted to the critical care unit 3 hours ago following cardiac surgery. The nurse's most recent assessment reveals that the patient's left pedal pulses are not palpable and that the right pedal pulses are rated at +2. What is the nurse's best response? A) Document this expected assessment finding during the initial postoperative period. B) Reposition the patient with his left leg in a dependent position. C) Inform the patient's physician of this assessment finding. D) Administer an ordered dose of subcutaneous heparin.

Ans: Inform the patient's physician of this assessment finding. Feedback: If a pulse is absent in any extremity, the cause may be prior catheterization of that extremity, chronic peripheral vascular disease, or a thromboembolic obstruction. The nurse immediately reports newly identified absence of any pulse.

A postsurgical patient has illuminated her call light to inform the nurse of a sudden onset of lower leg pain. On inspection, the nurse observes that the patient's left leg is visibly swollen and reddened. What is the nurse's most appropriate action? A) Administer a PRN dose of subcutaneous heparin. B) Inform the physician that the patient has signs and symptoms of VTE. C) Mobilize the patient promptly to dislodge any thrombi in the patient's lower leg. D) Massage the patient's lower leg to temporarily restore venous return.

Ans: Inform the physician that the patient has signs and symptoms of VTE. Feedback: VTE requires prompt medical follow-up. Heparin will not dissolve an established clot. Massaging the patient's leg and mobilizing the patient would be contraindicated because they would dislodge the clot, possibly resulting in a pulmonary embolism.

The nurse is taking a health history of a new patient. The patient reports experiencing pain in his left lower leg and foot when walking. This pain is relieved with rest. The nurse notes that the left lower leg is slightly edematous and is hairless. When planning this patient's subsequent care, the nurse should most likely address what health problem? A) Coronary artery disease (CAD) B) Intermittent claudication C) Arterial embolus D) Raynaud's disease

Ans: Intermittent claudication Feedback: A muscular, cramp-type pain in the extremities consistently reproduced with the same degree of exercise or activity and relieved by rest is experienced by patients with peripheral arterial insufficiency. Referred to as intermittent claudication, this pain is caused by the inability of the arterial system to provide adequate blood flow to the tissues in the face of increased demands for nutrients and oxygen during exercise. The nurse would not suspect the patient has CAD, arterial embolus, or Raynaud's disease; none of these health problems produce this cluster of signs and symptoms.

The nurse is preparing to administer warfarin (Coumadin) to a client with deep vein thrombophlebitis (DVT). Which laboratory value would most clearly indicate that the patient's warfarin is at therapeutic levels? A) Partial thromboplastin time (PTT) within normal reference range B) Prothrombin time (PT) eight to ten times the control C) International normalized ratio (INR) between 2 and 3 D) Hematocrit of 32%

Ans: International normalized ratio (INR) between 2 and 3 Feedback: The INR is most often used to determine if warfarin is at a therapeutic level; an INR of 2 to 3 is considered therapeutic. Warfarin is also considered to be at therapeutic levels when the client's PT is 1.5 to 2 times the control. Higher values indicate increased risk of bleeding and hemorrhage, whereas lower values indicate increased risk of blood clot formation. Heparin, not warfarin, prolongs PTT. Hematocrit does not provide information on the effectiveness of warfarin; however, a falling hematocrit in a client taking warfarin may be a sign of hemorrhage.

Preoperative education is an important part of the nursing care of patients having coronary artery revascularization. When explaining the pre- and postoperative regimens, the nurse would be sure to include education about which subject? A) Symptoms of hypovolemia B) Symptoms of low blood pressure C) Complications requiring graft removal D) Intubation and mechanical ventilation

Ans: Intubation and mechanical ventilation Feedback: Most patients remain intubated and on mechanical ventilation for several hours after surgery. It is important that patients realize that this will prevent them from talking, and the nurse should reassure them that the staff will be able to assist them with other means of communication. Teaching would generally not include symptoms of low blood pressure or hypovolemia, as these are not applicable to most patients. Teaching would also generally not include rare complications that would require graft removal.

The triage nurse in the ED assesses a 66-year-old male patient who presents to the ED with complaints of midsternal chest pain that has lasted for the last 5 hours. If the patient's symptoms are due to an MI, what will have happened to the myocardium? A) It may have developed an increased area of infarction during the time without treatment. B) It will probably not have more damage than if he came in immediately. C) It may be responsive to restoration of the area of dead cells with proper treatment. D) It has been irreparably damaged, so immediate treatment is no longer necessary.

Ans: It may have developed an increased area of infarction during the time without treatment. Feedback: When the patient experiences lack of oxygen to myocardium cells during an MI, the sooner treatment is initiated, the more likely the treatment will prevent or minimize myocardial tissue necrosis. Delays in treatment equate with increased myocardial damage. Despite the length of time the symptoms have been present, treatment needs to be initiated immediately to minimize further damage. Dead cells cannot be restored by any means.

During a shift assessment, the nurse is identifying the client's point of maximum impulse (PMI). Where will the nurse best palpate the PMI? A) Left midclavicular line of the chest at the level of the nipple B) Left midclavicular line of the chest at the fifth intercostal space C) Midline between the xiphoid process and the left nipple D) Two to three centimeters to the left of the sternum

Ans: Left midclavicular line of the chest at the fifth intercostal space Feedback: The left ventricle is responsible for the apical beat or the point of maximum impulse, which is normally palpated in the left midclavicular line of the chest wall at the fifth intercostal space.

.The staff educator is presenting a workshop on valvular disorders. When discussing the pathophysiology of aortic regurgitation the educator points out the need to emphasize that aortic regurgitation causes what? A) Cardiac tamponade B) Left ventricular hypertrophy C) Right-sided heart failure D) Ventricular insufficiency

Ans: Left ventricular hypertrophy Feedback: Aortic regurgitation eventually causes left ventricular hypertrophy. In aortic regurgitation, blood from the aorta returns to the left ventricle during diastole in addition to the blood normally delivered by the left atrium. The left ventricle dilates, trying to accommodate the increased volume of blood. Aortic regurgitation does not cause cardiac tamponade, right-sided heart failure, or ventricular insufficiency.

A critical care nurse is caring for a patient with a pulmonary artery catheter in place. What does this catheter measure that is particularly important in critically ill patients? A) Pulmonary artery systolic pressure B) Right ventricular afterload C) Pulmonary artery pressure D) Left ventricular preload

Ans: Left ventricular preload Feedback: Monitoring of the pulmonary artery diastolic and pulmonary artery wedge pressures is particularly important in critically ill patients because it is used to evaluate left ventricular filling pressures (i.e., left ventricular preload). This device does not directly measure the other listed aspects of cardiac function.

The nurse is caring for a patient who has been diagnosed with an elevated cholesterol level. The nurse is aware that plaque on the inner lumen of arteries is composed chiefly of what? A) Lipids and fibrous tissue B) White blood cells C) Lipoproteins D) High-density cholesterol

Ans: Lipids and fibrous tissue Feedback: As T-lymphocytes and monocytes infiltrate to ingest lipids on the arterial wall and then die, a fibrous tissue develops. This causes plaques to form on the inner lumen of arterial walls. These plaques do not consist of white cells, lipoproteins, or high-density cholesterol.

The nurse has performed a thorough nursing assessment of the care of a patient with chronic leg ulcers. The nurse's assessment should include which of the following components? Select all that apply. A) Location and type of pain B) Apical heart rate C) Bilateral comparison of peripheral pulses D) Comparison of temperature in the patient's legs E) Identification of mobility limitations

Ans: Location and type of pain, Bilateral comparison of peripheral pulses, Comparison of temperature in the patient's legs, Identification of mobility limitations Feedback: A careful nursing history and assessment are important. The extent and type of pain are carefully assessed, as are the appearance and temperature of the skin of both legs. The quality of all peripheral pulses is assessed, and the pulses in both legs are compared. Any limitation of mobility and activity that results from vascular insufficiency is identified. Not likely is there any direct indication for assessment of apical heart rate, although peripheral pulses must be assessed.

The nurse is caring for a patient who is scheduled to undergo mechanical valve replacement. Patient education should include which of the following? A) Use of patient-controlled analgesia B) Long-term anticoagulant therapy C) Steroid therapy D) Use of IV diuretics

Ans: Long-term anticoagulant therapy Feedback: Mechanical valves necessitate long-term use of required anticoagulants. Diuretics and steroids are not indicated and patient-controlled analgesia may or may be not be used in the immediate postoperative period.

The nurse is providing care for a patient with high cholesterol and triglyceride values. In teaching the patient about therapeutic lifestyle changes such as diet and exercise, the nurse realizes that the desired goal for cholesterol levels is which of the following? A) High HDL values and high triglyceride values B) Absence of detectable total cholesterol levels C) Elevated blood lipids, fasting glucose less than 100 D) Low LDL values and high HDL values

Ans: Low LDL values and high HDL values Feedback: The desired goal for cholesterol readings is for a patient to have low LDL and high HDL values. LDL exerts a harmful effect on the coronary vasculature because the small LDL particles can be easily transported into the vessel lining. In contrast, HDL promotes the use of total cholesterol by transporting LDL to the liver, where it is excreted. Elevated triglycerides are also a major risk factor for cardiovascular disease. A goal is also to keep triglyceride levels less than 150 mg/dL. All individuals possess detectable levels of total cholesterol.

The nurse is caring for an acutely ill patient who is on anticoagulant therapy. The patient has a comorbidity of renal insufficiency. How will this patient's renal status affect heparin therapy? A) Heparin is contraindicated in the treatment of this patient. B) Heparin may be administered subcutaneously, but not IV. C) Lower doses of heparin are required for this patient. D) Coumadin will be substituted for heparin.

Ans: Lower doses of heparin are required for this patient. Feedback: If renal insufficiency exists, lower doses of heparin are required. Coumadin cannot be safely and effectively used as a substitute and there is no contraindication for IV administration.

A patient comes to the walk-in clinic with complaints of pain in his foot following stepping on a roofing nail 4 days ago. The patient has a visible red streak running up his foot and ankle. What health problem should the nurse suspect? A) Cellulitis B) Local inflammation C) Elephantiasis D) Lymphangitis

Ans: Lymphangitis Feedback: Lymphangitis is an acute inflammation of the lymphatic channels. It arises most commonly from a focus of infection in an extremity. Usually, the infectious organism is hemolytic streptococcus. The characteristic red streaks that extend up the arm or the leg from an infected wound outline the course of the lymphatic vessels as they drain. Cellulitis is caused by bacteria, which cause a generalized edema in the subcutaneous tissues surrounding the affected area. Local inflammation would not present with red streaks in the lymphatic channels. Elephantiasis is transmitted by mosquitoes that carry parasitic worm larvae; the parasites obstruct the lymphatic channels and results in gross enlargement of the limbs.

The nurse is writing a plan of care for a patient with a cardiac dysrhythmia. What would be the most appropriate goal for the patient? A) Maintain a resting heart rate below 70 bpm. B) Maintain adequate control of chest pain. C) Maintain adequate cardiac output. D) Maintain normal cardiac structure.

Ans: Maintain adequate cardiac output. Feedback: For patient safety, the most appropriate goal is to maintain cardiac output to prevent worsening complications as a result of decreased cardiac output. A resting rate of less than 70 bpm is not appropriate for every patient. Chest pain is more closely associated with acute coronary syndrome than with dysrhythmias. Nursing actions cannot normally influence the physical structure of the heart.

The nurse is caring for an adult patient who has gone into ventricular fibrillation. When assisting with defibrillating the patient, what must the nurse do? A) Maintain firm contact between paddles and patient skin. B) Apply a layer of water as a conducting agent. C) Call "all clear" once before discharging the defibrillator. D) Ensure the defibrillator is in the sync mode.

Ans: Maintain firm contact between paddles and patient skin. Feedback: When defibrillating an adult patient, the nurse should maintain good contact between the paddles and the patient's skin to prevent arcing, apply an appropriate conducting agent (not water) between the skin and the paddles, and ensure the defibrillator is in the nonsync mode. "Clear" should be called three times before discharging the paddles.

A patient newly admitted to the telemetry unit is experiencing progressive fatigue, hemoptysis, and dyspnea. Diagnostic testing has revealed that these signs and symptoms are attributable to pulmonary venous hypertension. What valvular disorder should the nurse anticipate being diagnosed in this patient? A) Aortic regurgitation B) Mitral stenosis C) Mitral valve prolapse D) Aortic stenosis

Ans: Mitral stenosis Feedback: The first symptom of mitral stenosis is often dyspnea on exertion as a result of pulmonary venous hypertension. Symptoms usually develop after the valve opening is reduced by one-third to one-half its usual size. Patients are likely to show progressive fatigue as a result of low cardiac output. The enlarged left atrium may create pressure on the left bronchial tree, resulting in a dry cough or wheezing. Patients may expectorate blood (i.e., hemoptysis) or experience palpitations, orthopnea, paroxysmal nocturnal dyspnea (PND), and repeated respiratory infections. Pulmonary venous hypertension is not typically caused by aortic regurgitation, mitral valve prolapse, or aortic stenosis.

The nurse is caring for a patient with right ventricular hypertrophy and consequently decreased right ventricular function. What valvular disorder may have contributed to this patient's diagnosis? A) Mitral valve regurgitation B) Aortic stenosis C) Aortic regurgitation D) Mitral valve stenosis

Ans: Mitral valve stenosis Feedback: Because no valve protects the pulmonary veins from the backward flow of blood from the atrium, the pulmonary circulation becomes congested. As a result, the right ventricle must contract against an abnormally high pulmonary arterial pressure and is subjected to excessive strain. Eventually, the right ventricle fails. None of the other listed valvular disorders has this pathophysiological effect.

A patient is undergoing preoperative teaching before his cardiac surgery and the nurse is aware that a temporary pacemaker will be placed later that day. What is the nurse's responsibility in the care of the patient's pacemaker? A) Monitoring for pacemaker malfunction or battery failure B) Determining when it is appropriate to remove the pacemaker C) Making necessary changes to the pacemaker settings D) Selecting alternatives to future pacemaker use

Ans: Monitoring for pacemaker malfunction or battery failure Feedback: Monitoring for pacemaker malfunctioning and battery failure is a nursing responsibility. The other listed actions are physician responsibilities.

The ED nurse is caring for a patient with a suspected MI. What drug should the nurse anticipate administering to this patient? A) Oxycodone B) Warfarin C) Morphine D) Acetaminophen

Ans: Morphine Feedback: The patient with suspected MI is given aspirin, nitroglycerin, morphine, an IV beta- blocker, and other medications, as indicated, while the diagnosis is being confirmed. Tylenol, warfarin, and oxycodone are not typically used.

An adult patient is admitted to the ED with chest pain. The patient states that he had developed unrelieved chest pain that was present for approximately 20 minutes before coming to the hospital. To minimize cardiac damage, the nurse should expect to administer which of the following interventions? A) Thrombolytics, oxygen administration, and nonsteroidal anti-inflammatories B) Morphine sulphate, oxygen, and bed rest C) Oxygen and beta-adrenergic blockers D) Bed rest, albuterol nebulizer treatments, and oxygen

Ans: Morphine sulphate, oxygen, and bed rest Feedback: The patient with suspected MI should immediately receive supplemental oxygen, aspirin, nitroglycerin, and morphine. Morphine sulphate reduces preload and decreases workload of the heart, along with increased oxygen from oxygen therapy and bed rest. With decreased cardiac demand, this provides the best chance of decreasing cardiac damage. NSAIDs and beta-blockers are not normally indicated. Albuterol, which is a medication used to manage asthma and respiratory conditions, will increase the heart rate.

The physical therapist notifies the nurse that a patient with coronary artery disease (CAD) experiences a much greater-than-average increase in heart rate during physical therapy. The nurse recognizes that an increase in heart rate in a patient with CAD may result in what? A) Development of an atrial-septal defect B) Myocardial ischemia C) Formation of a pulmonary embolism D) Release of potassium ions from cardiac cells

Ans: Myocardial ischemia Feedback: Unlike other arteries, the coronary arteries are perfused during diastole. An increase in heart rate shortens diastole and can decrease myocardial perfusion. Patients, particularly those with CAD, can develop myocardial ischemia. An increase in heart rate will not usually result in a pulmonary embolism or create electrolyte imbalances. Atrial-septal defects are congenital.

The student nurse is preparing a teaching plan for a patient being discharged status post MI. What should the student include in the teaching plan? (Mark all that apply.) A) Need for careful monitoring for cardiac symptoms B) Need for carefully regulated exercise C) Need for dietary modifications D) Need for early resumption of prediagnosis activity E) Need for increased fluid intake

Ans: Need for careful monitoring for cardiac symptoms, Need for carefully regulated exercise, Need for dietary modifications Feedback: Dietary modifications, exercise, weight loss, and careful monitoring are important strategies for managing three major cardiovascular risk factors: hyperlipidemia, hypertension, and diabetes. There is no need to increase fluid intake and activity should be slowly and deliberately increased.

A patient has been admitted with an aortic valve stenosis and has been scheduled for a balloon valvuloplasty in the cardiac catheterization lab later today. During the admission assessment, the patient tells the nurse he has thoracolumbar scoliosis and is concerned about lying down for any extended period of time. What is a priority action for the nurse? A) Arrange for an alternative bed. B) Measure the degree of the curvature. C) Notify the surgeon immediately. D) Note the scoliosis on the intake assessment.

Ans: Notify the surgeon immediately. Feedback: Most often used for mitral and aortic valve stenosis, balloon valvuloplasty is contraindicated for patients with left atrial or ventricular thrombus, severe aortic root dilation, significant mitral valve regurgitation, thoracolumbar scoliosis, rotation of the great vessels, and other cardiac conditions that require open heart surgery. Therefore notifying the physician would be the priority over further physical assessment. An alternative bed would be unnecessary and documentation is not a sufficient response.

An ED nurse is assessing an adult woman for a suspected MI. When planning the assessment, the nurse should be cognizant of what signs and symptoms of MI that are particularly common in female patients? Select all that apply. A) Shortness of breath B) Chest pain C) Anxiety D) Numbness E) Weakness

Ans: Numbness, Weakness Feedback: Although these symptoms are not wholly absent in men, many women have been found to have atypical symptoms of MI, including indigestion, nausea, palpitations, and numbness. Shortness of breath, chest pain, and anxiety are common symptoms of MI among patients of all ages and genders.

The nurse is providing an educational workshop about coronary artery disease (CAD) and its risk factors. The nurse explains to participants that CAD has many risk factors, some that can be controlled and some that cannot. What risk factors would the nurse list that can be controlled or modified? A) Gender, obesity, family history, and smoking B) Inactivity, stress, gender, and smoking C) Obesity, inactivity, diet, and smoking D) Stress, family history, and obesity

Ans: Obesity, inactivity, diet, and smoking Feedback: The risk factors for CAD that can be controlled or modified include obesity, inactivity, diet, stress, and smoking. Gender and family history are risk factors that cannot be controlled.

The nurse is preparing a patient for cardiac surgery. During the procedure, the patient's heart will be removed and a donor heart implanted at the vena cava and pulmonary veins. What procedure will this patient undergo? A) Orthotopic transplant B) Xenograft C) Heterotropic transplant D) Homograft

Ans: Orthotopic transplant Feedback: Orthotopic transplantation is the most common surgical procedure for cardiac transplantation. The recipient's heart is removed, and the donor heart is implanted at the vena cava and pulmonary veins. Some surgeons still prefer to remove the recipient's heart, leaving a portion of the recipient's atria (with the vena cava and pulmonary veins) in place. Homografts, or allografts (i.e., human valves), are obtained from cadaver tissue donations and are used for aortic and pulmonic valve replacement. Xenografts and heterotropic transplantation are not terms used to describe heart transplantation.

A patient calls his cardiologist's office and talks to the nurse. He is concerned because he feels he is being defibrillated too often. The nurse tells the patient to come to the office to be evaluated because the nurse knows that the most frequent complication of ICD therapy is what? A) Infection B) Failure to capture C) Premature battery depletion D) Oversensing of dysrhythmias

Ans: Oversensing of dysrhythmias Feedback: Inappropriate delivery of ICD therapy, usually due to oversensing of atrial and sinus tachycardias with a rapid ventricular rate response, is the most frequent complication of ICD. Infections, failure to capture, and premature battery failure are less common.

A nurse working in a long-term care facility is performing the admission assessment of a newly admitted, 85-year-old resident. During inspection of the resident's feet, the nurse notes that she appears to have early evidence of gangrene on one of her great toes. The nurse knows that gangrene in the elderly is often the first sign of what? A) Chronic venous insufficiency B) Raynaud's phenomenon C) VTE D) PAD

Ans: PAD Feedback: In elderly people, symptoms of PAD may be more pronounced than in younger people. In elderly patients who are inactive, gangrene may be the first sign of disease. Venous insufficiency does not normally manifest with gangrene. Similarly, VTE and Raynaud's phenomenon do not cause the ischemia that underlies gangrene.

A patient is recovering in the hospital from cardiac surgery. The nurse has identified the diagnosis of risk for ineffective airway clearance related to pulmonary secretions. What intervention best addresses this risk? A) Administration of bronchodilators by nebulizer B) Administration of inhaled corticosteroids by metered dose inhaler (MDI) C) Patient's consistent performance of deep breathing and coughing exercises D) Patient's active participation in the cardiac rehabilitation program

Ans: Patient's consistent performance of deep breathing and coughing exercises Feedback: Clearance of pulmonary secretions is accomplished by frequent repositioning of the patient, suctioning, and chest physical therapy, as well as educating and encouraging the patient to breathe deeply and cough. Medications are not normally used to achieve this goal. Rehabilitation is important, but will not necessarily aid the mobilization of respiratory secretions.

A critically ill patient is admitted to the ICU. The physician decides to use intra-arterial pressure monitoring. After this intervention is performed, what assessment should the nurse prioritize in the plan of care? A) Fluctuations in core body temperature B) Signs and symptoms of esophageal varices C) Signs and symptoms of compartment syndrome D) Perfusion distal to the insertion site

Ans: Perfusion distal to the insertion site Feedback: The radial artery is the usual site selected. However, placement of a catheter into the radial artery can further impede perfusion to an area that has poor circulation. As a result, the tissue distal to the cannulated artery can become ischemic or necrotic. Vigilant assessment is thus necessary. Alterations in temperature and the development of esophageal varices or compartment syndrome are not high risks.

The nurse is caring for patient who tells the nurse that he has an angina attack beginning. What is the nurse's most appropriate initial action? A) Have the patient sit down and put his head between his knees. B) Have the patient perform pursed-lip breathing. C) Have the patient stand still and bend over at the waist. D) Place the patient on bed rest in a semi-Fowler's position.

Ans: Place the patient on bed rest in a semi-Fowler's position. Feedback: When a patient experiences angina, the patient is directed to stop all activities and sit or rest in bed in a semi-Fowler's position to reduce the oxygen requirements of the ischemic myocardium. Pursed-lip breathing and standing will not reduce workload to the same extent. No need to have the patient put his head between his legs because cerebral perfusion is not lacking.

The physician has placed a central venous pressure (CVP) monitoring line in an acutely ill patient so right ventricular function and venous blood return can be closely monitored. The results show decreased CVP. What does this indicate? A) Possible hypovolemia B) Possible myocardial infarction (MI) C) Left-sided heart failure D) Aortic valve regurgitation

Ans: Possible hypovolemia Feedback: Hypovolemia may cause a decreased CVP. MI, valve regurgitation and heart failure are less likely causes of decreased CVP.

A cardiac surgery patient's new onset of signs and symptoms is suggestive of cardiac tamponade. As a member of the interdisciplinary team, what is the nurse's most appropriate action? A) Prepare to assist with pericardiocentesis. B) Reposition the patient into a prone position. C) Administer a dose of metoprolol. D) Administer a bolus of normal saline.

Ans: Prepare to assist with pericardiocentesis. Feedback: Cardiac tamponade requires immediate pericardiocentesis. Beta-blockers and fluid boluses will not relieve the pressure on the heart and prone positioning would likely exacerbate symptoms.

A patient presents to the ED in distress and complaining of "crushing" chest pain. What is the nurse's priority for assessment? A) Prompt initiation of an ECG B) Auscultation of the patient's point of maximal impulse (PMI) C) Rapid assessment of the patient's peripheral pulses D) Palpation of the patient's cardiac apex

Ans: Prompt initiation of an ECG Feedback: The 12-lead ECG provides information that assists in ruling out or diagnosing an acute MI. It should be obtained within 10 minutes from the time a patient reports pain or arrives in the ED. Each of the other listed assessments is valid, but ECG monitoring is the most time dependent priority.

The nurse is caring for a patient with a large venous leg ulcer. What intervention should the nurse implement to promote healing and prevent infection? A) Provide a high-calorie, high-protein diet. B) Apply a clean occlusive dressing once daily and whenever soiled. C) Irrigate the wound with hydrogen peroxide once daily. D) Apply an antibiotic ointment on the surrounding skin with each dressing change.

Ans: Provide a high-calorie, high-protein diet. Feedback: Wound healing is highly dependent on adequate nutrition. The diet should be sufficiently high in calories and protein. Antibiotic ointments are not normally used on the skin surrounding a leg ulcer and occlusive dressings can exacerbate impaired blood flow. Hydrogen peroxide is not normally used because it can damage granulation tissue.

The critical care nurse is caring for a patient who has had an MI. The nurse should expect to assist with establishing what hemodynamic monitoring procedure to assess the patient's left ventricular function? A) Central venous pressure (CVP) monitoring B) Pulmonary artery pressure monitoring (PAPM) C) Systemic arterial pressure monitoring (SAPM) D) Arterial blood gases (ABG)

Ans: Pulmonary artery pressure monitoring (PAPM) Feedback: PAPM is used to assess left ventricular function. CVP is used to assess right ventricular function; SAPM is used for continual assessment of BP. ABG are used to assess for acidic and alkalotic levels in the blood.

The nursing educator is presenting a case study of an adult patient who has abnormal ventricular depolarization. This pathologic change would be most evident in what component of the ECG? A) P wave B) T wave C) QRS complex D) U wave

Ans: QRS complex Feedback: The QRS complex represents the depolarization of the ventricles and, as such, the electrical activity of that ventricle.

The nurse is caring for a patient who has had an ECG. The nurse notes that leads I, II, and III differ from one another on the cardiac rhythm strip. How should the nurse best respond? A) Recognize that the view of the electrical current changes in relation to the lead placement. B) Recognize that the electrophysiological conduction of the heart differs with lead placement. C) Inform the technician that the ECG equipment has malfunctioned. D) Inform the physician that the patient is experiencing a new onset of dysrhythmia.

Ans: Recognize that the view of the electrical current changes in relation to the lead placement. Feedback: Each lead offers a different reference point to view the electrical activity of the heart. The lead displays the configuration of electrical activity of the heart. Differences between leads are not necessarily attributable to equipment malfunction or dysrhythmias.

The nurse is caring for a patient with refractory atrial fibrillation who underwent the maze procedure several months ago. The nurse reviews the result of the patient's most recent cardiac imaging, which notes the presence of scarring on the atria. How should the nurse best respond to this finding? A) Recognize that the procedure was unsuccessful. B) Recognize this as a therapeutic goal of the procedure. C) Liaise with the care team in preparation for repeating the maze procedure. D) Prepare the patient for pacemaker implantation.

Ans: Recognize this as a therapeutic goal of the procedure. Feedback: The maze procedure is an open heart surgical procedure for refractory atrial fibrillation. Small transmural incisions are made throughout the atria. The resulting formation of scar tissue prevents reentry conduction of the electrical impulse. Consequently, scar formation would constitute a successful procedure. There is no indication for repeating the procedure or implanting a pacemaker.

A community health nurse is presenting an educational event and is addressing several health problems, including rheumatic heart disease. What should the nurse describe as the most effective way to prevent rheumatic heart disease? A) Recognizing and promptly treating streptococcal infections B) Prophylactic use of calcium channel blockers in high-risk populations C) Adhering closely to the recommended child immunization schedule D) Smoking cessation

Ans: Recognizing and promptly treating streptococcal infections Feedback: Group A streptococcus can cause rheumatic heart fever, resulting in rheumatic endocarditis. Being aware of signs and symptoms of streptococcal infections, identifying them quickly, and treating them promptly, are the best preventative techniques for rheumatic endocarditis. Smoking cessation, immunizations, and calcium channel blockers will not prevent rheumatic heart disease.

A patient with cardiovascular disease is being treated with amlodipine (Norvasc), a calcium channel blocking agent. The therapeutic effects of calcium channel blockers include which of the following? A) Reducing the heart's workload by decreasing heart rate and myocardial contraction B) Preventing platelet aggregation and subsequent thrombosis C) Reducing myocardial oxygen consumption by blocking adrenergic stimulation to the heart D) Increasing the efficiency of myocardial oxygen consumption, thus decreasing ischemia and relieving pain

Ans: Reducing the heart's workload by decreasing heart rate and myocardial contraction Feedback: Calcium channel blocking agents decrease sinoatrial node automaticity and atrioventricular node conduction, resulting in a slower heart rate and a decrease in the strength of the heart muscle contraction. These effects decrease the workload of the heart. Antiplatelet and anticoagulation medications are administered to prevent platelet aggregation and subsequent thrombosis, which impedes blood flow. Beta-blockers reduce myocardial consumption by blocking beta-adrenergic sympathetic stimulation to the heart. The result is reduced myocardial contractility (force of contraction) to balance the myocardium oxygen needs and supply. Nitrates reduce myocardial oxygen consumption, which decreases ischemia and relieves pain by dilating the veins and, in higher doses, the arteries.

The nurse is caring for a patient with mitral stenosis who is scheduled for a balloon valvuloplasty. The patient tells the nurse that he is unsure why the surgeon did not opt to replace his damaged valve rather than repairing it. What is an advantage of valvuloplasty that the nurse should cite? A) The procedure can be performed on an outpatient basis in a physician's office. B) Repaired valves tend to function longer than replaced valves. C) The procedure is not associated with a risk for infection. D) Lower doses of antirejection drugs are required than with valve replacement.

Ans: Repaired valves tend to function longer than replaced valves. Feedback: In general, valves that undergo valvuloplasty function longer than prosthetic valve replacements and patients do not require continuous anticoagulation. Valvuloplasty carries a risk of infection, like all surgical procedures, and it is not performed in a physician's office. Antirejection drugs are unnecessary because foreign tissue is not introduced.

The nurse is caring for a patient who has had a biventricular pacemaker implanted. When planning the patient's care, the nurse should recognize what goal of this intervention? A) Resynchronization B) Defibrillation C) Angioplasty D) Ablation

Ans: Resynchronization Feedback: Biventricular (both ventricles) pacing, also called resynchronization therapy, may be used to treat advanced heart failure that does not respond to medication. This type of pacing therapy is not called defibrillation, angioplasty, or ablation therapy.

The nurse is caring for a recent immigrant who has been diagnosed with mitral valve regurgitation. The nurse should know that in developing countries the most common cause of mitral valve regurgitation is what? A) A decrease in gamma globulins B) An insect bite C) Rheumatic heart disease and its sequelae D) Sepsis and its sequelae

Ans: Rheumatic heart disease and its sequelae Feedback: The most common cause of mitral valve regurgitation in developing countries is rheumatic heart disease and its sequelae.

A patient has undergone diagnostic testing and received a diagnosis of sinus bradycardia attributable to sinus node dysfunction. When planning this patient's care, what nursing diagnosis is most appropriate? A) Acute pain B) Risk for unilateral neglect C) Risk for activity intolerance D) Risk for fluid volume excess

Ans: Risk for activity intolerance Feedback: Sinus bradycardia causes decreased cardiac output that is likely to cause activity intolerance. It does not typically cause pain, fluid imbalances, or neglect of a unilateral nature.

The nurse's assessment of an older adult client reveals the following data: Lying BP 144/82 mm Hg; sitting BP 121/69 mm Hg; standing BP 98/56 mm Hg. The nurse should consequently identify what nursing diagnosis in the patient's plan of care? A) Risk for ineffective breathing pattern related to hypotension B) Risk for falls related to orthostatic hypotension C) Risk for ineffective role performance related to hypotension D) Risk for imbalanced fluid balance related to hemodynamic variability

Ans: Risk for falls related to orthostatic hypotension Feedback: Orthostatic hypotension creates a significant risk for falls due to the dizziness and lightheadedness that accompanies it. It does not normally affect breathing or fluid balance. The patient's ability to perform normal roles may be affected, but the risk for falls is the most significant threat to safety.

A patient has undergone a successful heart transplant and has been discharged home with a medication regimen that includes cyclosporine and tacrolimus. In light of this patient's medication regimen, what nursing diagnosis should be prioritized? A) Risk for injury B) Risk for infection C) Risk for peripheral neurovascular dysfunction D) Risk for unstable blood glucose

Ans: Risk for infection Feedback: Immunosuppressants decrease the body's ability to resist infections, and a satisfactory balance must be achieved between suppressing rejection and avoiding infection. These drugs do not create a heightened risk of injury, neurovascular dysfunction, or unstable blood glucose levels.

The nurse is caring for a patient who returned from the tropics a few weeks ago and who sought care with signs and symptoms of lymphedema. The nurse's plan of care should prioritize what nursing diagnosis? A) Risk for infection related to lymphedema B) Disturbed body image related to lymphedema C) Ineffective health maintenance related to lymphedema D) Risk for deficient fluid volume related to lymphedema

Ans: Risk for infection related to lymphedema Feedback: Lymphedema, which is caused by accumulation of lymph in the tissues, constitutes a significant risk for infection. The patient's body image is likely to be disturbed, and the nurse should address this, but infection is a more significant threat to the patient's physiological well-being. Lymphedema is unrelated to ineffective health maintenance and deficient fluid volume is not a significant risk.

The critical care nurse is caring for a patient who has been experiencing bradycardia after cardiovascular surgery. The nurse knows that the heart rate is determined by myocardial cells with the fastest inherent firing rate. Under normal circumstances where are these cells located? A) SA node B) AV node C) Bundle of His D) Purkinje cells

Ans: SA node Feedback: The heart rate is determined by the myocardial cells with the fastest inherent firing rate. Under normal circumstances, the SA node has the highest inherent rate (60 to 100 impulses per minute).

The cardiac care nurse is reviewing the conduction system of the heart. The nurse is aware that electrical conduction of the heart usually originates in the SA node and then proceeds in what sequence? A) SA node to bundle of His to AV node to Purkinje fibers B) SA node to AV node to Purkinje fibers to bundle of His C) SA node to bundle of His to Purkinje fibers to AV node D) SA node to AV node to bundle of His to Purkinje fibers

Ans: SA node to AV node to bundle of His to Purkinje fibers Feedback: The normal electrophysiological conduction route is SA node to AV node to bundle of HIS to Purkinje fibers.

A patient is admitted to the critical care unit (CCU) with a diagnosis of cardiomyopathy. When reviewing the patient's most recent laboratory results, the nurse should prioritize assessment of which of the following? A) Sodium B) AST, ALT, and bilirubin C) White blood cell differential D) BUN

Ans: Sodium Feedback: Sodium is the major electrolyte involved with cardiomyopathy. Cardiomyopathy often leads to heart failure which develops, in part, from fluid overload. Fluid overload is often associated with elevated sodium levels. Consequently, sodium levels are followed more closely than other important laboratory values, including BUN, leukocytes, and liver function tests.

The nurse is caring for a patient who is undergoing an exercise stress test. Prior to reaching the target heart rate, the patient develops chest pain. What is the nurse's most appropriate response? A) Administer sublingual nitroglycerin to allow the patient to finish the test. B) Initiate cardiopulmonary resuscitation. C) Administer analgesia and slow the test. D) Stop the test and monitor the patient closely.

Ans: Stop the test and monitor the patient closely. Feedback: Signs of myocardial ischemia would necessitate stopping the test. CPR would only be necessary if signs of cardiac or respiratory arrest were evident.

A nurse is planning discharge health education for a patient who will soon undergo placement of a mechanical valve prosthesis. What aspect of health education should the nurse prioritize in anticipation of discharge? A) The need for long-term antibiotics B) The need for 7 to 10 days of bed rest C) Strategies for preventing atherosclerosis D) Strategies for infection prevention

Ans: Strategies for infection prevention Feedback: Patients with a mechanical valve prosthesis (including annuloplasty rings and other prosthetic materials used in valvuloplasty) require education to prevent infective endocarditis. Despite these infections risks, antibiotics are not used long term. Activity management is important, but extended bed rest is unnecessary. Valve replacement does not create a heightened risk for atherosclerosis.

A nurse is closely monitoring a patient who has recently been diagnosed with an abdominal aortic aneurysm. What assessment finding would signal an impending rupture of the patient's aneurysm? A) Sudden increase in blood pressure and a decrease in heart rate B) Cessation of pulsating in an aneurysm that has previously been pulsating visibly C) Sudden onset of severe back or abdominal pain D) New onset of hemoptysis

Ans: Sudden onset of severe back or abdominal pain Feedback: Signs of impending rupture include severe back or abdominal pain, which may be persistent or intermittent. Impending rupture is not typically signaled by increased blood pressure, bradycardia, cessation of pulsing, or hemoptysis.

A nurse is describing the process by which blood is ejected into circulation as the chambers of the heart become smaller. The instructor categorizes this action of the heart as what? A) Systole B) Diastole C) Repolarization D) Ejection fraction

Ans: Systole Feedback: Systole is the action of the chambers of the heart becoming smaller and ejecting blood. This action of the heart is not diastole (relaxations), ejection fraction (the amount of blood expelled), or repolarization (electrical charging).

The nurse is analyzing a rhythm strip. What component of the ECG corresponds to the resting state of the patient's heart? A) P wave B) T wave C) U wave D) QRS complex

Ans: T wave Feedback: The T wave specifically represents ventricular muscle depolarization, also referred to as the resting state. Ventricular muscle depolarization does not result in the P wave, U wave, or QRS complex.

The nurse is caring for a patient who is believed to have just experienced an MI. The nurse notes changes in the ECG of the patient. What change on an ECG most strongly suggests to the nurse that ischemia is occurring? A) P wave inversion B) T wave inversion C) Q wave changes with no change in ST or T wave D) P wave enlargement

Ans: T wave inversion Feedback: T-wave inversion is an indicator of ischemic damage to myocardium. Typically, few changes to P waves occur during or after an MI, whereas Q-wave changes with no change in the ST or T wave indicate an old MI.

The nurse is assessing a woman who is pregnant at 27 weeks' gestation. The patient is concerned about the recent emergence of varicose veins on the backs of her calves. What is the nurse's best response? A) Facilitate a referral to a vascular surgeon. B) Assess the patient's ankle-brachial index (ABI) and perform Doppler ultrasound testing. C) Encourage the patient to increase her activity level. D) Teach the patient that circulatory changes during pregnancy frequently cause varicose veins.

Ans: Teach the patient that circulatory changes during pregnancy frequently cause varicose veins. Feedback: Pregnancy may cause varicosities because of hormonal effects related to decreased venous outflow, increased pressure by the gravid uterus, and increased blood volume. In most cases, no intervention or referral is necessary. This finding is not an indication for ABI assessment and increased activity will not likely resolve the problem.

A nurse is reviewing the physiological factors that affect a patient's cardiovascular health and tissue oxygenation. What is the systemic arteriovenous oxygen difference? A) The average amount of oxygen removed by each organ in the body B) The amount of oxygen removed from the blood by the heart C) The amount of oxygen returning to the lungs via the pulmonary artery D) The amount of oxygen in aortic blood minus the amount of oxygen in the vena caval blood

Ans: The amount of oxygen in aortic blood minus the amount of oxygen in the vena caval blood Feedback: The average amount of oxygen removed collectively by all of the body tissues is about 25%. This means that the blood in the vena cava contains about 25% less oxygen than aortic blood. This is known as the systemic arteriovenous oxygen difference. The other answers do not apply.

A patient's declining cardiac status has been attributed to decreased cardiac action potential. Interventions will be aimed at restoring what aspect of cardiac physiology? A) The cycle of depolarization and repolarization B) The time it takes from the firing of the SA node to the contraction of the ventricles C) The time between the contraction of the atria and the contraction of the ventricles D) The cycle of the firing of the AV node and the contraction of the myocardium

Ans: The cycle of depolarization and repolarization Feedback: This exchange of ions creates a positively charged intracellular space and a negatively charged extracellular space that characterizes the period known as depolarization. Once depolarization is complete, the exchange of ions reverts to its resting state; this period is known as repolarization. The repeated cycle of depolarization and repolarization is called the cardiac action potential.

Graduated compression stockings have been prescribed to treat a patient's venous insufficiency. What education should the nurse prioritize when introducing this intervention to the patient? A) The need to take anticoagulants concurrent with using compression stockings B) The need to wear the stockings on a "one day on, one day off" schedule C) The importance of wearing the stockings around the clock to ensure maximum benefit D) The importance of ensuring the stockings are applied evenly with no pressure points

Ans: The importance of ensuring the stockings are applied evenly with no pressure points Feedback: Any type of stocking can inadvertently become a tourniquet if applied incorrectly (i.e., rolled tightly at the top). In such instances, the stockings produce rather than prevent stasis. For ambulatory patients, graduated compression stockings are removed at night and reapplied before the legs are lowered from the bed to the floor in the morning. They are used daily, not on alternating days. Anticoagulants are not always indicated in patients who are using compression stockings.

.A patient who has undergone valve replacement surgery is being prepared for discharge home. Because the patient will be discharged with a prescription for warfarin (Coumadin), the nurse should educate the patient about which of the following? A) The need for regularly scheduled testing of the patient's International Normalized Ratio (INR) B) The need to learn to sleep in a semi-Fowler's position for the first 6 to 8 weeks to prevent emboli C) The need to avoid foods that contain vitamin K D) The need to take enteric-coated ASA on a daily basis

Ans: The need for regularly scheduled testing of the patient's International Normalized Ratio (INR) Feedback: Patients who take warfarin (Coumadin) after valve replacement have individualized target INRs; usually between 2 and 3.5 for mitral valve replacement and 1.8 and 2.2 for aortic valve replacement. Natural sources of vitamin K do not normally need to be avoided and ASA is not indicated. Sleeping upright is unnecessary.

The nurse is caring for a patient with acute pericarditis. What nursing management should be instituted to minimize complications? A) The nurse keeps the patient isolated to prevent nosocomial infections. B) The nurse encourages coughing and deep breathing. C) The nurse helps the patient with activities until the pain and fever subside. D) The nurse encourages increased fluid intake until the infection resolves.

Ans: The nurse helps the patient with activities until the pain and fever subside. Feedback: To minimize complications, the nurse helps the patient with activity restrictions until the pain and fever subside. As the patient's condition improves, the nurse encourages gradual increases of activity. Actions to minimize complications of acute pericarditis do not include keeping the patient isolated. Due to pain, coughing and deep breathing are not normally encouraged. An increase in fluid intake is not always necessary.

A resident of a long-term care facility has complained to the nurse of chest pain. What aspect of the resident's pain would be most suggestive of angina as the cause? A) The pain is worse when the resident inhales deeply. B) The pain occurs immediately following physical exertion. C) The pain is worse when the resident coughs. D) The pain is most severe when the resident moves his upper body.

Ans: The pain occurs immediately following physical exertion. Feedback: Chest pain associated with angina is often precipitated by physical exertion. The other listed aspects of chest pain are more closely associated with noncardiac etiologies.

The nurse is working with a patient who had an MI and is now active in rehabilitation. The nurse should teach this patient to cease activity if which of the following occurs? A) The patient experiences chest pain, palpitations, or dyspnea. B) The patient experiences a noticeable increase in heart rate during activity. C) The patient's oxygen saturation level drops below 96%. D) The patient's respiratory rate exceeds 30 breaths/min.

Ans: The patient experiences chest pain, palpitations, or dyspnea. Feedback: Any activity or exercise that causes dyspnea and chest pain should be stopped in the patient with CAD. Heart rate must not exceed the target rate, but an increase above resting rate is expected and is therapeutic. In most patients, a respiratory rate that exceeds 30 breaths/min is not problematic. Similarly, oxygen saturation slightly below 96% does not necessitate cessation of activity.

When assessing a patient diagnosed with angina pectoris it is most important for the nurse to gather what information? A) The patient's activities limitations and level of consciousness after the attacks B) The patient's symptoms and the activities that precipitate attacks C) The patient's understanding of the pathology of angina D) The patient's coping strategies surrounding the attacks

Ans: The patient's symptoms and the activities that precipitate attacks Feedback: The nurse must gather information about the patient's symptoms and activities, especially those that precede and precipitate attacks of angina pectoris. The patient's coping, understanding of the disease, and status following attacks are all important to know, but causative factors are a primary focus of the assessment interview.

Family members bring a patient to the ED with pale cool skin, sudden midsternal chest pain unrelieved with rest, and a history of CAD. How should the nurse best interpret these initial data? A) The symptoms indicate angina and should be treated as such. B) The symptoms indicate a pulmonary etiology rather than a cardiac etiology. C) The symptoms indicate an acute coronary episode and should be treated as such. D) Treatment should be determined pending the results of an exercise stress test.

Ans: The symptoms indicate an acute coronary episode and should be treated as such. Feedback: Angina and MI have similar symptoms and are considered the same process, but are on different points along a continuum. That the patient's symptoms are unrelieved by rest suggests an acute coronary episode rather than angina. Pale cool skin and sudden onset are inconsistent with a pulmonary etiology. Treatment should be initiated immediately regardless of diagnosis.

A patient with a complex cardiac history is scheduled for transthoracic echocardiography. What should the nurse teach the patient in anticipation of this diagnostic procedure? A) The test is noninvasive, and nothing will be inserted into the patient's body. B) The patient's pain will be managed aggressively during the procedure. C) The test will provide a detailed profile of the heart's electrical activity. D) The patient will remain on bed rest for 1 to 2 hours after the test.

Ans: The test is noninvasive, and nothing will be inserted into the patient's body. Feedback: Before transthoracic echocardiography, the nurse informs the patient about the test, explaining that it is painless. The test does not evaluate electrophysiology and bed rest is unnecessary after the procedure.

A group of nurses are participating in orientation to a telemetry unit. What should the staff educator tell this class about ST segments? A) They are the part of an ECG that reflects systole. B) They are the part of an ECG used to calculate ventricular rate and rhythm. C) They are the part of an ECG that reflects the time from ventricular depolarization through repolarization. D) They are the part of an ECG that represents early ventricular repolarization.

Ans: They are the part of an ECG that represents early ventricular repolarization. Feedback: ST segment is the part of an ECG that reflects the end of the QRS complex to the beginning of the T wave. The part of an ECG that reflects repolarization of the ventricles is the T wave. The part of an ECG used to calculate ventricular rate and rhythm is the RR interval. The part of an ECG that reflects the time from ventricular depolarization through repolarization is the QT interval.

The nurse is caring for a patient admitted with unstable angina. The laboratory result for the initial troponin I is elevated in this patient. The nurse should recognize what implication of this assessment finding? A) This is only an accurate indicator of myocardial damage when it reaches its peak in 24 hours. B) Because the patient has a history of unstable angina, this is a poor indicator of myocardial injury. C) This is an accurate indicator of myocardial injury. D) This result indicates muscle injury, but does not specify the source.

Ans: This is an accurate indicator of myocardial injury. Feedback: Troponin I, which is specific to cardiac muscle, is elevated within hours after myocardial injury. Even with a diagnosis of unstable angina, this is an accurate indicator of myocardial injury.

A patient with angina has been prescribed nitroglycerin. Before administering the drug, the nurse should inform the patient about what potential adverse effects? A) Nervousness or paresthesia B) Throbbing headache or dizziness C) Drowsiness or blurred vision D) Tinnitus or diplopia

Ans: Throbbing headache or dizziness Feedback: Headache and dizziness commonly occur when nitroglycerin is taken at the beginning of therapy. Nervousness, paresthesia, drowsiness, blurred vision, tinnitus, and diplopia do not typically occur as a result of nitroglycerin therapy.

The critical care nurse is caring for a patient with a pulmonary artery pressure monitoring system. The nurse is aware that pulmonary artery pressure monitoring is used to assess left ventricular function. What is an additional function of pulmonary artery pressure monitoring systems? A) To assess the patient's response to fluid and drug administration B) To obtain specimens for arterial blood gas measurements C) To dislodge pulmonary emboli D) To diagnose the etiology of chronic obstructive pulmonary disease

Ans: To assess the patient's response to fluid and drug administration Feedback: Pulmonary artery pressure monitoring is an important tool used in critical care for assessing left ventricular function (cardiac output), diagnosing the etiology of shock, and evaluating the patient's response to medical interventions, such as fluid administration and vasoactive medications. Pulmonary artery monitoring is preferred for the patient with heart failure over central venous pressure monitoring. Arterial catheters are useful when arterial blood gas measurements and blood samples need to be obtained frequently. Neither intervention is used to clear pulmonary emboli.

A patient with mitral valve prolapse is admitted for a scheduled bronchoscopy to investigate recent hemoptysis. The physician has ordered gentamicin to be taken before the procedure. What is the rationale for this? A) To prevent bacterial endocarditis B) To prevent hospital-acquired pneumonia C) To minimize the need for antibiotic use during the procedure D) To decrease the need for surgical asepsis

Ans: To prevent bacterial endocarditis Feedback: Antibiotic prophylaxis is recommended for high-risk patients immediately before and sometimes after the following invasive procedures, such as bronchoscopy. Gentamicin would not be given to prevent pneumonia, to avoid antibiotic use during the procedure, or to decrease the need for surgical asepsis.

The nurse is teaching a patient diagnosed with aortic stenosis appropriate strategies for attempting to relieve the symptom of angina without drugs. What should the nurse teach the patient? A) To eat a small meal before taking nitroglycerin B) To drink a glass of milk before taking nitroglycerin C) To engage in 15 minutes of light exercise before taking nitroglycerin D) To rest and relax before taking nitroglycerin

Ans: To rest and relax before taking nitroglycerin Feedback: The venous dilation that results from nitroglycerin decreases blood return to the heart, thus decreasing cardiac output and increasing the risk of syncope and decreased coronary artery blood flow. The nurse teaches the patient about the importance of attempting to relieve the symptoms of angina with rest and relaxation before taking nitroglycerin and to anticipate the potential adverse effects. Exercising, eating, and drinking are not recommended prior to using nitroglycerin.

The nurse caring for a patient whose sudden onset of sinus bradycardia is not responding adequately to atropine. What might be the treatment of choice for this patient? A) Implanted pacemaker B) Trancutaneous pacemaker C) ICD D) Asynchronous defibrillator

Ans: Trancutaneous pacemaker Feedback: If a patient suddenly develops a bradycardia, is symptomatic but has a pulse, and is unresponsive to atropine, emergency pacing may be started with transcutaneous pacing, which most defibrillators are now equipped to perform. An implanted pacemaker is not a time-appropriate option. An asynchronous defibrillator or ICD would not provide relief.

The nurse is providing care for a patient who has just been diagnosed with peripheral arterial occlusive disease (PAD). What assessment finding is most consistent with this diagnosis? A) Numbness and tingling in the distal extremities B) Unequal peripheral pulses between extremities C) Visible clubbing of the fingers and toes D) Reddened extremities with muscle atrophy

Ans: Unequal peripheral pulses between extremities Feedback: PAD assessment may manifest as unequal pulses between extremities, with the affected leg cooler and paler than the unaffected leg. Intermittent claudication is far more common than sensations of numbness and tingling. Clubbing and muscle atrophy are not associated with PAD.

A patient presents to the clinic complaining of the inability to grasp objects with her right hand. The patient's right arm is cool and has a difference in blood pressure of more than 20 mm Hg compared with her left arm. The nurse should expect that the primary care provider may diagnose the woman with what health problem? A) Lymphedema B) Raynaud's phenomenon C) Upper extremity arterial occlusive disease D) Upper extremity VTE

Ans: Upper extremity arterial occlusive disease Feedback: The patient with upper extremity arterial occlusive disease typically complains of arm fatigue and pain with exercise (forearm claudication) and inability to hold or grasp objects (e.g., combing hair, placing objects on shelves above the head) and, occasionally, difficulty driving. Assessment findings include coolness and pallor of the affected extremity, decreased capillary refill, and a difference in arm blood pressures of more than 20 mm Hg. These symptoms are not closely associated with Raynaud's or lymphedema. The upper extremities are rare sites for VTE.

A nurse is assessing a new patient who is diagnosed with PAD. The nurse cannot feel the pulse in the patient's left foot. How should the nurse proceed with assessment? A) Have the primary care provider order a CT. B) Apply a tourniquet for 3 to 5 minutes and then reassess. C) Elevate the extremity and attempt to palpate the pulses. D) Use Doppler ultrasound to identify the pulses.

Ans: Use Doppler ultrasound to identify the pulses. Feedback: When pulses cannot be reliably palpated, a hand-held continuous wave (CW) Doppler ultrasound device may be used to hear (insonate) the blood flow in vessels. CT is not normally warranted and the application of a tourniquet poses health risks and will not aid assessment. Elevating the extremity would make palpation more difficult.

The staff educator is teaching a CPR class. Which of the following aspects of defibrillation should the educator stress to the class? A) Apply the paddles directly to the patient's skin. B) Use a conducting medium between the paddles and the skin. C) Always use a petroleum-based gel between the paddles and the skin. D) Any available liquid can be used between the paddles and the skin.

Ans: Use a conducting medium between the paddles and the skin. Feedback: Use multifunction conductor pads or paddles with a conducting medium between the paddles and the skin (the conducting medium is available as a sheet, gel, or paste). Do not use gels or pastes with poor electrical conductivity.

A nurse is providing health education to a patient scheduled for cryoablation therapy. The nurse should describe what aspect of this treatment? A) Peeling away the area of endocardium responsible for the dysrhythmia B) Using electrical shocks directly to the endocarduim to eliminate the source of dysrhythmia C) Using high-frequency sound waves to eliminate the source of dysrhythmia D) Using a cooled probe to eliminate the source of dysrhythmia

Ans: Using a cooled probe to eliminate the source of dysrhythmia Feedback: Cryoablation therapy involves using a cooled probe to create a small scar on the endocardium to eliminate the source of the dysrhythmias. Endocardium resection involves peeling away a specified area of the endocardium. Electrical ablation involves using shocks to eliminate the area causing the dysrhythmias. Radio frequency ablation uses high-frequency sound waves to destroy the area causing the dysrhythmias.

A patient the nurse is caring for has a permanent pacemaker implanted with the identification code beginning with VVI. What does this indicate? A) Ventricular paced, ventricular sensed, inhibited B) Variable paced, ventricular sensed, inhibited C) Ventricular sensed, ventricular situated, implanted D) Variable sensed, variable paced, inhibit

Ans: Ventricular paced, ventricular sensed, inhibited Feedback: The identification of VVI indicates ventricular paced, ventricular sensed, inhibited.

The nurse is caring for a patient who has just had an implantable cardioverter defibrillator (ICD) placed. What is the priority area for the nurse's assessment? A) Assessing the patient's activity level B) Facilitating transthoracic echocardiography C) Vigilant monitoring of the patient's ECG D) Close monitoring of the patient's peripheral perfusion

Ans: Vigilant monitoring of the patient's ECG Feedback: After a permanent electronic device (pacemaker or ICD) is inserted, the patient's heart rate and rhythm are monitored by ECG. This is a priority over peripheral circulation and activity. Echocardiography is not indicated.

An occupational health nurse is providing an educational event and has been asked by an administrative worker about the risk of varicose veins. What should the nurse suggest as a proactive preventative measure for varicose veins? A) Sit with crossed legs for a few minutes each hour to promote relaxation. B) Walk for several minutes every hour to promote circulation. C) Elevate the legs when tired. D) Wear snug-fitting ankle socks to decrease edema.

Ans: Walk for several minutes every hour to promote circulation. Feedback: A proactive approach to preventing varicose veins would be to walk for several minutes every hour to promote circulation. Sitting with crossed legs may promote relaxation, but it is contraindicated for patients with, or at risk for, varicose veins. Elevating the legs only helps blood passively return to the heart and does not help maintain the competency of the valves in the veins. Wearing tight ankle socks is contraindicated for patients with, or at risk for, varicose veins; socks that are below the muscles of the calf do not promote venous return, the socks simply capture the blood and promote venous stasis.

The nurse is performing an intake assessment on a patient with a new diagnosis of coronary artery disease. What would be the most important determination to make during this intake assessment? A) Whether the patient and involved family members understand the role of genetics in the etiology of the disease B) Whether the patient and involved family members understand dietary changes and the role of nutrition C) Whether the patient and involved family members are able to recognize symptoms of an acute cardiac problem and respond appropriately D) Whether the patient and involved family members understand the importance of social support and community agencies

Ans: Whether the patient and involved family members are able to recognize symptoms of an acute cardiac problem and respond appropriately Feedback: During the health history, the nurse needs to determine if the patient and involved family members are able to recognize symptoms of an acute cardiac problem, such as acute coronary syndrome (ACS) or HF, and seek timely treatment for these symptoms. Each of the other listed topics is valid, but the timely and appropriate response to a cardiac emergency is paramount.

The nurse is caring for an 82-year-old patient. The nurse knows that changes in cardiac structure and function occur in older adults. What is a normal change expected in the aging heart of an older adult? A) Decreased left ventricular ejection time B) Decreased connective tissue in the SA and AV nodes and bundle branches C) Thinning and flaccidity of the cardiac values D) Widening of the aorta

Ans: Widening of the aorta Feedback: Changes in cardiac structure and function are clearly observable in the aging heart. Aging results in decreased elasticity and widening of the aorta, thickening and rigidity of the cardiac valves, increased connective tissue in the SA and AV nodes and bundle branches, and an increased left ventricular ejection time (prolonged systole).

The nurse is caring for a patient who is scheduled for cardiac surgery. What should the nurse include in preoperative care? A) With the patient, clarify the surgical procedure that will be performed. B) Withhold the patient's scheduled medications for at least 12 hours preoperatively. C) Inform the patient that health teaching will begin as soon as possible after surgery. D) Avoid discussing the patient's fears as not to exacerbate them.

Ans: With the patient, clarify the surgical procedure that will be performed. Feedback: Preoperatively, it is necessary to evaluate the patient's understanding of the surgical procedure, informed consent, and adherence to treatment protocols. Teaching would begin on admission or even prior to admission. The physician would write orders to alter the patient's medication regimen if necessary; this will vary from patient to patient. Fears should be addressed directly and empathically.

The nurse is caring for a patient on telemetry. The patient's ECG shows a shortened PR interval, slurring of the initial QRS deflection, and prolonged QRS duration. What does this ECG show? A) Sinus bradycardia B) Myocardial infarction C) Lupus-like syndrome D) Wolf-Parkinson-White (WPW) syndrome

Ans: Wolf-Parkinson-White (WPW) syndrome Feedback: In WPW syndrome there is a shortened PR interval, slurring (called a delta wave) of the initial QRS deflection, and prolonged QRS duration. These characteristics are not typical of the other listed cardiac anomalies.


Ensembles d'études connexes

EXPLAINED! Despite, In spite of, Although & Even though | English Grammar

View Set

Quiz 5- South and Southeast Asia

View Set

Chapter 10 - The Foreign Exchange Market

View Set

chapter 7- Bipolar and Depressive Disorders

View Set

AP Gov Civil Rights and Liberties

View Set

Accounting Final Multiple Choice Ch 10,11,13

View Set

ANTH 205 Lecture Four: Middle Range-theory, Experimental, and Ethnoarchaeology

View Set

Quiz 1: The Older Adult & Aging Well

View Set

HR Chapter 01: Mini Case: Organizational Culture Gone Wrong

View Set